82
Pq Ρ q Α PQ q Ρ· ' Γ Ελληνική Μαθηματική Εταιρεία

Ευκλειδης Β 58

  • Upload
    demi-de

  • View
    242

  • Download
    0

Embed Size (px)

DESCRIPTION

 

Citation preview

Page 1: Ευκλειδης Β 58

P=>q Ρ

q Α

P=>Q

q Ρ· '

Γ

Ελληνική Μαθηματική Εταιρεία

Page 2: Ευκλειδης Β 58

yια μαθητές και εκπαιδευτικούς

ΜΑΡΙΑ ΕΥΣ:fΑθΙΟV + ΕΛΕΥθΕΡΙΟΙ ΠΡΟΤΟf'!ΑΠΑΙ

σ' ενιοΙοu Λuκεlοu

Μεθοδολογία Άλγεβρας Α' Ενιαίου Λυκείου

Μ. Ευσταθίου, Ε. Πρωτοπαπάς

Γεωμετρία Β' Ενιαίου Λυκείου

Γ ενικής Παιδείας

Ε. Πρωτοπαπάς

ΕΚΔΟΣΕΙΣ ΠΑΤΑΚΗ www.patakis.qr

'

a .�·�

Γεωμετρία Α' Ενιαίου Λυκείου Γ. Βιδάλης, Β. Γ κιμίσης

μαθηματικά Γ' Ενιαίου Λυκείου

Μαθηματικά

iμ ·

;/'

Άλγεβρα Β' Ενιαίου Λυκείου

Γενικής Παιδείας (2 τόμοι) Γ. Βιδάλη ς, Β. Γ κιμίσης

*

μαθηματικά �· τόμος

θΕΠΚΗΣ ΚΑΙ ΤΕΧΝΟΛΟΓ!ΚΗΣ ΚΑτΕΎθΥΝΣΗΣ """ �

Μαθηματικά Γ Ενιαίου Λυκείου Γ Ενιαίου Λυκείου

Γ ενικής Παιδείας

Μ. Τ σιλπιρίδης

Θετικής και τεχνολογικής κατεύθυνσης

Α' τόμος: Ε. Πρωτοπαπάς

Β· τόμος: Ε. Πρωτοπαπάς, Σ. Γκούμας

σε όλο το βιβλιοπωλείο ΒιΒΛιΟΠΩΛΕΙΟ ΠΑΤΑΚΗ: ΑΚΑΔΗΜΙΑΣ 65, I 06 78 ΑΘΗΝΑ, ΤΗΛ. 2ι 0.38.Ιι.850

KENTPIKH ΔΙΑΘΕΣΗ: ΕΜΜ. ΜΠΕΝΑΚΗ 16, I 06 78 ΑΘΗΝΑ. ΤΗΛ. 210.38.31.078

ΘΕΣΣΑΛΟΝΙΚΗ: Ν. ΜΟΝΑΗΗΡΙΟΥ 122, ΤΗΛ. 2310.70.63.54-5

LΕτRιΝΑ SA. CEAτii"ICAτtON BODY

ISO 9001 •'<"rflfiCΙΙI�'

'\',, 1�. Ι.\� ,'i,; 1.17

Page 3: Ευκλειδης Β 58

ΕΛΛΗΝΙΚΗ ΜΑΘΗΜΑΤΙΚΗ ETAIPEIA Τεύχος 58 • Οκτώβριος • Νοέμβριος • Δεκέμβριος 2005 • Ευρώ: 3,50

e-mail: [email protected] www.hms.gr

ΜΑθΗΜΑ11ΚΟ ΠΕΡΙΟΔΙΚΟ ΠΑ το ΛΥΚΕΙΟ

ΠΕΡΙΕΧΟΜΕΝΑ ../ Εκδήλωση της Ε.Μ.Ε. . ............................................ 2 ../ Στοιχεία Μαθηματικής Λοyικής . . . . . . . . . . . . . . . . . . . . . . . . .. . . . . . . . . . . . . . . 8 ../ Homo Mathematicus ........................................................ 15 ../ Μαθηματικοί Διαyωνισμοί- Μαθηματικές Ολυμπιάδες . . . . . . . .. . . . . . . . . . . . . . . . . . . . 19

Μαθηματικά Α' Τάξης ../ Άλyεβρα ................................................ 26 ../ Σύνολα. rενικά περί συναρτήσεων ..... .... ..... ... .. ... . .. . ..... .. ... .... ... 30 ../ Παραλληλόyραμμα .. ... ................... . . ... . ... . . . . ... .. .. ...... .. .. ... 34

Μαθηματικά Β' Τάξης ../ Άλyεβρα: Πολυώνυμα - Εξισώσεις - Ανισώσεις .... .

../ rεωμετρία: Εμβαδά ................................. .

../ Ασκήσεις στην ευθ εία και στον κύκλο ............... .

Μαθηματικά Γ' Τάξης ../ Συναρτήσεις ....................... .

../ Ασκήσεις στα όρια ...................... . ../ Συνέ χεια σε κλειστό διάστημα και εφαρμοyές της .. ../ Μαθηματικά rενικής Παιδείας: Ασκήσεις Στατιστικής ..

. .. 37

. . .40 . . . . . . . . . . . . 47

. .. 52

. .. 59

. .. 62 . ........... 65

../ Ο Ευκλείδης προτ είνει ... Ευκλείδη και ... Διόφαvτο ....................... 72 ../ Στήλη του μαθητή . . . . . . . . . . . . . . . . . . . . . . . . . . . . . . . . . . . . . . . . . . . . . . 7 4

••••••••••••••••••••••••••••••••••••••••••••••••••••••••••••••••••••••••••••••

ΕΚΔΟΣΗ ΤΗΣ ΜΑΘΗΜΑΥΙΚΗΣ ΕΥΑΙΡΕΙΑΣ

ΓΙΑΝΕΠΙΠΗΜΙΟΥ 34 • 1 06 79 ΑΘΗΝΑ Τηλ.: 21 Ο 3617784 • 3616532 Fox:2103641025

Εκδ6της: Εξαρχ6κος Θε6δωρος

Διευθυντής: Τυρλής Ιω6ννης

Κωδικός ΕΛ.ΤΑ.: 2055 ISSN: 1105- 7998

Επιμέλεια 'Εκδοσηc;: Κυριακόπουλος Αντώνης Ευσταθίου Βαγγέλης

Εκτελεστική Γραμματεία

Πρ6εδρος:

Κuριακ6ποuλος Αντώνης

Αντιπρ6εδροι:

Α': Euaταθiou Βαγγέλης

Β': Τασα6ποuλος Γιώργος

Γραμματέας:

Χριατ6ποuλος Παναγιώτης

Μέλη: Αργuρ6κης Δ.

Δρούταας Π.

Λουρίδας Σ. Ταπειν6ς Ν.

Σuνταντική επιτροπή

Αθανασόπουλος Γεώργιος Ανδρουλακάκης Νίκος Αντωνόπουλος Νίκος Αργυράκης Δημήτριος Βακαλόπουλος Κώστας Βλάχου Αγγελική Γράψας Κων/νος Δρούτσας Παναγιώτης Ευσταθίου Βαγγέλης

Ζώτος Βαγγέλης Καλίκας Σταμάτης Καρακατσάνης Βασίλης Καρκάνης Βασίλης Κατσούλης Γιώργος Κερασαρίδης Γιαννης Κηπουρός Χρήστος Κόντζιας Νίκος Κυριακόπουλος Αντώνης Κυριακόπουλος Θανάσης Κυριακοπούλου­Κυβερνήτου Χρυστ .

Λαζαρίδης Χρήστος Λουρίδας Σωτήρης Μαλαφέκας Θανάσης Μεταξάς Νικόλαος Μυλωνάς Δημήτρης Μώκος Χρήστος

Σαtτη Εύα Σταθόπουλος Γεώργιος Σταϊκος Κώστας Στά'ίκος παναγιώτης Στρατής Γιάννης Ταπεινός Νικόλαος Τασσόπουλος Γιώργος Τριάντος Γεώργιος Τσικαλουδάκης Γιώργος Τσιούμας Θανάσης Τυρλής Ιωάννης Χαραλαμποπούλου Λίνα Χαραλάμπους Θάνος Χριστόπουλος Παναγιώτης

••••••••••••••••••••••••••••••••••••••••••••••••••••••••••••••••••••••••••••••

ΚΕΝΤΡΟ ΓΡΑΦΙΚΩΝ ΤΕΧΝΩΝ: ΔΙΗΝΕΚΕΣ ΣτοιχειοΒεαία - Σελιδοποίηση: Ακαδημίας 43, Αθιivα τηλ.: 270 3606760,

fαχ.: 2 ι ο 3606826 e-mαil: [email protected]

Εκτύπωαη: /Ν'ΓΕΡΠΡΕΣ Α.Ε. τηλ.: 210 8160330 Υnειίθuνος τunογpαφeίοu: Β. Σωτηριάδης

• Τα διαφημιζόμενα βιβλία δε σημαίνει ότι προτείνονται από την Ε.Μ.Ε.

• Οι συνεργάτες, τα άρθρα, οι προτεινόμενες ασκήσεις, οι λύσεις ασκήσεων κτλ. πρέπει

να στέλνονται έγκαιρα, στα γραφεία της Ε.Μ.Ε. με την ένδειξη "Για τον Ευκλείδη β"'.

Τα χειρόγραφα δεν επιστρέφονται.

Τιμή Τεύχους ευρώ 3,50

Ετήσια συνδρομή (10,00 + 4,00 Ταχυδρομικά = ευρώ 14,00)

Ετήσια συνδρομή για Σχολεία ευρώ 10,00

Το αντίτιμο για τα τεύχη που παραγγέλνονται στε"λνεται με απλή επιταγή σε διαταγή

Ε.Μ.Ε. Ταχ. Γραφείο Αθήνα 54 Τ. Θ. 30044 ή πληρώνεται σtα γραφεία της Ε.Μ.Ε.

Page 4: Ευκλειδης Β 58

ΕΚΔΗΛΩΣΗ ΤΗΣ Ε.Μ.Ε.

Ο Πρόεδρος του Δ.Σ. της Ε. ΜΕ. καθηγητής

Θεόδωρος Εξαρχάκος (αριστερά) βραβεύει τον κ. Γ. Ωραιόπουλο.

Τη Δευτέρα 24 Ο1crωβρίου 2005 και ώρα 7.00 μ.μ. η Ε.Μ.Ε. πραγματοποίησε με επιτυχία εκδήλωση στο αμφιθέατρο «lωαν. Δρακόπουλος» στο κε­ντρικό κτίριο του Πανεπιστημίου Αθηνών, με θέ­ματα: Α. Βράβευση του καθηγητή κ. Γ. Ωραιόπουλου

για την πολύχρονη προσφορά του στην Ελληνι­κή Μαθηματική Εταιρεία και στη Μαθηματική Εκπαίδευση .

Β . Ύλη, σκοποί και ιδιαίτεροι στόχοι των περιο­δικών Ευκλείδη Α ' και Ευκλείδη Β ' .

Από την εκδήλωση της Ε. ΜΕ.

Η εκδήλωση πραγματοποιήθηκε σύμφωνα με το παρακάτω πρόγραμμα: • Χαιρετισμός

Θ. Εξαρχάκος, Πρόεδρος του Δ.Σ. της ΕΜΕ. • Ομιλίες

Α. Το έργο και η δραστηριότητα του Συνα­δέλφου Γ. Ωραιόπουλου

Β .

Γ. Καρκούλιας Μέλος της Ε.Μ.Ε. Βράβευση του Συναδέλφου κ. Γ. Ωραιόπου­λου για την πολύχρονη προσφορά του στην Ελληνική Μαθηματική Εταιρεία και στην Ελληνική Παιδεία.

../Σκοποί και Στόχοι του περιοδικού "Ευ­

κλείδης Α"' Μ. Χρυσοβέργης, Ειδικός Γραμματέας του Δ.Σ. της Ε.Μ.Ε. I Πρόεδρος της Συντακτι­κής επιτροπής του Ευκλείδη Α ' .

../ Σκοποί και Στόχοι του περιοδικού "Ευ­

κλείδης Β"' Αντώνης Κυριακόπουλος, Μέλος του Δ.Σ. της Ε.Μ.Ε. I Πρόεδρος της Συντακτικής επι­τροπής του Ευκλείδη Β ' .

../ Η Μαθηματική παιδεία στο Γυμνάσιο και

η συμβολή του Ευκλείδης Α ' Τ. Πατρώνης, Επίκουρος Καθήγητής του Μαθηματικού Τμήματος του Πανεπιστημίου Πατρών.

../ Συζήτηση -Προτάσεις ΕΥΚΛΕΙΔΗΣ Β' λθ ' τ.2/2

Page 5: Ευκλειδης Β 58

Εκδήλωση της Ε.Μ.Ε.

Ο πρόεδρος της συντακτικής επιτροπής του πε­ριοδικού «Ευκλείδης Β '» κ. Αντώνης Κυριακό­πουλος, στην αρχή της ομιλίας του υποστήριξε, με προσωπικά βιώματα, την άποψη ότι και τα τρία περιοδικά της Ε.Μ.Ε. που απευθύνονται στους μα­θητές: «Μικρός Ευκλείδης», «Ευκλείδης Α '» και «Ευκλείδης Β '» μπορούν να παίξουν καθοριστικό ρόλο στη στάση που θα τηρήσουν οι μαθητές ένα­ντι των Μαθηματικών. Ιδιαίτερα οι μαθητές της επαρχίας. Γι' αυτό, τόνισε, προσπαθούμε τα περιο­δικά αυτά να είναι ελκυστικά, ευχάριστα και χρή­σιμα στους μαθητές.

Ο κ. Αντώνης Κυριακόπουλος συνέχισε την ο­μιλία ου ως εξής:

Το περιοδικό «Ευκλείδης Β '» απευθύνεται κυ­ρίως στους μαθητές του Λυκείου. Λέω κυρίως στους μαθητές του Λυκείου, γιατί το περιοδικό προσπαθεί, με διάφορα άρθρα του, να είναι χρήσι­μο και να βοηθάει και τους συναδέλφους μαθημα­τικούς στο δύσκολο έργο τους. Επίσης, όπως θα δούμε παρακάτω, περιέχει και στήλες που τους αφορούν.

Ο Πρόεδρος της συντακτικής επιτροπής του πε­ριοδικού Έυκλείδης Β' κ. Αντώνης Κυριακόπουλος.

Στο Λύκειο οι μαθητές, στην αρχή συστηματο­ποιούν και διευρύνουν σε θεωρητικό επίπεδο τις γνώσεις που απέκτησαν στο Γυμνάσιο. Έχουν ω­ριμάσει πλέον και διδάσκονται την Άλγεβρα και τη Γεωμετρία κανονικά, δηλαδή σαν Μαθηματικές θεωρίες: Αρχικοί όροι, οριζόμενοι όροι, αξιώματα, θεωρήματα. Έτσι συνειδητοποιούν την αυστηρή έννοια της απόδειξης και βλέπουν, ακόμα περισ-

σότερο, την τάξη, τη σαφήνεια και την ακρίβεια, που επικρατούν στα Μαθηματικά. Επειδή, λόγω ηλικίας, είναι «απόλυτοη>, υποσυνείδητα τους α­ρέσει και ευχαριστιούνται όταν βλέπουν στα Μα­θηματικά την αυστηρότητα των συλλογισμών και την βεβαιότητα των συμπερασμάτων. Αυτά, πολ­λούς μαθητές τους εντυπωσιάζουν, τους γοητεύουν θα έλεγα, και τους κάνουν να αγαπήσουν τα Μα­θηματικά και να ασχοληθούν περισσότερο με αυ­τά. Όλοι μας έχουμε δει τη χαρά που έχει ένας μα­θητής όταν κάνει μια απόδειξη μόνος του. Κατά τη γνώμη μου αυτό συμβαίνει γιατί, αντίθετα από ότι γίνεται στα άλλα μαθήματα που διδάσκεται, στα Μαθηματικά, όταν κάνει μία απόδειξη μόνος του, δεν αναπαράγει αποθηκευμένη γνώση, όπως συμ­βαίνει π.χ. στην ιστορία, αλλά δημιουργεί κάτι μό­νος του. Δημιουργεί μία σειρά συλλογισμών, με σαφήνεια και με λογική αυστηρότητα, που τον ο­δηγούν σ' αυτό που θέλει να αποδείξει. Και αυτό, για ένα μαθητή, και όχι μόνο, είναι πράγματι σπουδαίο, γιατί σε μία απόδειξη πρέπει να ξέρει: Τι θα πει, πότε θα το πει και γιατί θα το πει.

Το περιοδικό «Ευκλείδης Β'» προσπαθεί να βοηθήσει τους μαθητές του Λυκείου στα Μαθημα­τικά. Προς τούτο, επιδιώκει να κάνει τα Μαθημα­τικά του Λυκείου όσο γίνεται πιο απλά και κατα­νοητά. Αναλύει και αποσαφηνίζει τα δύσκολα ση­μεία της θεωρίας και επιδιώκει την σε βάθος κατα­νόηση των διαφόρων Μαθηματικών εννοιών. Ε­μπεδώνει τη θεωρία με πολλές κατάλληλες και, κατά το δυνατόν πρωτότυπες, ασκήσεις. Επίσης, επιδιώκει να αναπτύξει την κριτική και συνθετική ικανότητα των μαθητών, καθώς και την ικανότητά τους να επεξεργάζονται άγνωστα θέματα. Αυτό τους βοηθάει πολύ στις εξετάσεις που δίνουν.

Επίσης, το περιοδικό μας, προσπαθεί, με διά­φορα άρθρα γενικού ενδιαφέροντος, να αναδείξει τις αρετές και τη χρησιμότητα των Μαθηματικών. Προσπαθεί να καταλάβουν οι μαθητές ότι τα Μα­θηματικά είναι χρήσιμα παντού και ότι χωρίς αυτά δεν θα είχαμε την τεχνολογική εξέλιξη που βλέ­πουν καθημερινά γύρω τους. Επίσης, με τα άρθρα

ΕΥΚΛΕΙΔΗΣ Β' λθ ' τ.2/3

Page 6: Ευκλειδης Β 58

Εκδήλωση της Ε.Μ.Ε.

αυτά, επιζητεί να τραβήξει το ενδιαφέρον και την περιέργεια των μαθητών, για να ανακαλύψουν τον πλούτο της μαθηματικής γνώσης. Τελικός μας στόχος είναι η θετικότερη στάση και αγάπη των μαθητών για τα Μαθηματικά.

Παράλληλα, όπως σας είπα και παραπάνω, φρο­ντίζουμε το περιοδικό μας να είναι χρήσιμο και στους συναδέλφους μαθηματικούς, οι οποίοι διδά­σκουν τα Μαθηματικά του Λυκείου, καθώς και σε όσους ενδιαφέρονται για τα Μαθηματικά αυτά.

Η συντακτική επιτροπή του περιοδικού «Ευ­

κλείδης Β·», στην οποία έχω την τιμή να είμαι Πρόεδρος, αποτελείται από 35 συναδέλφους μα­θηματικούς. Συνεδριάζουμε κάθε δεύτερη Τρίτη στα γραφεία της Ε.Μ.Ε. από τις 9:30 το βράδυ μέ­χρι να κουραστούμε. Οι συνεδριάσεις είναι ανοι­κτές και μπορεί να συμμετάσχει όποιος συνάδελ­φος θέλει να βοηθήσει στην έκδοση του περιοδι­κού. Η συντακτική επιτροπή, επεξεργάζεται και αξιολογεί τις εργασίες που μας στέλνουν οι συνά­δελφοι μαθηματικοί και οι μαθητές. Στη συνέχεια, κάνει τις διορθώσεις των κειμένων και τα παρακο­λουθεί μέχρι την εκτύπωση του περιοδικού.

Εμείς, της συντακτικής επιτροπής, γράφουμε άρθρα μόνο για να συμπληρώσουμε την ύλη του περιοδικού, όταν δεν έχουμε εργασίες από άλλους συναδέλφους. Αξίζει να σας πω ότι πολλές φορές, για να συμπληρώσουμε την ύλη, αναθέτουμε σ' ένα συνάδελφο της επιτροπής να γράψει ένα άρ­θρο, αφιερώνοντας πολλές ώρες. Και την τελευ­ταία στιγμή, το ακυρώνουμε γιατί φτάνει τέτοιο άρθρο από άλλο συνάδελφο.

Το περιοδικό «Ευκλείδης Β '» περιέχει τις εξής στήλες, με τη σειρά που δημοσιεύονται:

I. ΙΣΤΟΡΙ ΚΕΣ ΜΑΘΗΜΑτΙ ΚΕΣ

ΑΝΑΦΟΡΕΣ

Την επιμέλεια της στήλης αυτής την έχει ο συ­νάδελφος Χρήστος Κηπουρός.

Σ' αυτή τη στήλη δημοσιεύονται άρθρα που αναφέρονται στην ιστορία και την εξέλιξη των Μαθηματικών. Έχει ενδιαφέρον, από ιστορικής απόψεως, να μάθουν οι μαθητές, πώς ξεκίνησαν

και πώς εξελίχτηκαν οι διάφορες Μαθηματικές έν­νοιες, καθώς και τα διάφορα Μαθηματικά σύμβο­λα. Για παράδειγμα, στο πρώτο τεύχος της φετινής χρονιάς, η στήλη αναφέρεται στην ιστορία και την καθιέρωση του δεκαδικού συστήματος αρίθμησης. Έχει μεγάλο ενδιαφέρον και σας συνιστώ να τη διαβάσετε.

Στη στήλη αυτή δεν αποκλείονται αναφορές στη ζωή και το έργο μεγάλων Μαθηματικών, αρ­χαίων και νεωτέρων. Θα έλεγα ότι είναι,υποχρέω­σή μας να το κάνουμε αυτό, για να τιμήσουμε ε­κείνους, οι οποίοι εργάστηκαν σκληρά για να φθά­σουν τα Μαθηματικά στη σημερινή τους μορφή .

Σκοπός της στήλης αυτής είναι να διεγείρει το ενδιαφέρον και την αγάπη των μαθητών για τα Μαθηματικά και να τους πληροφορήσει για την ιστορική πορεία της Μαθηματικής σκέψης.

2. ΣΤΟΙΧΕΙΑ ΜΑΘΗΜΑτΙΚΗΣ ΛΟΓΙΚΗΣ

Τη στήλη αυτή τη γράφει ο υποφαινόμενος. Στη στήλη αυτή θα δημοσιεύσουμε, σε συνέ­

χειες, όσα από τη Μαθηματική Λογική είναι τε­λείως απαραίτητα για τη σωστή αντιμετώπιση και κατανόηση των Μαθηματικών. Στη συνέχεια θα αναφερθούμε στους τρόπους απόδειξης μιας πρό­τασης στα Μαθηματικά, καθώς και στους τρόπους εύρεσης ενός Μαθηματικού αντικειμένου (αριθμό, διάνυσμα, συνάρτηση κτλ.), που είναι και ο αντι­κειμενικός σκοπός της στήλης αυτής. Εν των με­ταξύ, οι μαθητές θα βοηθηθούν και στην κατανόη­ση των «λεπτών» μαθηματικών εννοιών. Επίσης θα ασκηθούν στο να χρησιμοποιούν τα Μαθηματι­κά όχι μόνο ως γνώση, αλλά και ως μέθοδο σκέ­ψης και πράξης στην καθημερινή ζωή . Επίσης, θα βοηθήσει τους μαθητές να διατυπώνουν με ακρί­βεια και σαφήνεια τις απόψεις τους και να είναι σε θέση να τις υποστηρίζουν με επιχειρήματα.

Η Μαθηματική Λογική, εκτός του ότι στηρίζει και αναλύει τη μαθηματική σκέψη, συνδέεται στενά και με την πληροφορική. Είναι χρήσιμη και συμβάλ­λει στην τεχνητή νοημοσύνη, τη λογική σχεδίαση κυκλωμάτων, τον έλεγχο προγραμμάτων κτλ.

ΕΥΚΛΕΙΔΗΣ Β' λθ ' τ.2/4

Page 7: Ευκλειδης Β 58

Εκδήλωση της Ε.Μ.Ε.

Πιστεύω ότι το ενδιαφέρον της στήλης αυτής είναι γενικότερο και ότι θα είναι χρήσιμη όχι μόνο στους μαθητές, αλλά και σε όλους όσοι ασχολού­νται με τα Μαθηματικά.

3. ΗΟΜΟ MATHEMAτiCUS

Η στήλη αυτή, σύμφωνα με το συνάδελφο Γιάννη Κερασαρίδη που έχει την επιμέλεια, έχει σκοπό την ανταλλαγή απόψεων και την ανάπτυξη προβληματισμού πάνω στα εξής θέματα:

Ι) τι είναι μαθηματικά. 2) Πρέπει ή όχι να διδάσκονται. 3) Ποιοι είναι οι κλάδοι των Μαθηματικών και

ποιο το αντικείμενο του καθενός. 4) Ποιες είναι οι εφαρμογές τους. 5) Ποιες επιστήμες ή κλάδοι επιστημών απαι­

τούν καλή γνώση των Μαθηματικών για να μπορέσει κάποιος να τους σπουδάσει.

4. ΜΑΘΗΜΑτΙ ΚΟΙ ΔΙΑΓΩΝΙΣΜΟ Ι

ΚΑΙ ΜΑΘΗΜΑ τΙ ΚΕΣ ΟΛ ΥΜΠΙΑΔΕΣ

Η Ε.Μ.Ε. κάθε χρόνο συγκροτεί δύο εξαμελείς Εθνικές ομάδες Μαθηματικών. Η πρώτη ονομάζε­ται «Εθνική ομάδα των μικρών» και αποτελείται από μαθητές ηλικίας μέχρι δεκαπεντέμιση (15,5)

ετών και η δεύτερη ονομάζεται «Εθνική ομάδα

των μεγάλων» και αποτελείται από μαθητές του Λυκείου.

Για να επιλεγούν οι μαθητές των ομάδων αυτών η Ε.Μ.Ε. κάθε χρόνο διοργανώνει τρεις Πανελλή­νιους Μαθηματικούς διαγωνισμούς και για τους μικρούς και για τους μεγάλους, που ονομάζονται κατά σεφά διεξαγωγής

«ΘΑΛΗΣ», «ΕΥΚΛΕΙΔΗΣ»

και «ΑΡΧΙΜΗΔΗΣ»

Όσοι διακρίνονται στον Θαλή παίρνουν μέρος στον Ευκλείδη και όσοι διακρίνονται στον Ευκλεί­δη παίρνουν μέρος στον Αρχιμήδη . Από τους δια­κριθέντες στον τελευταίο αυτό διαγωνισμό, οι έξι πρώτοι από τους μικρούς και οι έξι πρψrοι από τους μεγάλους, είναι αυτοί που αποτελούν τις δύο Εθνικές μας ομάδες Μαθηματικών.

Οι ομάδες αυτές συμμετέχουν στην «Βαλκανι­κή Μαθηματική Ολυμπιάδα». Η Εθνική μας ο­μάδα των μεγάλων συμμετέχει ακόμα στη «Μεσο­γειακή Μαθηματική Ολυμπιάδα», καθώς και στη «Διεθνή Μαθηματική Ολυμπιάδα», η οποία το 2004 έγινε στην Αθήνα και τη διοργάνωσε η Ε.Μ.Ε. με απόλυτη επιτυχία.

Επίσης, εφέτος,η «Βαλκανική Μαθηματική Ολυμπιάδα» των μικρών έγινε στη Βέροια και τη διοργάνωσε με μεγάλη επιτυχία η Ε.Μ.Ε. με συ­νεργασία με το παράρτημα Ημαθίας.

Αξίζει να σας αναφέρω ότι, με ενέργειες της Ε.Μ.Ε. το 2003 ψηφίστηκε από τη Βουλή, νόμος σύμφωνα με τον οποίο, όσοι μαθητές διακρίνονται στις Ολυμπιάδες και τις Βαλκανιάδες, παίρνοντας χρυσό ή αργυρό ή χάλκινο μετάλλιο, αποκτούν το δικαίωμα να εγγραφούν, χωρίς εξετάσεις, σε όποια Πανεπιστημιακά τμήματα θέλουν, στα οποία όμως, για την εισαγωγή τους σε αυτά, το μάθημα των μαθηματικών εξετάζεται ως μάθημα κατεύθυνσης.

Τα θέματα των διαγωνισμών αυτών είναι υψη­λού επιπέδου σκέψης και απαιτούν όχι μόνο ειδική ικανότητα και εξειδικευμένη μαθηματική νοοτρο­πία, αλλά και κάποιες επιπλέον γνώσεις Μαθημα­τικών. Είναι θέματα για μαθηματικά ταλέντα.

Σκοπός της στήλης αυτής είναι να φέρει σε ε­παφή τους μαθητές που ενδιαφέρονται για τους παραπάνω διαγωνισμούς με το πνεύμα και τη νοο­τροπία των θεμάτων των διαγωνισμών αυτών.

Στη στήλη αυτή δημοσιεύονται τα θέματα και οι λύσεις των παραπάνω διαγωνισμών και ταυτό­χρονα αναπτύσσονται τρόποι και μεθοδολογίες για την αντιμετώπιση τέτοιων θεμάτων.

Την επιμέλεια της στήλης αυτής την έχει ο συ­νάδελφος Σωτήρης Λουρίδας. 5. ΜΑΘΗΜΑτΙ ΚΆ ΓΙΑ τΙΣ ΤΑΞΕΙΣ Α',

Β' ΚΑΙ Γ Λ ΥΚΕΟΥ

Στις στήλες αυτές δημοσιεύονται Μαθηματικά των τριών τάξεων του Λυκείου. Εδώ υλοποιούμε την βοήθεια προς τους μαθητές του Λυκείου στα Μαθηματικά, που σας ανέφερα παραπάνω. Εδώ, εκτός του ότι επισημαίνονται τα βασικά σημεία

ΕΥΚΛΕΙΔΗΣ Β ' λθ ' τ.2/5

Page 8: Ευκλειδης Β 58

Εκδήλωση της Ε.Μ.Ε.

της θεωρίας και διευκρινίζονται οι δυσνόητες έν­νοιες αυτής, φροντίζουμε ώστε η εμπέδωση να γί­νεται με ασκήσεις που είναι κατανοητές και οι ο­ποίες κεντρίζουν το ενδιαφέρον των μαθητών. Ε­πιδιώκουμε ακόμα η διάταξη των ασκήσεων να εί­ναι τέτοια, ώστε να μην aπογοητεύουν τον αρχά­ριο, να ενθαρρύνουν τον μέτριο και να ενθουσιά­ζουν τον προχωρημένο μαθητή .

Στο τελευταίο τέταρτο τεύχος κάθε χρονιάς, δημοσιεύουμε γενικές ασκήσεις από όλα τα κεφά­λαια κάθε τάξης. Φροντίζουμε ώστε οι ασκήσεις αυτές της Γ τάξης να είναι επιπέδου Πανελληνίων Εξετάσεων, για να βοηθήσουμε τους μαθητές στις εξετάσεις που θα δώσουν.

Τις στήλες αυτές τις επιμελούνται πολλοί συ­νάδελφοι από τη συντακτική επιτροπή . Για κάθε τάξη και για κάθε μάθημα έχουν οριστεί υπεύθυ­νοι, οι οποίοι επεξεργάζονται και αξιολογούν τις εργασίες των συναδέλφων που φθάνουν στην εται­ρεία μας για τις στήλες αυτές. Οι συνάδελφοι αυ­τοί είναι οι εξής:

Α' Τάξη

• Άλγεβρα: Ταπεινός Νίκος και Μώκος Χρή­στος.

• Γεωμετρία: Ευσταθίου Βαγγέλης και Τριάντος Γεώργιος.

Β' Τάξη

• Άλγεβρα: Κόντζιας Νίκος και Μυλωνάς Δημή­τριος.

• Γεωμετρία: Καρκάνης Βασίλης και Κατσού­λης Γεώργιος.

• Μαθηματικά Κατεύθυνσης: Δρούτσας Πανα­γιώτης, Λαζαρίδης Χρίστος και Τριάντος Γε­ώργιος.

Γ Τάξη

• Μαθηματικά Κατεύθυνσης: Τασσόπουλος Γεώργιος, Αργυράκης Δημήτριος και Χαραλά­μπους Θάνος.

• Μαθηματικά Γενικής Παιδείας: Βακαλόπου­λος Κώστας, Χαραλαμποπούλου Λίνα και Τσι­καλουδάκης Γεώργιος.

6. Ο ΕΥΚΛΕΙ ΔΗΣ ΠΡΟΤΕΙΝΕΙ . . . ΕΥΚΛΕΙ­

ΔΗ ΚΑΙ . . . Δ ΙΟΦΑΝΤΟ

Η στήλη αυτή απευθύνεται σε συναδέλφους και μαθητές με ζωηρότερο μαθηματικό ενδιαφέρον και γενικά σε κάθε «εραστψ> των Μαθηματικών, ο οποίος θέλει να κατακτήσει την καρδιά των Μα­θηματικών, που, σύμφωνα με τον Πωλ Χάλμος (PAUL HALMOS), είναι τα προβλήf.Ι.ατα και οι λύσεις.

Στη στήλη αυτή, δημοσιεύονται ασκήσεις που προτείνονται από συναδέλφους, και όχι μόνο, και εΠιλέγονται από τους υπεύθυνους της στήλης, ώ­στε: • Να είναι όσο το δυνατόν πρωτότυπες. • Να προκαλούν το ενδιαφέρον και να ενεργο­

ποιούν τη δημιουργική σκέψη . • Να είναι κατανοητές από τους μαθητές. • Να μην έχουν μακροσκελείς λύσεις και κατά

προτίμηση να περιέχουν κάποιες από αυτές τις υπέροχες μαθηματικές κινήσεις που ξαφνιάζουν και γοητεύουν. Στη στήλη αυτή λαμβάνουν μέρος, με ασκήσεις

και λύσεις που μας στέλνουν, και φίλοι των Μα­θηματικών από άλλους Επιστημονικούς χώρους: Μηχανικοί, Χημικοί, Γεωπόνοι κτλ.

Η στήλη παραμένει ανοικτή σε καθέναν που θα ήθελε να προτείνει κάτι πρωτότυπο, κάτι ξεχωρι­στό, κάτι που θα διεγείρει το μαθηματικό ενδιαφέ­ρον.

Την επιμέλεια της στήλης αυτής την έχει ο υ­ποφαινόμενος και οι συνάδελφοι: Γιάννης Στρα­τής, Γεώργιος Τριάντος και Νίκος Αντωνόπουλος.

7. ΤΟ ΒΗΜΑ ΤΟΥ ΕΥΚΛΕΙΔΗ

Πιστεύουμε ότι πολλοί «μαχόμενοι» συνάδελ­φοι, εννοώ τους συναδέλφους που έρχονται σε κα­θημερινή επαφή με τον πίνακα, όταν στέκονται μπροστά στον πίνακα και διδάσκουν, όλο και κάτι καινούργιο σκέπτονται, όλο και καινούργιες ιδέες τους έρχονται, όλο και κάτι παρατηρούν, όχι μόνο ως προς τη σειρά και την παρουσίαση, αλλά και ως προς την ουσία των Μαθηματικών που διδάσκουν.

ΕΥΚΛΕΙΔΗΣ Β' λθ ' τ.2/6

Page 9: Ευκλειδης Β 58

Εκδήλωση της Ε.Μ.Ε.

Πολλές φορές, οι ίδιοι οι μαθητές κάνουν ερωτή­σεις και παρατηρήσεις, οι οποίες μας ξαφνιάζουν και μας οδηγούν σε πολύ χρήσιμα συμπεράσματα.

Οι εμπειρίες αυτές των συναδέλφων, πολλές φορές, τους οδηγούν να γράψουν εργασίες, οι ο­ποίες έχουν να μας πουν κάτι νέο και πρωτότυπο, σχετικά με τα Μαθηματικά του Λυκείου.

Η στήλη αυτή, φιλοξενεί τέτοιες εργασίες συ­ναδέλφων (αν έχουμε λάβει φυσικά).

Την επιμέλεια της στήλης αυτής την έχουν οι συνάδελφοι: Γιάννης Στρατής και Βαγγέλης Ευ­σταθίου.

8. ΣΤΗΛ Η ΤΟΥ ΜΑΘΗΤΗ

Η στήλη αυτή ανήκει, εξολοκλήρου στους μα­θητές. Στη στήλη αυτή δημοσιεύονται ασκήσεις και λύσεις που μας στέλνουν οι μαθητές του Λυ­κείου. Επίσης, απαντάμε σε ερωτήσεις και απορίες των μαθητών στα Μαθηματικά.

Την επιμέλεια της στήλης αυτής την έχει ο υ­ποφαινόμενος και ο συνάδελφος Γιάννης Στρατής.

9. ΤΑ ΜΑΘ ΗΜΑτΙΚΆ

ΜΑΣ Δ ΙΑΣΚΕΔΑΖΟΥΝ

Την επιμέλεια της στήλης αυτής την έχει ο συ­νάδελφος Παναγιώτης Χριστόπουλος.

Σ' αυτή τη στήλη δημοσιεύονται προβλήματα, τα οποία δεν απαιτούν ιδιαίτερες μαθηματικές γνώσεις, αλλά μας διασκεδάζουν με την εκφώνησή τους ή τη λύση τους και είναι μία ευχάριστη και συναρπαστική ασχολία.

Τα μαθηματικά, αν και είναι επιστήμη που α­παιτεί αυστηρή διατύπωση έχουν τη μαγεία να προκαλούγ το ενδιαφέρον όλων των ανθρώπων.

Επινοήσεις σε προβλήματα, με κατάλληλο τρό­πο διατυπωμένα, εξάπτουν το πνεύμα, διεγείρουν τη φαντασία και κεντρίζουν την περιέργεια. Πρώ­τοι οι Αρχαίοι Έλληνες, όπως ο Διόφαντος, ο Ζή­νωνας κ.ά. , διατύπωσαν τέτοια προβλήματα.

Σκοπός της στήλης αυτής είναι να φέρει τους μαθητές πιο κοντά στα μαθηματικά, μέσα από πε­ρίεργα και διασκεδαστικά προβλήματα.

Με τα προβλήματα αυτά, οι μαθητές μαθαίνουν νέους τρόπους σκέψης και σταδιακά αποκτούν την ικανότητα να αντιμετωπίζουν τα προβλήματα με μεγαλύτερη ευκολία.

Κυρίες και κύριοι. Εμείς της συντακτικής επιτροπής, διαθέτουμε

aνιδιοτελώς πολλές ώρες από τον ελεύθερο χρόνο μας για να βγει το περιοδικό « Ευκλείδης Β'». Γιατί το κάνουμε αυτό; Γιατί είμαστε ερωτευμένοι με τα Μαθηματικά. Και όπου υπάρχει έρωτας μην ψάχνετε για εξηγήσεις. Αγαπάμε λοιπόν τα Μαθη­ματικά και θέλουμε να βοηθήσουμε ώστε να τα κατανοήσουν και να τα αγαπήσουν και οι μαθητές. Γιατί, τότε θα μπορούν να βάλουν υψηλούς στό­χους στη ζωή τους και να τους πετύχουν. Επίσης, γιατί τα Μαθηματικά συμβάλλουν στην ολοκλή­ρωση της προσωπικότητας του μαθητή, καθόσον καλλιεργούν την πειθαρχημένη σκέψη και συμπε­ριφορά.

Δεν ισχυρίζομαι ότι τα κάνουμε όλα τέλεια. Ό­χι. Αλλά τουλάχιστον προσπαθούμε. Προσπαθού­με να κάνουμε το περιοδικό, όλο και καλύτερο, όλο και πιο χρήσιμο στους μαθητές και στους συ­ναδέλφους. Πιστεύω όμως ότι η βελτίωση και η ποιοτική αναβάθμιση του περιοδικού μας δεν μπο­ρεί να γίνει χωρίς τη συμμετοχή των συναδέλφων Μαθηματικών, καθώς και των μαθητών. Για το λόγο αυτό, ως πρόεδρος της συντακτικής επιτρο­πής του περιοδικού «Ευ κλείδης Β'», καλώ τους συναδέλφους μαθηματικούς, καθώς και τους μα­θητές του Λυκείου, να μας στείλουν, εκτός από τις εργασίες τους, τη δημιουργική κριτική τους και τις προτάσεις τους, για ό,τι αυτοί νομίζουν ότι είναι σημαντικό που θα βοηθούσε στη βελτίωση του πε­ριοδικού.

Το περιοδικό είναι όλων των Ελλήνων Μαθη­ματικών. Είναι όλων μας.

Σας ευχαριστώ που με ακούσατε.

ΕΥΚΛΕΙΔΗΣ Β' λθ ' τ.2/7

Page 10: Ευκλειδης Β 58

"Τα Μαθηματικά θεμελιώνονται, κατανοούνται και αναπτύσσονται με τη βοήθεια της Μαθηματικής Λογικής"

του Αντώνη Κυριακόπουλου

(Συνέχεια από το προηγούμενο τεύχος)

1 .10 ΛΟΓΙΚΕΣ ΠΑΡΑΣΤΑΣΕΙΣ

• Κάθε έκφραση, που αποτελείται από μετα­

βλητές: p, q r, . . . κα ι λογικούς συνδέσμους

και η οποία γίνεται μία πρόταση , όταν οι με­

ταβλητές αντικατασταθούν με οποιεσδήποτε

προτάσεις, λέγεται λογική παράσταση ή α­

πλώς παράσταση.

Συμφωνούμε ότι στις παραστάσεις περιλαμβάνονται και οι μεταβλητές: p, q, r, . . .

Π.χ. οι παρακάτω εκφράσεις είναι παραστά­σεις:

p Λ q, p, p � q, (p ν q) <=> ( q � η, [(p<::::>q)Λ f]�(pΛr).

Οι παρενθέσεις και οι αγκύλες χρησιμοποιού­νται για να καθορίσουν με σαφήνεια τη σειρά των πράξεων (όπως και στην Άλγεβρα).

1 .1 1 ΠΙΝΑΚΕΣ τΙΜΩΝ ΑΛΗΘΕΙΑΣ

Έστω μία παράσταση Ρ με μεταβλητές: p, q, r,

. . . Μπορούμε να κατασκευάσουμε ένα πίνακα, στον οποίο να εμφανίζεται η τιμή αληθείας, που λαμβάνει η παράσταση Ρ για όλους τους δυνατούς συνδυασμούς των τιμών αληθείας των μεταβλητών της p, q, r, . . . Ένας τέτοιος πίνακας λέγεται πίνα­

κας τιμών αληθείας της παράστασης Ρ . Π.χ. ο πίνακας τιμών αληθείας της παράστασης: :

(p ν q) Λ (p � q)

είναι ο παρακάτω :

Ρ q cl pvq p:::>q (p V q) Λ (p ::::> q) α α Υ α α Υ α α Υ α Υ Υ Υ Υ α α

(l) (2) (3) (4)

α α Υ Υ α Υ α α

(5) (6)

Οι μεταβλητές της παράστασης αυτής είναι δύ­ο, οι p και q. Στον παραπάνω πίνακα, κάτω από τις μεταβλητές p και q έχουν τεθεί όλοι οι δυνατοί συνδυασμοί των τιμών αληθείας των, που είναι τέσσερις [στήλες (1) και (2)]. Η στήλη (3) προκύ­πτει από τη (2) και περιέχει τις τιμές αληθείας, τις οποίες λαμβάνει η παράσταση q . Η στήλη ( 4) προκύπτει από τις στήλες (1) και (3) και περιέχει τις τιμές αληθείας, τις οποίες λαμβάνει η παρά­σταση p ν q . Η στήλη (5) προκύπτει από τις στή-λες (1) και (2) και περιέχει τις τιμές αληθείας, τις οποίες λαμβάνει η παράσταση p � q . Τέλος, η στήλη ( 6), που είναι και η ζητούμενη, προκύπτει από τις στήλες (4) και (5) και περιέχει τις τιμές αληθείας, τις οποίες λαμβάνει η δοσμένη παρά­σταση .

Ανάλογα βρίσκουμε ότι ο πίνακας τιμών αλη­θείας της παράστασης:

(p� p)Λp

είναι ο παρακάτω: Ρ Ρ Ρ ::::> q (p ::::> p) Λ Ρ α Υ Υ Υ y α α α

Σημείωση: Αν μία παράσταση περιέχει μία μετα­βλητή, τότε οι δυνατοί συνδυασμοί των τιμών α­ληθείας είναι 2. Αν περιέχει δύο μεταβλητές, οι

ΕΥΚΛΕΙΔΗΣ Β' λθ ' τ.2/8

Page 11: Ευκλειδης Β 58

Ρ

α α Υ Υ

------------- Στοιχεία Μαθηματικής Λογικής

δυνατοί συνδυασμοί των τιμών αληθείας είναι 4 = 22• Αν περιέχει τρεις μεταβλητές, οι δυνατοί συνδυασμοί των τιμών αληθείας είναι 8 = 23 • Και γενικά, αν περιέχει ν μεταβλητές, οι δυνατοί συν­δυασμοί των τιμών αληθείας είναι 2ν.

1.12 ΕΙΔΗ ΠΑΡ ΑΣΤ ΑΣΕΩΝ

• Κάθε παράσταση, η οποία για κάθε συνδυα­

σμό των τιμών αληθείας των μεταβλητών

της λαμβάνει:

α) τιμή αληθείας α, ονομάζεται ταυτολογία,

β) τιμή αληθείας y, ονομάζεται αυτοαντίφαση.

Κάθε παράσταση, που δεν είναι ταυτολο­

γία, ούτε αυτοαντίφαση, ονομάζεται σχετική.

Υπάρχουν λοιπόν τρία είδη παραστάσεων: Οι ταυτολογίες, οι αυτοαντιφάσεις

και οι σχετικές

Παραδείγματα: I) Η παράσταση :

είναι μια ταυτολογία. Πράγματι, στην τελευταία στήλη του παρακάτω πίνακα τιμών αληθείας αυ­τής, εμφανίζεται μόνο το α:

- p�q -- - --

q q p=>q pΛq p�q�pΛq α Υ α Υ Υ α Υ α Υ α α α α Υ α Υ Υ α Υ α α Υ Υ α

2) Η παράσταση : p Λ p είναι μία αυτοαντίφαση.

Πράγματι, στην τελευταία στή­λη, του , παραπλεύρως πίνακα τιμών αληθείας αυτής, εμφανίζε­ται μόνο το y.

3) Η παράσταση : (p Λ q) ν (p :::::> q)

Ρ Ρ pλq α Υ Υ Υ α Υ

είναι σχετική. Πράγματι, στην τελευταία στήλη, του παρακάτω πίνακα τιμών αληθείας αυτής, εμ­φανίζονται και το α και το y:

Ρ q pΛq p�q (p Λ q) ν (p ::::> q) α α α α α α Υ Υ Υ Υ Υ α Υ α α Υ Υ Υ α α

- Είναι φανερό ότι: α) Αν μία παράσταση Ρ είναι ταυτολογία, τότε

η παράσταση Ρ είναι αυτοαντίφαση και α­ντιστρόφως.

β) Αν μία παράσταση Ρ είναι σχετική, τότε η παράσταση Ρ είναι σχετική και αντιστρό­φως.

1 . 13 ΝΟΜΟΙ ΤΗΣ ΛΟΓΙΚΗΣ ΤΩΝ ΠΡΟΤΑΣΕΩΝ

Οι ταυτολογίες παίζουν πρωτεύοντα ρόλο στη Μαθηματική Λογική (και γενικότερα στα Μαθη­ματικά). Αυτές ονομάζονται και αρχές ή νόμοι

της λογικής των προτάσεων ή απλά νόμοι της

Λογικής. Παρακάτω παραθέτουμε τους σπουδαιό­τερους από τους νόμους αυτούς (το ότι οι παρακά­τω παραστάσεις είναι νόμοι της Λογικής, δηλαδή ταυτολογίες, διαπιστώνεται εύκολα κατασκευάζο­ντας τον πίνακα τιμών αληθείας τους) :

1. p � p, p => p (ταυτότητας) 2. pνp

3. pΛp

• Α ντιμεταθετικοί νόμοι:

4. pνq �qνp

5. pΛq�qΛp

6. (p � q)�(q � p)

• Προσεταιριστικοί νόμοι:

( αποκλείσεως τρίτου) ( αντιφάσεως)

7. (pνq)νr � pν(q ν r)

8. (pΛq)Λr�pΛ(qΛr)

• Επιμεριστικοί νόμοι:

9. p ν ( q Λ r) � (p ν q) Λ (p ν r)

10. pΛ(q νr)�(pΛq)ν(pΛr)

ΕΥΚΛΕΙΔΗΣ Β' λθ ' τ.2/9

Page 12: Ευκλειδης Β 58

------------- Στοιχεία Μαθηματικής Λογικής

• Νόμοι αρνήσεως:

11. p�p 12. pΛq � pνq 13. pνq � pΛq 14 . p � q �pΛq 15. p � q � (p Λ q) ν (p Λ q)

• Νόμοι αντικαταστάσεως:

16. (p � q)�(pνq) 17. (p � q) � [ (p � q) Λ ( q � Ρ)]

• Νόμοι διαλογισμού: 18. (p � q)Λp � q (αποσπάσεωςήΜοduspοηeηs)

1 9 . (p � q) Λ q � p (Modus tollens)

20. (p � q)Λ(q�r) � (p � r) (υποθετικού συλλογισμού)

21. (p ν q) Λ p � q (διαζευκτικού συλλογισμού)

22. (p � q) � (q � p) ( αντιθετοαντιστροφής)

23. [ (p � q) Λ (p � q)] � q (δυνατών περιπτώσεων)

24. (p Λ q � r Λ "'f) � (p � q) (της εις άτοπο απαγωγής)

25. (p � ( q � r)] � (p Λ q � r) (εξαγωγής)

. . . . . . . . . . . . . . • . •••••••••••

26. (p � q) � (p � q) 27. (p � q) Λ ( q � r) � (p � r) 28. (p � r) Λ ( q � r) � (p ν r � r) 29. (p � q) Λ (r � t) � (p Λ r � q Λ t)

(p � q)Λ(r � t) � (pνr � q ν t) 30. (p � q)Λ(r� t) � (pΛr� q Λ t)

(p � q)Λ(r � t) � (pν r � q ν t)

1 .14 ΜΟΡΦΩΣΗ ΤΑΥΤΟΛΟΓΙΩΝ

Με τους πίνακες τιμών αληθείας δεν μπορούμε

να σχηματίσουμε ταυτολογίες, παρά μόνο να ελέγ­

ξουμε αν μία δοσμένη παράσταση είναι ταυτολο­

γία ή όχι (και αυτό, όταν το πλήθος των μεταβλη­

τών είναι μεγάλο, είναι εξαιρετικά επίπονο). Αυτό

οδήγησε στην διατύπωση κανόνων μορφώσεως

(σχηματισμού) ταυτολογιών, από τους οποίους

σπουδαιότεροι είναι οι εξής:

1) Κανόνας αρνήσεως.

• Α ν μία παράσταση Ρ είναι αυτοαντίφαση,

τότε η παράσταση Ρ είναι ταυτολογία και

αντιστρόφως.

Π.χ. επειδή η παράσταση: p Λ p είναι αυτοαντί-

φαση (§1.12), σύμφωνα με τον παραπάνω κανόνα,

η παράσταση: p Λ p είναι ταυτολογία.

2) Κανόνας διαζεύξεως.

• Α ν από δύο παραστάσεις Ρ και Q η μία του­

λάχιστον είναι ταυτολογία, τότε η παράστα­

ση Ρ ν Q είναι ταυτολογία.

Π. χ. επειδή η παράσταση: p ν p είναι ταυτολογία

(§1.13, 2), σύμφωνα με τον παραπάνω κανόνα, η παράσταση: (p ν p) ν (p � q) είναι επίσης ταυτο-

λογία.

3) Κανόνας συζεύξεως

• Α ν δύο παραστάσεις Ρ και Q είναι ταυτολο­

γίες, τότε και η παράσταση: Ρ Λ Q είναι

ταυτολογία.

Π. χ. επειδή οι παραστάσεις:

pΛq � pνq και (p � q)� pνq είναι ταυτολογίες (§1.13), σύμφωνα με τον παρα­

πάνω κανόνα, η παράσταση:

(p Λ q � p ν q) Λ [ (p � q) � p ν q] είναι επίσης ταυτολογία.

4) Κανόνας αποσπάσεως ή Modus ponens.

- Κανόνας αποσπάσεως για προτάσεις:

• Θεωρούμε δύο προτάσεις p και q. Αν οι προ­

τάσεις p και p � q είναι αληθείς, τότε και η

πρόταση q είναι αληθής(§ 1.6).

- Κανόνας αποσπάσεως για παραστάσεις:

• Θεωρούμε δύο παραστάσεις Ρ και Q. Α ν οι

παραστάσεις Ρ και Ρ � Q είναι ταυτολογίες,

τότε και η παράσταση Q είναι ταυτολογία.

ΕΥΚΛΕΙΔΗΣ Β' λθ' τ.2/10

Page 13: Ευκλειδης Β 58

------------- Στοιχεία Μαθηματικής Λογικής

5) Κανόνας συνεπαγωγής

• Αν από δύο παραστάσεις Ρ και Q, η Q λαμ­

βάνει τιμή αληθείας α για κάθε συνδυασμό

των τιμών αληθείας των μεταβλητών τους,

για τον οποίο η Ρ λαμβάνει τιμή αληθείας α,

τότε η παράσταση: Ρ=> Q είναι ταυτολογία

και αντιστρόφως.

Π.χ. η παράσταση:

Ρ Λ q => (p => q) ( 1 ) είναι μία ταυτολογία. Πράγματι, έστω ότι για ένα

συνδυασμό των τιμών αληθείας των μεταβλητών της p και q είναι τ(p Λ q) = α . Τότε τ(p) =α και

τ( q) = α και άρα τ(p => q) = α . Συνεπώς, σύμφω-

να με τον παραπάνω κανόνα, η παράσταση ( 1 ) εί­

ναι μία ταυτολογία.

6) Κανόνας ισοδυναμίας

• Α ν μια παράσταση Ρ λαμβάνει την ίδια τιμή

αληθείας με μία παράσταση Q, για κάθε

συνδυασμό των τιμών αληθείας των μετα­

βλητών τους, τότε η παράσταση: Ρ <=> Q

είναι ταυτολογία και αντιστρόφως.

Π.χ. η παράσταση:

Ρ<=> (p => p) (2)

είναι μία ταυτολογία. Πράγματι: α) έστω ότι τ(p) = α . Τότε τ(p) = y και άρα τ(p => p) =α .

β) Έστω ότι τ(p) = y , τότε τ(p) = α και άρα

τ(p => p) = y. Συνεπώς, σύμφωνα με τον παραπά-

νω κανόνα, η παράσταση (2) είναι μία ταυτολογία.

7) Κανόνας αντικατάστασης μεταβλητών.

• Αν σε μία ταυτολογία αντικαταστήσουμε μία

ή περισσότερες μεταβλητές της με οποιεσδή­

ποτε παραστάσεις, τότε η προκύπτουσα πα­

ράσταση είναι πάλι μία ταυτολογία.

Π.χ. αν στην ταυτολογία:

(p => q) Λp => q (§ . 1 . 1 3 , 8)

αντικαταστήσουμε το p με την παράσταση p και το q

με την p <=> q , τότε θα προκύψει η εξής παράσταση:

[ρ => (p <=> q)] Λ Ρ => (p <=> q) '

η οποία, σύμφωνα με τον παραπάνω κανόνα, είναι

μια ταυτολογία.

Σημείωση: Οι αποδείξεις των παραπάνω κανόνων,

καθώς και των κανόνων και προτάσεων που θα ακο­

λουθήσουν στην εργασία αυτή, υπάρχουν στο βιβλίο

του γράφοντος: «ΜΑΘΗΜΑτΙΚΗ ΛΟΓIΚΗ». Οι

αναγνώστες του περιοδικού μας μπορούν να μας

γράψουν για ποιες αποδείξεις ενδιαφέρονται και θα

τους τις στείλουμε φωτοτυπημένες.

1 . 15 ΛΟΓΙΚΉ ΣΥΝΕΠΑΓΩΓΗ ΠΑΡΑΣΤΆΣΕΩΝ

• Μ ία παράσταση Ρ λέμε ότι συνεπάγεται λο­

γικά ή συνεπάγεται ταυτολογικά μία παρά­

σταση Q αν, και μόνο αν, η παράσταση

Ρ=> Q είναι μια ταυτολογία.

Για παράδειγμα, η παράσταση: p ν r συνεπάγεται

λογικά την παράσταση: q => (p => q) , γιατί, όπως

μπορούμε να δείξουμε εύκολα, η παράσταση:

p ν r => [ q => (p => q)] είναι μια ταυτολογία.

1 . 16 ΛΟΓΙΚΉ ΙΣΟΔ ΥΝΑΜΙΑ ΠΑΡΑΣΤΆΣΕΩΝ

• Μ ία παράσταση Ρ λέμε ότι είναι λογικά ισο­

δύναμη ή απλά ισοδύναμη με μία παράσταση

Q, και γράφουμε: Ρ = Q αν, και μόνο αν, η

παράσταση: Ρ <=> Q είναι μια ταυτολογία.

Για παράδειγμα:

p => q=pνq ,

γιατί η παράσταση:

(p => q) <=> Ρ ν q

είναι μία ταυτολογία (§ 1 . 1 3 , 16). - Βρίσκουμε εύκολα ότι ισχύουν οι παρακάτω

προτάσεις:

Πρόταση I. Για οποιεσδήποτε παραστάσεις Ρ, Q και R ισχύουν:

ΕΥΚΛΕΙΔΗΣ Β ' λθ' τ.2/11

Page 14: Ευκλειδης Β 58

------------- Στοιχεία Μαθηματικής Λογικής

α) Ρ Ξ Ρ (αυτοπαθής) β) Α ν Ρ Ξ Q , τότε Q Ξ Ρ

(συμμετρική) γ) Αν Ρ Ξ Q και Q Ξ R, τότε Ρ Ξ R

(μεταβατική) Πρόταση Ι Ι. Δύο παραστάσεις είναι ισοδύναμες αν,

και μόνο αν, καθεμία από αυτές συνε­πάγεται λογικά την άλλη (§ 1.15).

1.17 ΚΑΝΟΝΕΣ ΑΝτΙΚΑΤΑΣΤΑΣΗΣ

- Κανόνας αντικατάστασης για προτάσεις:

• Έστω ότι μία σύνθετη πρόταση περιέχει μία

πρόταση p και ότι η πρόταση p <::::> q είναι

αληθής. Τότε, η τιμή αληθείας της σύνθετης

αυτής πρότασης δεν μεταβάλλεται αν αντι­

καταστήσουμε την p με την q. - Κανόνας αντικατάστασης για παραστάσεις:

• Έστω ότι μία παράσταση Ρ' περιέχεται σε μία

άλλη παράσταση Ρ. Έστω ακόμα Q η παρά­

σταση που προκύπτει από την Ρ όταν αντικατα­

στήσουμε την Ρ' με μία άλλη παράσταση Q'.

α) Αν Ρ' Ξ Q', τότε Ρ Ξ Q .

β) Αν η Ρ είναι μία ταυτολογία και Ρ' Ξ Q',

τότε και η Q είναι ταυτολογία.

Για παράδειγμα, έστω η παράσταση:

p � (p � q) . Σ' αυτή περιέχεται η παράσταση p � q και

ισχύει: p � qΞpνq (§ 1.16).Άρα:

Ρ � (p � q) Ξ Ρ => Ρ ν q ·

1.18 ΑΛΓΕΒΡΑ ΤΩΝ ΠΑΡΑΣΤΆΣΕΩΝ

Όπως είδαμε παραπάνω (§ 1.16, πρότ. 1), η

σχέση Ξ μεταξύ των παραστάσεων είναι αυτοπα­

θής, συμμετρική και μεταβατική. Όπως λέμε στα

Μαθηματικά, είναι μία «σχέση ισοδυναμίας». Η σχέση αυτή λέγεται ακόμα και «λογική ισότητα»

ή απλά «ισότητα». Έτσι, πολλές φορές, αντί να

λέμε ότι δύο παραστάσεις είναι ισοδύναμες, λέμε

ότι είναι ίσες.

Με βάση τις ιδιότητες της σχέσεως Ξ , τον κα­

νόνα αντικατάστασης για παραστάσεις ( § 1.17) και

γνωρίζοντας μερικές απλές ισοδυναμίες ( ισότη­

τες), μπορούμε, όταν μας δώσουν μία παράσταση

Ρ, να βρούμε μία άλλη παράσταση ισοδύναμη

(ίση) με την Ρ και η οποία να περιέχει όσο το δυ-'

νατόν λιγότερους λογικούς συνδέσμους, δηλαδή,

όπως λέμε, να είναι απλούστερης μορφής. Η εργα­

σία αυτή ονομάζεται «απλοποίηση» της δοσμένης

παράστασης Ρ και το σχετικό μέρος της Λογικής

ονομάζεται «Άλγεβρα των παραστάσεων».

1 . 19 ΝΟΜΟΙ ΤΗΣ ΑΛΓΕΒΡΑΣ ΤΩΝ ΠΑΡΑΣΤΆΣΕΩΝ

Όπως είπαμε παραπάνω, για να απλοποιήσουμε

μία παράσταση, πρέπει να γνωρίζουμε ένα επαρκές

πλήθος ισοδυναμιών (ισοτήτων). Για να τονίσουμε

την σπουδαιότητα των ισοδυναμιών (ισοτήτων)

αυτών, τι� ονομάζουμε νόμους της άλγεβρας των

παραστάσεων. Οι σπουδαιότεροι από τους νόμους

αυτούς είναι οι εξής:

1. Ρ ΞΡ 2. pνpΞp, pΛpΞp 3. pνqΞqνp, pΛqΞqΛp 4. pν(qνr)Ξ(pνq)νr

p Λ ( q Λ r) Ξ (p Λ q) Λ r 5. pν(qΛr)Ξ(pνq)Λ(pνr)

p Λ ( q ν r) Ξ (p Λ q) ν (p Λ r) 6. p=:>qΞpνq

Ρ <::::> q Ξ (p => q) Λ ( q => Ρ) 7. pνqΞpΛq

pΛqΞpνq 8. p=?qΞpΛQ

(νόμοι

του deMorgan)

p <::::> q Ξ (p Λ q) ν ( q Λ p) Α ν Α είναι μία μεταβλητή, η οποία μπορεί να

αντικατασταθεί μόνο με μία οποιαδήποτε αληθή

πρόταση και Ψ είναι μία μεταβλητή, η οποία μπο-

ΕΥΚΛΕΙΔΗΣ Β' λθ' τ.2/1 2

Page 15: Ευκλειδης Β 58

------------ Στοιχεία Μαθηματικής Λογικής

ρεί να αντικατασταθεί μόνο με μία οποιαδήποτε

ψευδή πρόταση, τότε ισχύουν οι παρακάτω ισοδυ­

ναμίες: 9. Ρ v Α Ξ Α, Ρ v ψ Ξ p, Ρ Λ Α Ξ p, Ρ Λ ψ Ξ ψ

- -

10. pvpΞA, pΛpΞΨ, ΑΞΨ, ΨΞΑ - Σημειώνουμε ακόμα ότι οι λογικές πράξεις δεν

είναι ανεξάρτητες μεταξύ τους. Μερικές από αυτές

μπορούν να αντικαταστήσουν όλες τις άλλες:

· ι Οι τρεις πράξεις -, v' Λ μπορούν να αντικα­

ταστήσουν όλες τις άλλες.

Πράγματι, έχουμε:

p � q Ξ p v q και p <::::> q Ξ (p v q) Λ (q v p) . • ι Οι δύο πράξεις - , ν μπορούν να αντικατα­

στήσουν όλες τις άλλες.

Πράγματι, έχουμε:

pΛqΞpvq, p�qΞpvq και

Ρ <::::> q Ξ (p v q) v (p v q) . · ι Οι δύο πράξεις - ' Λ μπορούν να αντικατα­

στήσουν όλες τις άλλες.

Πράγματι, έχουμε:

p v q Ξ p Λ q, p � q Ξ p Λ q και

Ρ <::::> q Ξ (p Λ q) Λ (p Λ q) .

1.20 ΕΦΑΡΜΟΓΕΣ

Με βάση τα προηγούμενα μπορούμε:

α) Να aπλοποιήσουμε μία δοσμένη παράσταση.

β) Να βρούμε το είδος μιας δοσμένης παρά-

στασης. Δηλαδή, να βρούμε αν είναι ταυτολογία ή

αυτοαντίφαση ή σχετική, χωρίς να κατασκευάσου­

με τον πίνακα τιμών αληθείας της.

γ) Να βρούμε αν δύο δοσμένες παραστάσεις εί-

ναι ισοδύναμες (ίσες) ή όχι.

(Στα παρακάτω παραδείγματα, οι νόμοι που εί­

ναι γραμμένοι στις παρενθέσεις αναφέρονται

στην § 1.19).

Παράδειγμα 1 . Να απλοποιηθεί η παράσταση:

(pvq)�(q�p).

Λύση. Έχουμε:

(p v q) � ( q � Ρ) Ξ (p v q) v ( q � Ρ) Ξ(pΛq)v(qΛp) , Ξ (p Λ q) V (p Λ q) ΞpΛ(qvq) ΞpΛΑ =p

(Ν.6)

(Ν.7,8)

(Ν.3)

(Ν.5)

(N.lO) (Ν.9)

Παράδειγμα 2. Να βρείτε το είδος της παρά­

στασης:

(p v q) => l(p => q) => ql. Λύση. Έχουμε:

(p V q) => [ (p � q) � q] Ξ Ξ (p V q) => [ (p V q) � q] Ξ (p V q) => [ (p V q) V q] Ξ (p V q) => [(p Λ q) V q] Ξ (p V q) V [ (p Λ q) V q] Ξ (p V q) V [ (p V q) Λ (q V q)] Ξ (p v q) v [ (p v q) Λ Α] Ξ (p V q) V (p V q)

(Ν.6)

(Ν.6)

(Ν.7,1)

(Ν.6)

(Ν.5)

(Ν. Ι Ο)

(Ν.9)

Επειδή η τελευταία παράσταση είναι προφανώς

μία ταυτολογία, έπεται ότι και η δοσμένη παρά­

σταση είναι μία ταυτολογία.

Παράδειγμα 3. Να εξετάσετε αν οι παρακάτω

δύο παραστάσεις είναι ισοδύ­

ναμες: p v /q Λ (p v q)/ και

IP v (p => q)/ Λ /(q v (p Λ q)/ Λύση. Έχουμε: • Ρ v [ q Λ (p v q)] Ξ (p v q) Λ [p v (p v q)] Ξ

Ξ (p V q) Λ (p V q) Ξ ΞΡV (q Λ q)Ξ pvΨ Ξ p.

8 [p V (p => q)] Λ [ q V (p Λ q)] Ξ Ξ [p v (p Λ q)] Λ [ ( q v Ρ) Λ ( q v q)] Ξ Ξ (p v Ρ) Λ (p v q) Λ (p v q) Λ Α Ξ Ξ Ρ Λ (p v q) Λ (p v q) Ξ Ξ Ρ Λ [p v (q Λ q)] Ξ Ρ Λ (p v Ψ) Ξ ΞpΛpΞp.

Συμπεραίνουμε ότι οι δύο δοσμένες παραστά­

σεις είναι ισοδύναμες.

ΕΥΚΛΕΙΔΗΣ Β ' λθ ' τ.2/13

Page 16: Ευκλειδης Β 58

-------------- Στοιχεία Μαθηματικής Λογικής

ΑΣΚΗΣΕΙΣ

3. Να δείξετε, χωρίς πίνακες τιμών αληθείας, ότι: α) pv(pΛq)Ξp και pΛ(pvq)Ξp .

β) Αν ΡΛQΞΡΛR και ΡνQΞΡνR, τότε

QΞR.

4. Να βρείτε μία παράσταση P(p, q), δύο μετα­

βλητών p και q, η οποία έχει πίνακα τιμών α­

ληθείας τον παρακάτω:

Ρ q Ρ (ρ, q) α α α α Υ α Υ α Υ Υ Υ α

5. Στο σπίτι του παρακάτω σχήματος κατοικούν

δύο δίδυμα αδέλφια, εντελώς όμοια, από τους

οποίους ο ένας λέει πάντοτε ψέματα και ο άλ­

λος λέει πάντοτε την αλήθεια και ο καθένας

απαντά μόνο σε ερωτήσεις, στις οποίες δεν α­

ναφέρεται ο άλλος.

Από τους δρόμους Α και Β ο ένας οδηγεί σ'

ένα χωριό και ο άλλος σ' ένα βουνό. Ένας,

που θέλει να πάει στο χωριό, φθάνει στο σπίτι

αυτό από το δρόμο Γ και θέλει να μάθει ποιος

δρόμος από τους Α και Β οδηγεί στο χωριό.

Κάνει μία μόνο ερώτηση σ' ένα από τα δύο

αδέλφια (χωρίς να ξέρει αν είναι αυτός που

λέει αλήθεια ή αυτός που λέει ψέματα) και

βρήκε το δρόμο που οδηγεί στο χωριό. Ποια

ερώτηση είναι δυνατόν να του υπέβαλε; Και

πώς βρήκε το σωστό δρόμο;

(Σημειώνουμε ότι: α) Η ερώτηση «αν ρωτού­

σα τον αδελφό σου ποιο δρόμο θα μου έδειχνε

για να πάω στο χωριό;» αποκλείεται από τις

υποθέσεις. β) Τα προβλήματα του τύπου αυ­

τού στη Μαθηματική Λογική αντιμετωπίζο­

νται με τη σχετική θεωρία και όχι σαν προ­

βλήματα «σπαζοκεφαλιάς» ή «γρίφων» ή

«μαντεψιάς» ). (Συνεχίζεται)

Τυπογραφικές αβλεψίες και άλλα ... Η αλήθεια είναι ότι, τα μαθηματικά είναι «σκληρό κείμενο» για γρά­ψιμο. Ειδικά όταν είναι πολλές σύνθετες μορφές μαζί με γράμματα και αριθμούς ... κάτι παράξενα σύμβολα ... και όλη η προσοχή νάναι στο να μη ξεφύγει τίποτα. Όσο και να λες αποκλείεται ... όλα έχουν γραφεί κάτι ξεφεύγει στο τέλος ... Έτσι με όλη την ευθύνη της επανόρθωσης ή της παράλειψης γρά­φουμε τα «διορθωτέα» από το προηγούμενο τεύχος και με την υπό­σχεση ότι στο επόμενο ο «δαίμονας» του τυπογραφείου θάναι έξω από το περιοδικό, μια και η προσοχή μας θάναι ιδιαίτερα αυξημένη ... Αν και το τυχαίο τυπογραφικό λάθος δημιούργησε μία νέα ενδιαφέ­ρουσα άποψη σχολαστικής ανάγνωσης - όπως αναφέρουν μερικοί αναγνώστες μας - στην παρακολούθηση των μαθηματικών συλλογι­σμών σε μερικές ασκήσεις, αφού εντοπίζεται, ότι κάτι δεν πάει καλά, αναζητείται το λάθος με αποτέλεσμα να δημιουργείται μια άλλη άσκηση ή άλλος συλλογισμός με αρκετό ενδιαφέρον. Αλλά θα επα­νέλθουμε σ' αυτό ... Έτσι επιμένοντας στα ουσιαστικά σημεία αυτής της παρέμβασης . Επισημαίνουμε τα παρακάτω για το τεύχος 57: σελ. 14: στίχος 21 β' στήλης να γραφεί το ορθό: <φ είναι ικανή και

αναγκαία συνθήκη της φ> σελ. 19: ++S · να γίνει +S ·

π π σελ. 20: το ;:; 64- να γίνει: σφ-6 6 σελ. 21: i. η 2η λύση: να μπει μετά την απόδειξη της Άσκησης 3.

μέλος -3 ii. Άσκηση 4 να γίνει Α ;:; -

2 σελ. 27: 16 το ορθό ρ θετικός ακέραιος

σελ. 28: 4: αντί Jo- 5)2 το ορθό �·(1

---JS-5=-)

- 2 σελ. 29: Α4: να φύγει το 3 απ' τη λύση στις δύο τελευταίες σειρές και

το αποτέλεσμα τελικά είναι = I. σελ. 30: να φύγει στην πρώτη στήλη η έκφραση «πολλαπλασιάζοντος

1821» σελ. 33: Α3: στις προτεινόμενες να γραφεί το ορθό:

χ2 (y + z) + / (z + χ)+ z2 (χ+ y);:; 6xyz σελ. 40: Στην άσκηση 7 μπορούσε να μη δοθεί ότι Α = 45• αφού

ΒΓ2 = ΑΒ2 + ΑΓ2 - 2ΑΒ · ΑΔ = ΑΒ2 + ΑΓ2 - 2ΑΗ ·ΑΕ αφού ΒΕΗΔ εγγράψιμο => ΑΒ · ΑΔ = ΑΗ ·ΑΕ

σελ. 5 I: Στην αρχή 2η σειρά αντί Γ(3", Ο) να γραφεί Γ(3α, Ο) αντί να γραφεί το ορθό Μ

( 3α +4

3β , 4γ)

.

ΕΥΚΛΕΙΔΗΣ Β ' λθ' τ.2/14

Page 17: Ευκλειδης Β 58

ι;�� �-��� ! :J:τι, � ι )j H/JN/J NATHeNAT/CVf Η Homo Mathematιcus είναι μια στήλη στο περιοδικό μας, με σκοπό την ανταλλαγή απόψεων και την ανάπτυξη προβληματι­σμού πάνω στα εξής θέματα: I ) Τι είναι τα Μαθηματικά, 2) Πρέπει ή όχι να διδάσκονται, 3) Ποιοι είναι οι κλάδοι των Μαθημα­τικών και ποιο το αντικείμενο του καθενός, 4) Ποιες είναι οι εφαρμογές τους, 5) Ποιες επιστήμες ή κλάδοι επιστημών απαιτούν καλή γνώση των Μαθηματικών για να μπορέσει κάποιος να τους σπουδάσει. Για τους συνφγ{ιη;ς της στι']/.ιjς : παράκληση ! τα κείμενα της στήλης αυτής, ως προς το περιεχόμενό τους και ως προς το επί­πεδό τους, θα πρέπει να είναι συμβιβαστά με τα ενδιαφέροντα και το επίπεδο κατανόησης από μέρους των παιδιών.

I . "που και με ποιο τpl)πο εφαρμl}(ο !!ται τα ΜαΟιzματιh·ά,· "

του Γιάννη Κερασαρίδη Τα Μ αθη ματικά και η τέχνη της Εκτίμησης

Π ρολεγόμενα Ένας καλός φίλος μόλις είχε γυρίσει από τις έκλεισε η κουβέντα. Όμως, το ερώτημά του το

καλοκαιρινές διακοπές του και έσπευσε να με βά- κλωθογύριζα στο μυαλό μου και άρχισα να ψάχνω λει σε . . . μπελάδες. Ούτε λίγο ούτε πολύ μου ζήτα- και να ρωτάω. Μετά από πολλές αναζητήσεις βρή­γε να του εξηγήσω πως γίνεται κι ένα πολύ μικρό κα μια εργασία του G.V. Meledin με τίτλο «Η τέ­κουνουπάκι πετώντας κάνει εκείνο τον εκνευρι- χνη της εκτίμησης». Ήταν το κατάλληλο άρθρο για στικό βόμβο, ενώ ένα μεγάλο (πχ. , η λιβελούλα) να κάνω τον . . . «ξύπνιο» στο φίλο μου. Εμείς το παρά το πολλαπλάσιο μέγεθος φτερών και σώμα- δημοσιεύουμε για να δουν τα παιδιά ότι χωρίς τα τος, πετά αθόρυβα. Του υπενθύμισα πως είμαι μα- Μαθηματικά μέσα δεν θα μπορούσε να μελετηθεί θηματικός και όχι εντομολόγος ή φυσικός και εκεί η πτήση (άλλοτε θορυβώδης και άλλοτε αθόρυβη)

«Η τέχνη της εκτίμησης»(πρόβλημα 7), του C.V. Meledin «Είναι φυσικό να υποθέσουμε ότι ο ήχος πα- ( 1 / 1 0)13 για τον όγκο του κουνουπιού, εφόσον το

ράγεται από το περιοδικό φτερούγισμα του κου- πλάτος και το ύψος του είναι σημαντικά μικρότερο νουπιού. Στην πραγματικότητα, η φυσική της πτή- από το μήκος του. Θα θεωρήσουμε ότι η πυκνότη­σης του κουνουπιού δεν είναι τόσο απλή . Θα χρη- τα του σώματος του εντόμου είναι ίση με την πυ­σιμοποιήσουμε, ωστόσο, ένα εξαιρετικά χονδροει- κνότητα του νερού Pv· Συμβολίζοντας τη συχνότη­δές μοντέλο, υποθέτοντας ότι το φτερούγισμα πα- τα του φτερουγίσματος με ν, έχουμε υ-Ιν. Η συν­ράγει ανυψωτική δύναμη που αντισταθμίζει την θήκη ισορροπίας μας δίνει έλξη της βαρύτητας. Αυτή η ανυψωτική δύναμη F == Ρaυ212 == Paυ214=mg == Pvl3g! I O δίνεται από τη μεταβολή της ορμής (η οποία μετα- το οποίο οδηγεί στην φέρεται στον αέρα που βρίσκεται κάτω από τα .-----

φτερά, ανά μονάδα χρόνου) : ΔP/Δt=mg

Κινούμενα προς τα κάτω, τα φτερά ωθούν προς τα κάτω μάζα αέρα Δm=ρ0υΔtS σε χρονικό διάστημα Δt (όπου : S η επιφάνεια των φτερών, υ η ταχύτητά τους, Ρο η πυκνότητα του αέρα). Αυτή συνοδεύεται από μεταφορά ορμής ΔΡ=Δmυ=ρ0υ2ΔtS . Η προκύπτουσα ανυψωτική δύναμη είναι

I

Το μήκος του κουνουπιού είναι, ας πούμε, 1�4mm. Η επιφάνεια των φτερών του είναι S-12 (υποθέτουμε ότι το μήκος του σώματος του κου­νουπιού είναι της ίδιας τάξης μεγέθους με το ά­νοιγμα των φτερών του). Είναι εύλογο να πάρουμε

ν η == ν == Ρν g ____;_..:...__ · - == 400 Hz Ι Ο ·ρ a · I I Το αποτέλεσμα δίνει μια εύλογη τάξη μεγέ­

θους, αν και δεν μπορούμε να είμαστε πολύ κατη­γορηματικοί για το ψηφίο «4».

Αυτός ο τύπος οδηγεί στην πρόβλεψη ότι η συχνότητα μεταβάλλεται αντιστρόφως ανάλογα προς τη τετραγωνική ρίζα του I . Με άλλα λόγια, όσο μεγαλύτερο είναι το έντομο τόσο χαμηλότερης συχνότητα είναι ο ήχος που παράγει . Πραγματικά η σύγκριση του βόμβου μιας αγριομέλισσας με τον υψηλής συχνότητας ήχο ενός κουνουπιού δείχνει ότι αυτή η πρόβλεψη είναι δικαιολογημένη»

[πηγή : "Quantum", τόμ. 4, τεύχ. 6, εκδ. "ΚΆΤΟΠΤΡΟ"] 1 1 . ' 'Α υτδ το ξιiρατι:; "

Τι είναι το «ζεύγος τριγώνων Robίnson» και που μας χρησιμεύουν; [η απάντηση στο τέλος της στήλης]

1 1 1 . "Οι συ vεργάτε( τικ στιμης γράφοιJ ΙΙ-ερω τοι)v "

ΕΥΚΛΕΙΔΗΣ Β' λθ ' τ.2/15

Page 18: Ευκλειδης Β 58

Homo Mathematicus

Α. Η τριχοτόμηση της γωνίας Π ρολεγόμενα. Ήμασταν έτοιμοι να δημοσιεύσουμε μια σειρά

δύο άρθρων-αφιέρωμα στο μαθηματικό πρόβλημα των "πλακοστρώσεων" (ένας καινούργιος κλάδος των Μαθηματικών), όταν λάβαμε μια εργασία που, για μας, ήταν μεγάλη "πρόκληση" . Συγκεκριμένα, ο φίλος της στήλης μας και μέλος της ΕΜΕ, Ιππο­κράτης Π. Δάκογλου (Χολαργός), μας έστειλε μια εργασία του πάνω στο περίφημο θέμα της τριχοτό­μησης μιας γωνίας με τη βοήθεια των ιδεών του Πυθαγόρα. Εμείς, σε πρώτη φάση, δεν θα πάρουμε θέση πάνω στο θέμα, περιμένοντας τις αντιδράσεις

των αναγνωστών μας, με έναν όρο: στο βαθμό που είναι δυνατόν, οι τοποθετήσεις σας να είναι, όσο γίνεται, κατανοητές από τους μαθητές Λυκείου. Στο τέλος, θα πάρουμε και εμείς θέση . Παραθέ­τουμε αυτούσιο το άρθρο (διατηρώντας την ορθο­γραφία και σύνταξη του συγγραφέα. Δυστυχώς, λόγω έλλειψης της σχετικής γραμματοσειράς δεν μπορούμε να αποδώσουμε τις «ψιλές», τις «δασεί­ες», τις «περισπωμένες» κλπ. με τις οιιτοίες είναι γραμμένο το κείμενο) .

«Το πρόβλημα της τριχοτόμησης της γωνίας», από τον Ι . Π .Δάκογλου «Στο τεύχος 56/2005 και στη στήλη "Homo Α. γωνία ω0=3°·λ, με λ= ι ,2 ,3 ,4, . . .

mathematicus" και στη σελίδα ι s δίδεται απάντη- Β. γωνία ω0=(90°·λ)/(2+λ), με λ= 1 ,2,3 ,4, . . . ση σε ερώτημα του μαθητού Θ. Κίντζη σχετικά με Γ. γωνία ω0=(270°·λ)/(2+3λ), με λ= 1 ,2,3 ,4, . . . την τριχοτόμηση γωνιών. Επισυνάπτονται ενδεικτικά τρία παραδείγματα

Η απάντηση που δόθηκε βασίστηκε σε πηγή τριχοτομήσεως ένα από κάθε ομάδα των κάτωθι γνώσεων του έτους 1 970 όπως ακριβώς αναγράφε- γωνιών: ται στο τέλος της απαντήσεως. Α) Από την ομάδα (Α) για λ= ι 2 οπότε είναι ω-

Όμως υπάρχει νεώτερη πηγή η οποία όχι μόνο 0=3 · 1 2=36° καταρρίπτει το οριστικό του αδυνάτου της τριχο- Β) Από την ομάδα (Β) για λ=2 οπότε είναι ω-τομήσεως, μόνο με κανόνα και διαβήτη, λόγω υ- 0==(90°·2)/(2+2)=45° πάρξεως λάθους στη μαθηματική του απόδειξη, Γ) Από την ομάδα (Γ) για λ= ι οπότε είναι ω-αλλά και παρέχει αποδείξεις και πολλά παραδείγ- 0=(270°· 1 )/(2+3 · 1 )=54° ματα τριχοτομήσεως των κάτωθι γωνιών ομάδων ΆΠΕΙΡΩΝ γωνιών.

Παράδειγμα 1"

Να τριχοτομηθεί μία γωνία 36°=3 ·12° με τη βοήθεια μόνο κανόνα και χάρακα Λύση

Το πρόβλημα έχει δύο σκέλη ._ Αρχικά πρέπει Η αντίστοιχη κατασκευή ενός κανονικού ι Ογώνου μπορεί να γίνει με πολλούς τρόπους, είτε με την πυθαγόρεια γενική μέθοδο κατασκευής ό­λων των κανονικών πολυγώνων, . . . είτε με ο­ποιονδήποτε τρόπο.

να βρεθεί ο τρόπος της κατασκευής αυτής ταύτης της δοθείσας γωνίας και ύστερα να τριχοτομηθεί.

Για το πρώτο σκέλος παρατηρούμε ότι η δο­θείσα γωνία των 36° αντιστοιχεί σε μια επίκεντρη γωνία ενός κανονικού ι Ογώνου εγγεγραμμένου σ' έναν τυχόντα κύκλο, διότι 360°/36°= 1 0°.

Άρα μπορούμε να κατασκευάσουμε τη γωνία των 36° με την βοήθεια της κατασκευής ενός κα­νονικού Ι Ογώνου.

σ . . · · · · · · · · · · :-. . .

a/2

c/2

α ' i &----····------·

σχήμα ι

\ \

.:-.< � · . ·· '

\ ·, i \l e

Στο σχέδιο που συνοδεύει την παρούσα λύση, η κατασκευή της γωνίας των 36° γίνεται με τη βο­ήθεια της κατασκευής ενός τυχόντος ισοσκελούς τριγώνου με κορυφή τις 36°, οπότε η βάση θα είναι διπλάσια της κορυφής. Ήτοι με βάση την πρόταση 1 0/IV των στοιχείων του Ευκλείδη, η οποία χρησι­μοποιεί τη χρυσή τομή τμήματος ευθείας, πράγμα που πολλοί μαθηματικοί την θεωρούν ως πυθαγό­ρεια.

Για το δεύτερο σκέλος του προβλήματος της τριχοτόμησης της γωνίας των 36° που ήδη κατα­σκευάστηκε στην κορυφή του τριγώνου ΑΒΓ, ερ­γαζόμαστε ως εξής:

Ξεκινάμε από το σημείο Α και την ευθεία ΑΔ την κατασκευή των δύο ορθογωνίων τριγώνων της πυθαγόρειας πολυγωνικής έλικας ΑΒΕΖ. Μεταφέ­ρουμε την ακτίνα της έλικας ΑΖ στην προέκταση ΑΖ' της ευθείας ΑΒΔ. Η ακτίνα ΑΖ=ΑΖ ' = J3 .

ΕΥΚΛΕΙΔΗΣ Β' λθ ' τ.2/16

Page 19: Ευκλειδης Β 58

Homo Mathematicus

Στη θέση ΑΖ ' κατασκευάζουμε το ορθογώνιο τρίγωνο . . ΑΖ ' Θ με μήκη των καθέτων πλευρών ΑΖ '= J3 μονάδες, Ζ 'Θ= 3 μονάδες, οπότε η γωνία ΘΑΖ '=60°. Η προέκταση της υποτείνουσας ΘΑ

τέμνει τη μεσοκάθετο της πλευράς ΑΓ στο σημείο Ι. Λέμε ότι η ευθεία ΓΙ είναι η ζητούμενη τριχοτό­μος. Πράγματι η γων.ΙΓΑ=γων.ΙΑΓ=72°-60° = \ 2° =36°/3 , όπως ακριβώς φαίνεται στο σχήμα I »

Β. Εφαρμογές από τη «θεωρία του μαθη ματικού μπιλιάρδου» Προλεγόμενα Από τον φίλο της στήλης μας, Η επιστολή : «Αγαπητέ Homo mathematicus,

μαθηματικό Θάνο Λύπα (Φάληρο), λάβαμε το πα- επειδή στη στήλη σου βλέπουν το φως της δημο­ρακάτω μήνυμα και συνημμένα μια εργασία με τον σιότητας "παράξενα" θέματα, αποφάσισα να σου ισχυρισμό ότι τα Μαθηματικά έλυσαν το «πρό- στείλω μια εργασία (προϊόν ψαξίματος) με την ο­βλ η μα του αεικίνητου». Σας τα παραθέτουμε αυ- ποία φαίνεται πως τα Μαθηματικά "λύνουν" , ακό­τούσιο το μήνυμα. Την εργασία θα τη δημοσιεύ- μη, και τα "άλυτα" προβλήματα της Φυσικής. σουμε σε δύο συνέχειες, λόγω έλλειψης χώρου. Με εκτίμηση

Θάνος Λύπας «Τα Μαθ ικό), από το Θάνο Λύπα

«Είναι αδύνατο να μεταφερθεί θερμότητα από ένα σώμα σε κάποιο άλλο χωρίς κατανάλωση ενέργειας εάν η θερμοκρασία του πρώτου σώματος δεν είναι υψηλότερη από του δεύτεροω>

Αυτή η απαγόρευση δεν μοιάζει ιδιαίτερα πει- ξουν το ρόλο ενός παρόμοιου «δαίμονα», όλες, στική . Ισχύει άραγε πάντοτε; Ο μεγάλος Άγγλος όμως, στην πράξη αποτυγχάνουν εξαιτίας προβλη­φυσικός James Maxwell επινόησε ένα μηχανισμό μάτων που δεν έχουν προβλεφτεί. (είναι γνωστός ως «δαίμονας του Maxwell») με Παρά ταύτα, εκεί που απέτυχε η Φυσική, ίσως σκοπό να καταρρίψει αυτό το νόμο. Φανταστείτε τα καταφέρουν τα Μαθηματικά. Θέλω να προτεί­ένα δοχείο χωρισμένο σε δύο ίσα τμήματα από νω μια συσκευή, χωρίς δαίμονες ή κακόβουλα ένα διάφραγμα. Τ ο διάφραγμα έχει μια μικρή τρύ- πνεύματα, που είναι δυνατόν να κατασκευαστεί πα και κοντά σ ' αυτή στέκεται ένας «δαίμονας». ακόμη και σ' ένα σχολικό εργαστήριο. Θα παρου­Γεμίζουμε το ένα τμήμα του δοχείου με κάποιο σιάσω υπολογισμούς που αποδεικνύουν πεντακά­αέριο και ζητούμε από τον «δαίμονα» να επιτρέπει θαρα ότι αυτή η συσκευή μπορεί να μεταφέρει να περνούν προς το άλλο τμήμα του δοχείου μόνο θερμότητα από ένα σώμα σε κάποιο άλλο όταν αυ­τα μόρια που έχουν μεγάλη ταχύτητα. Αφού η μέ- τά έχουν αρχικά την ίδια θερμοκρασία. ση κινητική ενέργεια για κάθε μόριο του αερίου Πριν προχωρήσουμε στην παρουσίαση του χαρακτηρίζει τη θερμοκρασία του αερίου, είναι «αεικίνητού» μου, πρέπει να aφομοιώσετε κάποιες φανερό ότι το αέριο στο ένα τμήμα του δοχείου θ' μαθηματικές γνώσεις έτσι ώστε να πειστείτε για αρχίσει να ψύχεται, ενώ στο άλλο να θερμαίνεται. την ορθότητα των συλλογισμών και των κατα­Έχουν προταθεί διάφορες συσκευές που, σύμφωνα σκευών που θ' ακολουθήσουν. Θα ασχοληθούμε, με τους σχεδιαστές τους, θα μπορούσαν να παί- λοιπόν, με την έλλειψη και τις ιδιότητές της. ορισμός Έλλειψη είναι η καμπύλη που σχηματίζεται απ ' όλα εκείνα τα σημεία του επιπέδου για τα οποία το άθροισμα των αποστάσεών τους από δύο συγκεκριμένα σημεία F1 και F2 είναι σταθερό.

Τα σημεία F 1 και F2 ονομάζονται εστίες της έλλειψης, και το σταθερό άθροισμα των αποστάσεων συμβολίζεται συνήθως με 2α.

Την ιδιότητα αυτή τη χρησιμοποιούν οι κηπουροί όταν θέλουν να φτιάξουν παρτέρια λουλουδιών με οβάλ σχήμα. Καρφώνουν στο χώμα δύο ραβδιά (στις εστίες), δένουν σ' αυτά τις άκρες ενός σχοινιού, και παίρνουν ένα ακόμη ραβδί με μυτερή άκρη με το οποίο τεντώνουν το σχοινί. Α ν μετακινήσουμε το τρίτο ραβ�ί έτσι ώστε το σχοινί να παραμείνει τεντωμένο, η μυτερή άκρη θα σχεδιάσει στο χώμα μια έλ­λειψη. Το μέγεθος και το σχήμα της έλλειψης εξαρτάται από το μήκος του σχοινιού και από την απόστα­ση μεταξύ των εστιών. Αυτό μπορείτε να το επαληθεύσετε εύκολα σ' ένα φύλλο χαρτιού χρησιμοποιώ­ντας δύο πινέζες κι ένα μολύβι αντί για ραβδιά.

Σχεδιάζοντας διάφορες ελλείψεις θα διαπιστώσετε ότι κάθε έλλειψη είναι μια κλειστή κυρτή καμπύ­λη μ' ένα κέντρο συμμετρίας και δύο άξονες συμμετρίας (το τμήμα F 1 F2 και τη μεσοκάθετό του). Επί­σης, βλέπουμε εύκολα ότι για τα σημεία που βρίσκονται στο εσωτερικό της έλλειψης το άθροισμα των aποστάσεών τους από τις εστίες είναι μικρότερο από 2α , ενώ για τα σημεία που βρίσκονται στο εξωτε­ρικό της είναι μεγαλύτερο.

ΕΥΚΛΕΙΔΗΣ Β' λθ ' τ.2/17

Page 20: Ευκλειδης Β 58

Homo Mathematicus

Ιδιότητες Η τελευταία πληροφορία αρκεί για ν' αποδείξουμε μια σημαντική και όχι ιδιαίτερα προφανή ιδιότητα της έλλειψης: τα ευθύγραμμα τμήματα που συνδέουν τις εστίες μιας έλλειψης μ ' ένα τυχαίο σημείο της Μ, σχηματίζουν ίσες γωνίες με την εφαπτόμενη της έλλειψης στο σημείο Μ

Οπτική ιδιότητα της έλλειψης Α ν συγκρίνουμε αυτή την ιδιότητα με τον νόμο

της ανάκλασης του φωτός (η γωνία πρόσπτωσης ι­σούται με τη γωνία ανάκλασης), καταλήγουμε στην εξής διατύπωση: μια ακτίνα φωτός που πηγάζει από μια εστία της έλλειψης μετά την ανάκλασή της στην έλλειψη διέρχεται από την άλλη εστία.

Αυτή είναι η «οπτική ιδιότητα» της έλλειψης, που μπορούμε να την παρατηρήσουμε στη φύση.

_Q �\�!

σχήμα 2

Υπάρχουν σπηλιές με ελλειψοειδείς θόλους όπου μπορούμε να βρούμε δύο σημεία, αρκετά απομα­κρυσμένα μεταξύ τους, τέτοια ώστε όταν μιλάει ένας άνθρωπος σ ' ένα σημείο, κάποιος άλλος που κάθε­ται στρ δεύτερο τον ακούει σαν βρίσκεται δίπλα του. Υπάρχουν, επίσης, αίθουσες σε παλάτια και κάστρα κατασκευασμένες επί τούτου με τέτοιο τρόπο ώστε να δημιουργείται το συγκεκριμένο φαινόμενο. Επει­δή η οπτική ιδιότητα της έλλειψης θα παίξει σημα­ντικό ρόλο σε όσα επακολουθήσουν, θα παρου­σιάσω την απόδειξή της (είναι μάλλον σύντομη και απλή).

Η συνέχεια στο επόμενο

σχήμα 3

Ι lα . "Α υτό το έ ατε · " lη απάντηση) Τι είναι: «ζεύγος τριγώνων Robίnson» λέμε ότι

αποτελούν δύο ισοσκελή τρίγωνα (τι), (τ2), με γωνίες 3r/, 72°, 72° και 1 08°, 36°, 3r/, αντίστοιχα, τα ο­ποία είναι τέτοια ώστε η βάση του (τι) να έχει το ί­διο μήκος με τις ίσες πλευρές του (τ2) .

Στα τρίγωνα αυτά συνήθως θεωρούμε ότι η βάση του (τ ι ) και οι ίσες πλευρές του ( τ2) έχουν μοναδιαίο μήκος, δηλ. , Ι μονάδα μήκους.

Οι δύο ίσες πλευρές ΑΒ,ΑΓ του (τ 1 ) είναι ίσες με τη βάση ΒΔ του (τ2) [γων.Α=γων.Δ=36°, άρα το τρίγωνο ΑΒΔ είναι ισοσκελές, άρα ΑΓ=ΑΒ=ΒΔ=χ] (σχήμα 1 ) .

Τα τρίγωνα ΑΒΔ και ΒΓ Δ είναι όμοια γων.ΒΔΓ κοινή, γων. ΔΑΒ=γων.ΓΒΔ=36°,

ο , ΑΒ ΒΔ ΑΔ , γων.ΔΓΒ=γων.ΑΒΔ= 1 08 , αρα - = - =- η χ χ χ + ι - - - - --

ι ι χ

ΒΓ ΓΔ ΒΔ

απ' την οποία x2-x- l =O, άρα

ι + νΓs ι + JS χ= -- , δηλ., ΑΓ=ΑΒ=ΒΔ=χ= -- , δηλ. , το 2 2

11 Ι . Της τελευταίας στιγμής

μήκος αυτό είναι ίσο με το περίφημο Φ, σταθερά της Χρυσής τομής ή της διαίρεσης σε μέσο και άκρο λόγο άκρο λόγο

Β σχήμα 4

Που χρησιμεύουν : Τα τρίγωνα Robinson χρη­σιμεύουν στα Μαθηματικά των ημικρυστάλλων, ενός νέου είδους υλικού που ανακαλύφθηκε το 1 984. Οι κατασκευές αυτές είχαν ονομαστεί «δια­τάξεις Penrose», από το όνομα του μαθηματικού Roger Penrose που πρώτος ( 1 97 4) ανακάλυψε τις λεγόμενες «ημιπεριοδικές πλακοστρώσεις»

Σε "πηγαδάκι" της συνέλευσης της ΕΛΜΕ Νότιας Αθήνας αλιεύσαμε ( ωτακούοντας) το παρακάτω "πρόβλημα του D. Tamarkin" : <<Χρωματίζουμε κόκκινες q κορυφές ενός κανονικού (6ν+ 1)-γώνου και μπλε όλες τις υπόλοιπες. Αποδείξτε ότι το πλήθος των ισοσκελών τριγώνων που έχουν κορυφές ίδιου χρώματος εξαρτάται μόνο από το πλήθος και όχι από τη διευθέτηση των χρωμάτων»

ΕΥΚΛΕΙΔΗΣ Β ' λθ ' τ.2/18

Page 21: Ευκλειδης Β 58

Επιμέλεια: Σω '

«46η ΔΙΕΘΝΗΣ ΜΑΘΗΜΑ, TIKH ΟΛ ΥΜΠIΑΔΑ» Merίtla,. Yucatάn, Μεξικό

13-14 /ουλίου 2005 Η 46η Δ ιεθνής Μαθηματική Ολυμπιάδα διεξήχθη στην πόλη Merίda της πολιτείας Yucatan του Μεξικού από 9-1 9

Ιουλίου 2005. Η Ελληνική αποστολή συμμετείχε με τους μαθητές: 1. Θεοδόσιος Δουβρόπουλος (Χάλκινο Μετάλλιο), 2. Φραγκί­

σκος Κουφογιάννης, 3. Ιωάννης Παναγέας, 4. Μάριος Παπαμιχάλης (Εύφημη Μνεία), 5. Θεοφάνης Παπαμιχάλης, 6. Ελευθέριος Τσιώκος (Χάλκινο Μεττάλιο).

Συνοδοί της αποστολής ήταν οι Μαθηματικοί Ευστράτιος Ράππος (αρχηγός), και Δημήτριος Κοντογιάννης (υπαρχηγός). Την αποστολή συνόδευσε επίσης το μέλος του συμβουλευτικού συμβουλίου (JMO Adνίsory Board - ΙΜΟΑΒ), Καθ. Νικόλα­ος Αλεξανδρής ο οποίος επιλέχθηκε υποψήφιος για τη θέση εκλεκτού μέλους του ΙΜΟΑΒ.

Πρόβλημα 1 Έξι σημεία επιλέγονται στις πλευρές ενός ισό­πλευρου τριγώνου ABC: Τα σημεία Α1, Α2 στην πλευρά BC, τα Βι , Β2 στην πλευρά CA, τα Cι, C2 στην πλευρά ΑΒ. Τα σημεία αυτά είναι κορυφές ενός κυρτού εξαγώνου ΑιΑ2ΒιΒ2CιC2 το οποίο έχει πλευρές ίσου μήκους. Να αποδείξετε ότι οι ευθείες Α1Β2, ΒιC2 και C1A2 διέρχονται από το ίδw σημείο. Απόδειξη

Έστω Ρ σημείο στο εσωστερικό του τριγώνου ABC τέτοιο ώστε το τρίγωνο Α1Α2Ρ να είναι ισόπλευρο, παρατηρώντας ότι ΡΑ1 < AC1 και ΡΑ2 < CB2 οπότε το σημείο Ρ είναι εσωτερικό του AιAzB ιBzC ιCz. Ε­πειδή ισχύει ότι A1P // C1C2 και Αι Ρ = clc2 = AIC2 ' το τετράπλευρο A,PC ,Cz είναι ρόμβος. Ομοίως το AzPBzBι είναι ρόμβος, άρα το τρίγωνο C1B2P είναι ισόπλευρο. Θέτουμε φ = Β2Β1Α2 , ψ = Β1Α2Α1 και ω = C1C2A1 • Οι φ και ψ είναι εξωτερικές γωνίες του τριγώνου CB1A2 με C = 60° άρα φ + ψ = 240° . Παρατηρούμε επίσης ότι Β2ΡΑ = φ και C1PA1 = ω , άρα

φ + ω = 360° - (C1PB2 - Α1ΡΑ2 ) = 240° . Α

Β c

Συνεπώς ψ = ω. Όμοια έχουμε C1B2B1 = ψ , άρα τα τρίγωνα Α ιΑzΒ ι , Β ιΒzC ι και C ιCzAι είναι ίσα, ε­πομένως το τρίγωνο A 1B 1C 1 είναι ισόπλευρο. Αυ­τό σημαίνει ότι οι B ιCz, Α ιΒz και C ιAz είναι με­σοκάθετες των A 1C 1 , C 1B 1 και Β 1Α1 αντίστοιχα, οπότε το ζητούμενο έχει αποδειχτεί. Πρόβλημα 2 Έστω α1, α2, . . . μία ακολουθία ακέραιων αριθμών με άπειρο πλήθος θετικών όρων και άπειρο πλή­θος αρνητικών όρων. Υποθέτουμε ότι για κάθε θετικό ακέραw η, οι αριθμοί α1, α2, . . . , αn αφήνουν η διαφορετικά υπόλοιπα διαιρούμενοι με τον α­ριθμό η. Να αποδείξετε ότι κάθε ακέραιος εμφανί­ζεται ακριβώς μία φορά στην ακολουθία.

ΕΥΚΛΕΙΔΗΣ Β' λθ' τ.2/19

Page 22: Ευκλειδης Β 58

Μαθηματικοί Διαγωνισμοί - Μαθηματικές Ολυμπιάδες

Λύση

Η υπόθεση του προβλήματος είναι ισοδύναμη με τους αριθμούς α , , α2, . . . , αΝ να αποτελούν πλήρες σύστημα καταλοίπων modulo Ν για κάθε Ν. Πα­ρατηρούμε ότι αν i < j τότε α; '# αj , διαφορετικά το σύνολο {α1 , • • • , αj } θα περιείχε το πολύ j - 1 διαφορετικά κατάλοιπα modulo j . Επιπλέον, αν i < j � n τότε Ια; - αj l :::; η - 1 γιατί αν m = Ια; - αj l � η τότε το σύνολο {α 1 , • • • , αm} θα εί­χε δύο αριθμούς ισοϋοπόλοιπους ως προς modulo

m που είναι αδύνατο. Έστω τώρα η � 1 και έστω ί(η) και j(η) δείκτες τέ­τοιοι ώστε ο αριθμός αi<n> και αj<n> να είναι αντί-στοιχα ο μικρότερος και ο μεγαλύτερος μεταξύ των α 1 , α2, . . . , αη . Από τα παραπάνω έχουμε ότι lαi<n> - αj(nJ I = η - 1 και συνεπώς το σύνολο { α1 , • • • ,

αη } αποτελείται από όλους τους ακέραιους μεταξύ των αi<n> και α j(n) . Έστω χ ένας τυχαίος ακέραιος . . Αφού αk < Ο για άπειρο πλήθος δεικτών k και οι όροι της ακολου­θίας είναι διαφορετικοί μεταξύ τους, συμπεραί­νουμε ότι θα υπάρχει i τέτοιο ώστε α1 < χ . Ομοίως

Λύση

Προσθέτοντας και αφαιρώντας στους aριθμητές τους δύο τελευταίους όρους των παρανομαστών, η ζητούμενη ανισότητα γράφεται ισοδύναμα

x2 + y2 + z2 x 2 + y2 + z2 x 2 + y2 + z2 5 ? ? + 5 ? ? + 5 ? ? � 3 χ + y- + z- y + z- + χ - z + χ - + y-

Από την ανισότητα (Β - C - S) σε συνδιασμό με την ανισότητα xyz � 1 , έχουμε:

(χ 5 + y2 + z2 )(yz + y2 + zz ) � � (χ5 Ι 2 (yz)ι ι 2 + yz + z2 ) � (x z + yz + zz )z

ή

Αθροίζοντας κυκλικά και χρησιμοποιώντας το γε­γονός ότι χ 2 + y2 + z2 � yz + zx + xy παίρνουμε:

x2 + y2 + z2 x 2 + y2 + z2 x 2 + y2 + z2 5 ? ? + 5 ? ? + 5 ? ? :::; χ + y- + z- y + z- + χ - z + χ- + y-

� 2 + y:+� + x� :::; 3, χ- + y- + z-και το ζητούμενη έχει αποδειχτεί.

Πρόβλημα 4 Θεωρούμε την ακολουθία α1, α2, • • • που ορίζεται

θα υπάρχει j τέτοιο ώστε χ < α j , και αν θεωρή- από τη σχέση σου με η > max {i , j} τότε κάθε αριθμός μεταξύ του απ = 2n + 3n + 6" - 1 (η = 1, 2, . . . ). α; και αj (άρα συμπεριλαμβανομένου και του χ) θα υπάρχει στο σύνολο {α 1 , • • • , αη } .

Π αρατήρηση :

Δεν είναι αρκετό να αποδείξουμε ότι κάθε όρος της

ακολουθίας είναι μοναδικός αφού για παράδειγμα

το σύνολο των άρτιων έχει αυτή την ιδιότητα αλλά

δεν περιέχει όλους τους ακέραιους. Επίσης η από­

δειξη ότι για κάθε n το σύνολο {α1, . • • , αn} αποτελεί­

ται από n διαδοχικούς ακεραίους μπορεί να γίνει με

επαγωγή.

Πρόβλημα 3 Έστω χ, y και z θετικοί πραγματικοί αριθμοί τέτοιοι ώστε xyz � 1 . Να αποδείξετε ότι:

xs _ χ2 ys _ y2 zs _ zz ----,----,- + + > ο xs + y2 + z2 ys + z2 + χ2 zs + χ2 + yz -

Να προσδιορίσετε όλους τους θετικούς ακέραι­ους αριθμούς, οι οποίοι είναι σχετικά πρώτοι με κάθε ένα από τους όρους της ακολουθίας. Λύση

Παρατηρούμε ότι α" = (2" + 1) · (3" + 1) - 2 . Επει-δή 3" + Ι = 2 · t αποκλείονται οι άρτιοι θετικοί α­κέραιοι αριθμοί. Για η = 2 ::::::> α2 = 48 = πολλ. 3 . Άρα αποκλείονται και τα πολλαπλάσια του 3 . Ε­πειδή κάθε ακέραιος μεγαλύτερος του 3 αναλύεται σε γινόμενο δυνάμεων πρώτων αριθμών ας θεω­ρήσουμε τον πρώτο αριθμό p > 3 . Από το θεώρημα του Fennat έχουμε: (2p- ι = kp + I και 3p-1 = f · p + I) ή (6 · 2p-2 = 3kp + 3 και 6 · 3p-2 = 2fp + 2) ή (6 · (2p-2 + 1) = 3kp + 9 και 6 · (3p-2 + Ι = 2fp + 8). Άρα: 36 · (αΡ_2 + 2) = c · p + 72 οπότε 22 32 / 22 32 I · · αp-2 = c · Ρ ::::::> Ρ • · αp-2 ::::::> Ρ αp-2

ΕΥΚΛΕΙΔΗΣ Β' λθ' τ.2/20

Page 23: Ευκλειδης Β 58

Μαθηματικοί Διαγωνισμοί - Μαθηματικές Ολυμπιάδες

Άρα κάθε αριθμός πρώτος έστω p > 3 διαιρεί κά­ποιον όρο της ακολουθίας αρ. Και επειδή το ίδιο συμβαίνει και με τους πρώτους 2,3 η μοναδική λύ­ση είναι ο αριθμός Ι .

Παρατήρηση :

Λύνεται και με γνώση ισοτιμιών ως προς mod p.

Πρόβλη μα 5

Δίνεται ένα κυρτό τετράπλευρο ABCD έτσι ώ­στε οι πλευρές BC και AD να είναι ίσες και να μην είναι παράλληλες. Θεωρείστε τα σημεία Ε και F, διαφορετικά των κορυφών, τα οποία α­νήκουν στις πλευρές BC και AD αντίστοιχα και ικανοποιούν τη σχέση ΒΕ = DF, Οι ευθείες AC και BD τέμνονται στο σημείο Ρ, οι ευθείες BD και EF τέμνονται στο σημείο Q, οι ευθείες EF και AC τέμνονται στο σημείο R. Θεωρείστε όλα τα τρίγωνα PQR καθώς τα σημεία Ε και F κι­νούνται πάνω στις πλευρές. Να αποδείξετε ότι οι περιγεγραμμένοι κύκλοι αυτών των τριγώνων έχουν και ένα άλλο κοινό σημείο εκτός από το Ρ� Λύση

Θεωρούμε τις μεσοκάθετους των AC και BD οι οποίες τέμνονται στο Ο. Θα αποδείξουμε ότι οι περιγεγραμμένοι κύκλοι των τριγώνων PQR διέρ­χονται από το Ο, το οποίο είναι σταθερό. Από τις ισότητες ΟΑ = OC, ΟΒ = OD και DA = BC έχουμε ότι τα τρίγωνα ODA και OBC είναι ί­σα. Συνεπώς η στροφή γύρω από το Ο υπό γωνία ίση με την BOD μεταφέρει το σημείο Β στο D και το σημείο C στο Α. Επειδή ΒΕ = DF, η στροφή ε­πίσης μεταφέρει το σημείο Ε στο F. Αυτό σημαίνει ότι ΟΕ = OF και ότι EOF = BOD = COA . Από τις ισότητες αυτές έχουμε ότι τα τρίγωνα EOF, BOD και COA είναι όμοια.

c

Α

Ας υποθέσουμε ότι οι ΑΒ, CD και EF δεν είναι παράλλτjλες. Έστω Χ το σημείο τομής των ευθειών EF και CD. Από το θεώρημα Μενελάου έχουμε:

AR AF DX CE DX DQ - = - · - = - · - = -RC FD XC ΕΒ XC QB Στην περίπτωση που ΑΒ 11 EF // CD το τετρά­πλευρο ABCD είναι ισοσκελές τραπέζιο και τα Ε, F είναι τα μέσα των πλευρών. Και στην περίπτωση

. . . . AR DQ αυτη εχουμε προφανως οτι - = - . RC QB Α . . AR DQ . πο την σχεση - = - και την ομοιοτητα των RC QB τριγώνων BOD και COA προκύπτει ότι τα τρίγωνα BOQ και COR είναι όμοια, άρα BQO = CRO , δη-λαδή τα σημεία Ρ, Q, R και Ο είναι ομοκυκλικά. Παρατήρηση :

Λύνεται άμεσα σαν εφαρμογή του γενικευμένου Θεωρήματος του Mac Laurin σε γωνία. Πρόβλη μα 6 Σε ένα διαγωνισμό μαθηματικών δόθηκαν 6 προβλήματα στους διαγωνιζόμενους. Κάθε ζευ­γάρι προβλημάτων λύθηκε από περισσότερους από τα 2/5 των διαγωνιζομένων. Κανένας από τους διαγωνιζόμενους δεν έλυσε και τα 6 προ­βλήματα. Να αποδείξετε ότι υπήρχαν τουλάχι­στον δύο διαγωνιζόμενοι οι οποίοι έλυσαν ακρι­βώς 5 προβλήματα ο καθένας. Λύση

Έστω ότι υπάρχαν η διαγωνιζόμενοι. Θα μετρή­σουμε το πλήθος Ν των διατεταγμένων ζευγών (C, Ρ) όπου Ρ είναι ένα ζεύγος προβλημάτων το οποίο λύθηκε από τον διαγωνιζόμενο C. Υπάρχουν Ι5

ζεύγη προβλημάτων, και για καθένα από αυτά, υ-. λ ' (2n + I) δ ζ ' παρχουν του αχιστον ιαγωνι ομενοι οι

5

οποίοι έλυσαν και τα δύο προβλήματα του ζεύ­γους. Συνεπώς:

2n + I Ν 2:: Ι5 · -- = 6n + 3 (1) 5

Υποθέτουμε τώρα ότι k διαγωνιζόμενοι έλυσαν 5

προβλήματα. Τότε καθένας από αυτούς έλυσε Ι Ο

ζεύγη προβλημάτων, ενώ καθένας από τους υπό-ΕΥΚΛΕΙΔΗΣ Β' λθ ' τ.2/21

Page 24: Ευκλειδης Β 58

Μαθηματικοί Διαγωνισμοί - Μαθηματικές Ολυμπιάδες

λοιπους η - k έλυσε το πολύ 6 ζευγάρια προβλη­μάτων. Συνεπώς:

Ν s I Ok + 6(n - k) = 6n + 4k (2) Από τις δύο παραπάνω σχέσεις συμπεραίνουμε ότι k � 1 . Αν ο αριθμός (2n + 1) δεν είναι ακέραιος, 5 τότε για κάθε ζεύγος προβλημάτων θα είχαμε του-λάχιc:iτον (2n + 1) (δηλαδή > (2n + 1) ) διαγωνιζό-5 5 μενους οι οποίοι έλυσαν και τα δύο προβλήματα

ζ , ( , (2n + 1) ) δ , του ευγους αντι για � σε ιαφορετικη 5 περίπτωση). Τότε από την ( 1 ) έχουμε ότι Ν � 6n + 6 και αυτό σημαίνει ότι k � 2 . Α ν τώρα υπήρχε κάποιος δια­γωνιζόμενος έλυσε λιγότερα από 4 προβλήματα, τότε θα είχε λύσει το πολύ 3 ζεύγη προβλημάτων (αντί για 6 όπως προηγουμένως) και η δεύτερη α­νισότητα γίνεται Ν s 6n + 4k - 3 , η οποία με την ανισότητα Ν � 6n + 3 δίνει επίσης k � 2 . Συνεπώς μένει μόνο η περίπτωση όπου ο αριθμός (2n + 1) , , , θ δ ,

�-� ειναι ακεραιος και κα ε ιαγωνιζομενος 5 έλυσε 4 ή 5 προβλήματα. Ας υποθέσουμε ότι k = 1 (εις άτοπο απαγωγή) και ας ονομάσουμε τον διαγωνιζόμενο που έλυσε 5 προβλήματα το <f'άιιηtjιeυμε Ν = 6n + 4 αφού ο νικητής έλυσε α­κριβώς 1 Ο ζεύγη προβλημάτων και οι υπόλοιποι διαγωνιζόμενοι έλυσαν ακριβώς 6 ζεύγη προβλη­μάτων ο καθένας. Ας ονομάσουμε ένα ζεύγος προβλημάτων «ειδικό»

, , (2n + 1) δ ζ ' αν περισσοτεροι απο ιαγωνι ομενοι έ-5 λυσαν και τα δύο προβλήματα του ζευγαριού. Α ν υπάρχουν περισσότερα από ένα ειδικά ζεύγη, τότε η ανισότητα ( 1 ) αντικαθίσταται από την

2n + 1 ( 2n + 1 ) Ν � 1 3 - -5- + 2 -5- + 1 = 6n + 5

το οποίο είναι αδύνατο. Όμοια, αν υπάρχει ένα ειδικό ζεύγος, τότε το πολύ (2n + 1) �-� διαγωνιζόμενοι μπορεί να · έλυσαν και τα 5 + 1

δύο προβλήματα του ζεύγους, γιατί διαφορετικά θα είχαμε

2n + 1 ( 2n + 1 ) Ν � 14 · -5- + -5- + 2 = 6n + 5

Ας μετρήσουμε το πλήθος Μ των ζευγών (C, Ρ) όπου το «δύσκολο» πρόβλημα (αυτό που δεν έλυ­σε ο νικητής) είναι ένα από τα προβλήματα του Ρ. Για καθένα από τα 5 ζεύγη που περιέχουν το δύ­σκολο πρόβλημα, υπάρχουν είτε

(2n + 1) ή (2n + 1) + 1 5 5 διαγωνιζόμενοι που έλυσαν και τα δύο προβλήμα­τα του ζευγαριού. Αυτό σημαίνει ότι Μ = 2n + 1 ή Μ = 2n + 2 , και το δεύτερο είναι δυνατό μόνο στην περίπτωση που υπάρχει ένα ειδικό ζεύγος, και επιπλέον αυτό το ειδικό ζεύγος περιέχει το δύσκολο πρόβλημα, τότε καθένας από αυτούς πρέπει να έλυσε 3 άλλα προ­βλήματα και συνεπώς 3 ζεύγη προβλημάτων που περιέχουν το δύσκολο πρόβλημα. Αυτό σημαίνει ότι Μ = 3m, και συνεπώς 2n + 1 = Ο ή 2 (mod3) . Έστω p ένα πρόβλημα διαφορετικό από το «δύ­σκολο», και ας μετρήσουμε το πλήθος L των ζευ­γών (C, Ρ) για τα οποία p ε Ρ . Μπορούμε πάντα να βρούμε ένα p για το οποίο το ειδικό ζεύγος προβλημάτων, αν υπάρχει, δεν περιέχει το p. Στην περίπτωση αυτή θα έχουμε L = 2n + 1 (σε καθένα από τα 5 ζεύγη προβλημάτων που περιέχουν το p

, β , (2n + 1) δ ζ , αντιστοιχουν ακρι ως ιαγωνι ομενοι οι 5 οποίοι έλυσαν και τα δύο προβλήματα του ζεύ­γους) . Τέλος έστω I το πλήθος των διαγωνιζομένων, ε­κτός του νικητή, που έλυσαν το πρόβλημα p. Τότε θα έχουμε ότι L = 3 1 + 4, αφού ο νικητής έ­λυσε το πρόβλημα p και άλλα 4 προβλήματα (άρα και 4 ζεύγη προβλημάτων που περιέχουν το p ), και οι υπόλοιποι I έλυσαν το p και άλλα 3 προβλήματα ο καθένας (άρα και 3 ζεύγη που περιέχουν το p ) . Αυτό σημαίνει ότι 2n + 1 = I (mod 3), το οποίο εί­ναι άτοπο, και το ζητούμενο έχει αποδειχθεί.

ΕΥΚΛΕΙΔΗΣ Β' λθ' τ.2/22

Page 25: Ευκλειδης Β 58

Μαθηματικοί Διαγωνισμοί - Μαθηματικές Ολυμπιάδες

66ος ΠΑΝΕΛΛΗΝΙΟΣ ΜΑΘΗΤΙΚΟΣ ΔΙΑΓΩΝΙΣΜΟΣ ΣΤΑ ΜΑΘΗΜΑΤΙΚΆ "0 ΘΑΛΗΣ"

ΣΑΒΒΑΤΟ, 12 ΝΟΕΜΒΡΙΟΥ 2005

Α ' ΛΥΚΕΙΟΥ 1 . Ν' αποδειχθεί ότι ο αριθμός

2003 . 20053 - 2004 . 20023 ' είναι κύβος ακεραίου αριθμού.

Λύση

Οι αριθμοί 2003 , 2005 διαφέρουν κατά δύο. Οι α­ριθμοί 2003 , 2004 διαφέρουν κατά Ι όπως επίσης και οι αριθμοί α( α + 2)3 - (α + Ι)(α - Ι)3 . Παρατη-ρούμε ότι μετά από πράξεις έχουμε: α( α + 2)3 - (α + 1)(α - 1)3 = (2α + Ι)3 . Άρα: 2003 · 20053 - 2004 · 20023 = (2 · 2003 + 1)3 που · εί­ναι κύβος ακεραίου αριθμού. 2. Ν' απλοποιηθεί η παράσταση

�13 + 30�2 + �9 + 4Ji . Λύση

Παρατηρούμε ότι: 9 + 4J2 = Ι + 8 + 2 · I(2J2) = = Ι2 + (2J2)2 + 2 · 1 (2J2) = (1 + 2J2)2 • Επομένως: A = �1 3 + 30�2 + �9 + 4J2 = �1 3 + 30�3 + 2J2 Με βάση την ίδια λογική έχουμε 3 + 2J2 = (J2)2 + 12 + 2 · J2 · 1 = (J2 + 1)2 • Άρα: Α = �1 3 + 30( J2+1) = � 43 + 30J2 = = �52 + (3J2)2 + 2 · 5 · 3J2 = �(5 + 3J2)2 . Τελικά Α = 5 + 3J2. 3. Να αναλυθεί το πολυώνυμο

x s + χ3 + Ι = x s + χ4 + χ3 _ χ4 _ χ3 + χ + Ι =

= χ 3 (χ2 + χ + Ι) - χ3 (χ + Ι) + χ + Ι = = χ3 (χ2 + χ + Ι) - (χ + Ι)(χ3 - Ι) = = χ 3 (χ 2 + χ + Ι) - (χ + Ι )(χ - Ι)( χ 2 + χ + Ι) = = (χ2 + χ + Ι)[χ 3 - (χ + Ι)(χ - 1)] = = (χ2 + χ + Ι)(χ3 - χ2 + Ι) . Τελικά έχουμε: χ6 - 2χ5 +χ2 - χ - 2=(χ - 2)(χ2 +χ+ Ι)(χ3 - χ2 + Ι) . 4. Ν' αποδειχθεί ότι αν η ευθεία που ενώνει τα

μέσα των δύο απέναντι πλευρών ενός κυρ­τού τετραπλεύρου διαιρεί το τετράπλευρο σε δύο ισεμβαδικά τετράπλευρα, τότε το τετράπλευρο είναι τραπέζιο.

Λύση

Έστω Μ μέσο της πλευράς ΑΒ και Ν μέσο της πλευράς Γ Δ. Έχουμε σαν δεδομένο ότι: (ΑΜΝΔ) = (ΒΓΝΜ) =>

(ΝΜΑ) + (ΑΝΔ) = (ΝΜΒ) + (ΒΝΓ) ( Ι ) Μ

Παρατηρούμε ότι: (ΝΜΑ) = (ΝΜΒ) (ίσες βάσεις κοινό ύψος) . (1) => (ΑΝΔ) = (ΒΝΓ) =>

ΑΑ' · ΔΝ = ΒΒ' · ΝΓ με ΔΝ = ΝΓ. 2 2 χ6 _ 2xs + χ2 _ χ _ 2 Τελικά ΑΑ' = ΒΒ' δηλαδή ΑΒ I/ ΔΓ οπότε ΑΒΓ Δ

σε γινόμενο τριών πολυωνύμων θετικού τραπέζιο. βαθμού.

Λύση

χ6 - 2χ5 + χ 2 - χ - 2 = χ5 (χ - 2) + χ 2 - 2χ + χ - 2 = = χ5 (χ - 2) + χ(χ - 2) + (χ - 2) = (χ - 2)(χ 5 + χ + Ι). Μένει να αναλύσουμε το χ 5 + χ3 + 1 σε γινόμενο δύο πολυωνύμων.

Β ' ΛΥΚΕΙΟΥ 1 . Αν α και β είναι πραγματικοί αριθμοί τέ-

τοιοι ώστε να ισχύει

(α + .Jα2 + 1 )(β + �β2 + 1) = 1 ,

ν' αποδειχθεί ότι α + β = Ο.

ΕΥΚΛΕΙΔΗΣ Β' λθ' τ.2/23

Page 26: Ευκλειδης Β 58

Μαθηματικοί Διαγωνισμοί - Μαθηματικές Ολυμπιάδες

Λύση

l0ς τpόπος: Αφού (α + .Jα2 + 1 ) · (β + �β2 + 1 ) = 1 , θα υπάρχει γωνία φ ώστε εφφ = α + .Jα2 + 1 και σφφ = β + �β2 + 1 με εφφ :;t: Ο και σφφ :;e Ο . εεφ = α + .J α 2 + 1 =:::;. εφφ - α = .J α 2 + 1 =:::;. εφ2φ - 2αεφφ + α2 = α2 + 1 ή εφ2φ - 2αεφφ = 1 ( 1 ). Όμοια έχουμε:

σφ2φ - 2βσφφ = 1 =:::;. 1 - 2βεφφ = εφ2φ (2) Προσθέτουμε τις ( 1 ), (2) και παίρνουμε;

-2(α + β) · εφφ = 0 =:::;. α + β = Ο . 2"ς τρόπος: (α + .Jα2 + 1 )(β + �β2 + 1 ) = 1 πολλαπλασιάζουμε με την συζυγή της .Jα2 + 1 + α και παίρνουμε: β + �β2 + 1 = .Jα2 + 1 - α =:::;. β + α = .Jα2 + 1 - �β2 + 1 ( 1 ) . Αν πολλαπλασιά­ζουμε την αρχική επί την συζυγή της �β2 + 1 + β παίρνουμε α + .Jα2 + 1 = �β2 + 1 - β οπότε

α + β = �β2 + 1 - .Jα2 + 1 Με πρόσθεση της ( 1 ), (2) κατά μέλη έχουμε:

2( α + β) = Ο =:::;. α + β = Ο .

(2)

2. Έστω ένα τρίγωνο ΑΒΓ με ΑΒ = ΑΓ. Ονο­μάζουμε Μ το μέσο της ΒΓ και Δ το σημείο τομής της πλευράς ΑΓ με την κάθετη από το σημείο Μ στην πλευρά ΑΓ. Έστω Ε το σημείο τομής της ΒΔ με την ΑΜ. Ποιο από

ΑΜ .l ΒΓ από το ισοσκελές τρίγωνο ΑΒΓ με Μ μέσο της ΒΓ. Άρα: ΑΕΔ = ΒΕΜ = 90" - ΕΒΜ ( 1 ) ΑΔΕ = ΕΒΜ + r = ΕΒΜ + Β (2). (Ι) - (2) =:::;. ΑΕΔ - ΑΔΕ = 90" + Β - 2ΕΒΜ =:::;. =:::;. ΑΕΔ - ΑΔΕ = (90" - ΕΒΜ) + (Β - ΕΒΜ) > ο. Επομένως: ΑΕΔ > ΑΔΕ =:::;. ΑΔ > ΑΕ. 3. Έστω ΑΒΓ Δ τετράπλευρο εγγεγραμμένο σε

κύκλο κέντρου Ο το οποίο είναι εσωτερικό σημείο του τετραπλεύρου και Κ, Λ, Μ, Ν τα μέσα των πλευρών ΑΒ, ΒΓ, Γ Δ, ΔΑ α­ντίστοιχα. Να αποδειχθεί ότι οι διχοτόμοι των γωνιών ΚΟΜ και ΛΟΝ είναι κάθετες μεταξύ τους.

Λύση

Έστω Οχ, Oy οι διχοτόμοι των γωνιών ΚΟΜ , ΛΟΝ αντίστοιχα. Τότε χόy = χόΜ + Μόy = χόΜ + Νόy - ΜΟΝ =

= κόΜ + ΛόΝ - ΜόΝ = κόΝ + ΛόΜ = 2 2 2 = Β + Δ = 90". 2

τα ευθύγραμμα τμήματα ΑΕ και ΑΔ είναι 4. Να εξετασθεί αν υπάρχουν πραγματικοί α-μεγαλύτερο;

Λύση Α

Ε_.. - - --;>' - - - - - I .,.. '

B �-=-�- -----��-�- ----� r

Μ

ριθμοί χ τέτοιοι ώστε να ισχύει (1 + χ3 )4 + (1 + χ2 )4 = 2χ4 .

Λύση

Προφανώς για χ = Ο έχουμε 2 = Ο, αδύνατη, άρα χ * Ο . Από ( 1 ) έχουμε

(l + x2 )4 ::;; 2x4 ή (2 i x i )4 :::;; 2χ4 ή 1 6χ4 :::;; 2χ4 ή 8 :::;; 1 άτοπο.

Άρα δεν υπάρχουν χ ε IR. : (1 + χ3 )4 + (Ι + χ 2 )4 = 2χ4 .

ΕΥΚΛΕΙΔΗΣ Β ' λθ ' τ.2/24

Page 27: Ευκλειδης Β 58

Μαθηματικοί Διαγωνισμοί - Μαθηματικές Ολυμπιάδες

Γ ΛΥΚΕ Ι Ο Υ

I . Έστω k μη μηδενικός πραγματικός αριθμός και (αν) μια ακολουθία θετικών αριθμών τέ­τοιων ώστε να ισχύει (αν+1 )2 + αν+ \ = k((α. )2 + α. ) + (k - l)αν+ Ιαν για κάθε ν θετικό ακέραιο. Ν' αποδειχτεί ότι η ακολουθία (αv) είναι γεω­μετρική πρόοδος.

Λύση

Η συνθήκη γράφεται: (αν+ ) - κ . αν )(αν+ Ι + αν + Ι) = ο .

Όμως η αv είναι ακολουθία θετικών αριθμών οπό­τε:

Επομένως:

Αυτό σημαίνει ότι η αν είναι γεωμετρική πρόοδος.

Λύση

χ ψ

Αφού Ρ σταθερό εσωτερικό σημείο της γωνίας χΟψ σημαίνει ότι η ευθεία ΟΡ είναι μοναδική . Άρα και οι γωνίες φ = ΑΟΡ και ω = ΡΟΒ . Όμως: ΡΑΒ = ω, ΡΒΑ = φ,ΑΡΒ = 180° - χόψ. Το τρίγωνο λοιπόν ΡΑΒ διατηρεί τις γωνίες του. Αυτό σημαί-

, ΡΑ ( θ . θ ' θ ' ) νει οτι: - = t 1 t 1 στα ερος ετικος αρι μος και ΑΒ ΡΒ = t, ( t, σταθερός θετικός αριθμός) . ΑΒ - -

Ε , ΡΑ + ΡΒ , πομενως: = t 1 + t , η ΑΒ -2 . Για ακεραίους rn και η, ν' αποδειχθεί ότι αν ΑΒ

ο αριθμός rn2 + 28rnn + n2 διαιρείται δια ΡΑ + ΡΒ t 1 + t2 του 13, τότε και ο αριθμός rn3 + n3 διαιρεί-ται δια του 13.

Λύση

Ο 1 3 διαιρεί την m 2 + 28mη + η 2 = ( m + η )2 + 26m · η

οπότε: I 3 l (m + η)2 � 1 3 Im + η

αφού ο 1 3 είναι πρώτος. Άρα

3 . Έστω ΧόΥ μια κυρτή γωνία, Ρ εσωτερικό σημείο της και C ο κύκλος που διέρχεται από τα σημεία Ο, Ρ και τέμνει τις ΟΧ, ΟΥ, αντίστοιχα στα σημεία Α και Β διαφορετι­κά από το Ο. Ν' αποδειχθεί ότι ο λόγος

ΑΒ ΡΑ + ΡΒ

είναι σταθερός για οποιαδήποτε θέση του κύκλου C.

4 . Για πραγματικούς αριθμούς α, β, γ, χ τέ­τοιους ώστε α < β, γ < β, χ > (α + γ) / 2 ν'

αποδειχθεί ότι

Λύση

χ - γ χ - α β - χ 3 -- + -- + � - . β - α β - γ 2χ - α - γ 2

Θέτουμε u = β - α, ν = β - γ, w = 2χ - α - γ , οπότε u > Ο, ν > Ο, w > Ο. Θεωρούμε:

(ν - u + w) (u - ν + w) χ - γ = 2 ' χ - α = 2 ' και

β (u + ν - w) , ' δ ξ , - χ = και η υπο απο ει η ανισοτητα 2

, (ν - u + w) (u - v + w) (u + v - w) 3 γινεται: . + + � u ν w

ή ισοδύναμα, (� +�) + (: + : ) + (: + : ) � 6

που εύκολα προκύπτει από την πολύ γνωστή ανι­σότητα: � + � � 2 όταν α · β > Ο. β α

ΕΥΚΛΕΙΔΗΣ Β ' λθ ' τ.2/25

Page 28: Ευκλειδης Β 58

� ......... . ,_ �ιι' �� ��Ιι �- Αιι8-Ι•

jιλ γεβpα

Λ Υ Μ Ε Ν ΕΣ ΑΣΚΗΣΕ Ι Σ

1 ' ' f( ) χ2 + 23 . Δινεται η συναρτηση χ = 2 •

χ - 4χ + 3 i . Βρείτε το πεδίο ορισμού της. ii. Βρείτε τα σημεία τομής της γραφικής

παράστασης της f με την ευθεία y = 3.

Λύση

1 . Θα πρέπει χ2 - 4χ + 3 =t= Ο . Λύνουμε την εξί­σωση χ2 - 4χ + 3 = Ο για να βρούμε ποια είναι τα χ Ε IR τα οποία μηδενίζουν την παράσταση χ2 - 4χ + 3 . Δ = β2 - 4αγ = 1 6 - 1 2 = 4 . Άρα χ = -β + ΓΔ = 3 2α

-β - ΓΔ χ = = 1 2α Άρα χ2 - 4χ + 3 = 0 και μόνο αν χ = 1 ή χ = 3 . Επομένως χ2 - 4χ + 3 =t= Ο αν και μόνο χ =t= l και χ =t= 3 . Έτσι το πεδίο ορισμού της f είναι το IR - {1, 3} .

i i . Για να βρούμε τα σημεία τομής της γραφικής παράστασης της f με την ευθεία y = 3 λύνουμε την εξίσωση

χ2 + 23 2 = 3 με χ =t= 1 και χ =t= 3 . χ - 4χ + 3

'Ε χ2 + 23 3 τσι = � χ2 - 4χ + 3 � χ2 + 23 = 3 ( χ2 - 4χ + 3 ) � χ2 - 6χ - 7 = ο Δ = ( -6)2 - 4 · I · (7) = 64 . Ά 6 + 8 1 4 7 δ . ρα χ 1 = -2

- = 2 = εκτη,

6 - 8 -2 χ2 = -- = - = -1 δεκτή . 2 2

Σπύρος Τζινιέρης

Έτσι τα σημεία τομής της γραφικής παράστασης της f με την ευθεία y = 3 είναι τα Α(-1 ,3) και Β(7,3) .

2. Αν το σημείο Α(λ-2,μ+1) με λ,μ Ε IR ανήκει στην ευθεία (ε1) με εξίσωση την -2χ + y - 3 = Ο και οι ευθείες (ε2) και (ε3) με εξισώσεις αντίστοιχα τις y = (μ + 2)χ - 3 και y = ( λ2 - λ - μ + 4) χ - 1 είναι παράλλη-

λες μεταξύ τους βρείτε τους λ και μ. Λύση

Ένα σημείο ανήκει σε μια ευθεία όταν οι συντε­ταγμένες του επαληθεύουν την εξίσωση της. Αφού λοιπόν το Α(λ-2, μ+ I ) ανήκει στην (ε 1 ) θα ισχύει -2(λ - 2)+μ+1 - 3 = ο � -2λ + 4 + μ + 1 - 3 = 0 � � μ - 2λ + 2 = 0 � μ = 2λ - 2 ( 1 ) Επειδή (ε2)//(ε3) θ α ισχύει μ + 2 = λ2 - λ - μ + 4 � λ2 - λ - 2μ + 2 = 0 (2) Η (2) λόγω της ( I ) γίνεται: λ2 - λ - 2(2λ - 2) + 2 = 0 � � λ2 - 5λ + 6 = Ο η οποία δίνει λ =2 ή λ = 3 . Για λ =2 έχουμε μ = 2 . Για λ = 3 έχουμε μ = 4. 3. Θεωρούμε την παράσταση

Α( χ) = χ2 + D)X + .e

με (m + .€ + 2)2 > (.€ + 2)2 + 2 [m(i + 2) + 2i] όπου m, .e Ε IR . i. Δείξετε ότι η Α(χ) έχει δύο ρίζες πραγ­

ματικές και άνισες. ί ί . Αν ρι , ρ2 είναι ρίζες · της Α(χ) με

ΕΥΚΛΕΙΔΗΣ Β' λθ ' τ.2/26

Page 29: Ευκλειδης Β 58

Μαθηματικά για την Α ' Λυκείου

Ι ρ . + • Ι :s; 2 και lρz - 21 :s; ι δείξτε ότι ��A(x)

l - 3 :s; lxl για κάθε χ Ε JR .

Λύση

Έχουμε από την υπόθεση ότι (m + R + 2)2 > (R + 2)2 + 2 [ m(R + 2) + 2R ] <=> <=> m2 + R2 + 4 + 2mR + 4m + 4R > > R2 + 4R + 4 + 2mR + 4m + 4R <=> m2 > 4R ( 1 ) 0 ί . Για να δείξουμε ότι η Α(χ) έχει δύο ρίζες πραγ­

ματικές και άνισες αρκεί να δείξουμε ότι Δ = β 2 - 4αγ > Ο ο

Έχουμε Δ = m2 - 4R > 0 λόγω της ( 1 )ο ίί . Αφού το τριώνυμο Α(χ) έχει ρίζες ρ ι και ρ2 θα

ισχύουν ρ ι + ρz = - m ρι ρ2 = R

(2) και (3)

Από την υπόθεση έχουμε: l ρ ι + 1 1 :s; 2 τότε -2 :s; ρι + 1 :s; 2 ή -3 :s; ρ1 :::; 1 ή -3 :s; ρ1 :::; 1 < 3 Έτσι

lριl:::; 3 (4)

Επίσης έχουμε: l ρ2 - 2 1 :s; 1 ο Τότε - 1 :::; ρ2 - 2 :::; 1 ή 1 :::; ρ2 :::; 3 ή -3 < 1 :::; ρ2 :::; 3 Έτσι lρ2

1:::; 3 (5)

Γνωρίζουμε ότι lρ 1 + ρ21 :::; lρ 1 I + lρ2

1 τότε από τις (2), (4) και (5) προκύπτει l-mi :S; 3 + 3 ή lml:::; 6 (6)

Επίσης lρ 1 ο ρ21 = lρ 1

lol ρ21 τότε από τις (3), ( 4),

(5) προκύπτει I R I :s; 3 ο 3 ή I R I :::; 9 (7) 'Ετσι από IA(x) l = l x2 + mx + RI παίρνουμε IA(x)

Ι:::;Ix21 +Ιm o x

i+IRI τότε

IA(x)

l:::;lxl 2 + lm

l lxi + IR

I οπότε από τις (6) και (7) προκύπτει IA(x)

l:::; lxl2 + 6 lx

l + 9 οπότε IA(x)

l:::; ( lxl + 3 )2 απ' όπου προκύπτει

��A(x)

l:::; lxl + 3 και τελικά ��A(x)l - 3 :S; lx l

για κάθε χ Ε JR ο

4. Δίνεται η εξίσωση Χz - (2λ + μ)χ + λ + μ = 0 (1) με λ,μ Ε JR

καθώς και η συνάρτηση f(x) = χ + 1 . Αν ρι χ + 2

και ρz με ρ1 , ρz Ε JR - {-2} είναι οι ρίζες της

εξίσωσης (1) και ισχύουν f(ρ1 ) + f(ρz ) = 17 12

και f(ρ1 ) • f(ρz ) = .!_ , να βρεθούν: 2 ί. τα λ, μ Ε JR ίί. οι ρίζες της εξίσωσης (1 ).

Λύση

Έχουμε ότι οι ρ ι και ρ2 είναι ρίζες της (l)o Άρα θα ισχύουν ρ1 + ρ2 = 2λ + μ , ρ 1 ο ρ2 = λ + μ και Δ = (2λ + μ)2 - 4(λ + μ) > Ο ο

Επίσης έχουμε f(ρ1 ) + f(ρ2 ) = .!2 <=> 1 2 ρ1 + 1 ρ2 + 1 1 7 <=> -- + -- = - <=> ρ1 + 2 ρ2 + 2 1 2

<=> 1 2 ο [(ρ. + 1)(ρ2 + 2) + (ρ2 + 1)(ρ ι + 2)] = = 1 7(ρ1 + 2)(ρ2 + 2) <=>

<=> 7ριρ2 + 2(ρι + ρ2 ) = 20 <=> <=> 7(λ + μ) + 2(2λ + μ) = 20 <=> <=> 7λ + 7μ + 4λ + 2μ = 20 <=> 1 1λ + 9μ = 20 (2) Επιπλέον ισχύει f( )f( ) _

1 ρ1 + 1 ρ2 + 1 _ 1 ρ ι ρ2 - - <=> -- ο -- - - <=> 2 ρ1 + 2 ρ2 + 2 2

<=> 2(ρ ιρ2 + ρι + ρ2 + 1) = ρι ρ2 + 2ρι + 2ρz + 4 <=> <=> ριρ2 = 2 <=> λ + μ = 2 (3 ) Λύνουμε τώρα το σύστημα

{1 1λ + 9μ = 20 <=>

{1 1λ + 9μ = 20 <=> {μ = 1 λ + μ = 2 λ = 2 - μ λ = 1

Για μ = 1 και λ = 1 η ( 1 ) γράφεται χ 2 - 3χ + 2 = 0 η οποία έχει Δ = (-3)2 - 4 · 1 ° 2 = 9 - 8 = 1 και

2 3 ±

JΪ 3 ± 1 / ρ ι = 2 = 1

ρι ο2 = -2- = -2- "- - 4 -ρ2 - - - 2 2 5. Δίνεται η εξίσωση

ΧΖ - 2λχ + λΖ - 1 = 0. α. Να δείξετε ότι έχει δύο ρίζες άνισες. β. Να βρεθεί ο λ Ε JR ώστε οι δυο ρίζες να

είναι θετικές. γ. Αν χ1, χ2 είναι οι δύο θετικές ρίζες της να

βρεθεί η εξίσωση με ρίζες Fι, .j;;. ΕΥΚΛΕΙΔΗΣ Β ' λθ ' τ.2/27

Page 30: Ευκλειδης Β 58

Μαθηματικά για την Α ' Λυκείου

ΛίJση α. Δ = 4λ2 - 4(λ2 - 1) = 4λ2 - 4λ2 + 4 = 4 > 0 p. Για να έχει δύο ρίζες θετικές πρέπει S > Ο και Ρ > Ο τότε { 2�:

ιο �{ λ:σ.

ιο �{ λ > Ο

και � λ > 1 λ2 - 1 > ο iλl > 1 λ > 1 ή λ < -1

γ . Αφού χ 1 , χ 2 είναι οι θετικές ρίζες θα ισχύει λ > 1 . 'Ετσι <Fι

+Fz

)2 = (Fι

)2 + 2Fι Fz + (,Fz)2 =

Χ 1 + 2Μ+χ2=2λ+2

� = 2(λ +

.Jλ2 - 1 )

Τότε Fι

+Fz

= �2(λ+

.Jλ2 - 1 ) και

Fι.Fz=M

=�

Η ζητούμενη εξίσωση δίνεται από τον τύπο χ2 - Sx + Ρ = Ο ή χ2 -

�2(λ+

�)χ +

.Jλ2 - 1 = 0 .

6. Να λυθούν οι εξισώσεις: α. Ιχ2 + χ Ι + Ιχ2 -

2χ -

31 = ο β. (t - lx

l)2 = 2

Λύση α. Η εξίσωση lx 2 + x

l + lx 2 - 2χ - 3

1 = Ο � {l x2 + xi = O {χ2 + χ = Ο � και � και �

Ιχ2 - 2χ -31 = ο χ2 - 2χ - 3 = ο {χ = 0 ή χ = -1 � και � χ = -1

χ = -1 ή χ = 3 β. Η εξίσωση {1 - lx l = +fi {l x l = 1 -fi

(1 - lxl)2 = 2� ή � ή

1 - lxl= -fi l

xl = 1 +

fi

Επειδή 1 -J2 < Ο . Η εξίσωση ( 1 ) είναι αδύνατη .

(1)

(2)

Έτσι λύσεις της δοθείσας προκύπτουν από την (2) και είναι χ = ±(1 + J2) .

7. Αν η εξίσωση χ2 - αχ + β = Ο έχει πραγμα­

τικές ρίζες χ1, χ2 να βρεθεί η εξίσωση με ρί­ζες ρ1

= I χ. Ι και ρ2 = lx2 1 , όπου α, β πραγ-

ματικοί αριθμοί και β < Ο. Λύση

Από τους τύπους Vίeta έχω χ 1 + χ2 = α και χ 1 • χ 2 = β τ

ότε

( lx ιl + l x 2

1 )2 = lx ιl 2 + 2 lx ι

l lx 21 +1x 21 2 =

= Χ � + 2lx 1 x 2

1 + x; = (χ 1 + χ2 )2 - 2Χ 1 Χ 2 + 2lx 1 x 2

1 = = α2 - 2 · β + 2

1βi = α2 - 2β - 2β = α2 - 4β ( 1 )

αφού η εξίσωση έχει 2 ρίζες πραγματικές θα ισχύει Δ > Ο δηλαδή α2 - 4β > Ο . Τότε η ( 1 ) γίνεται: lx ιl + lx2

1 = ±�α2 - 4β και επειδή

lx ιl + lx 21 > Ο θα

είναι lx ιl + lx2

1 = �α2 - 4β . Άρα

ρ1 + ρ2 =�α2 - 4β και

ρι ρ2 = lx ιl lx 21 = Ιχ ι χ2

Ι = iβ

i = -β ·

Η ζητούμενη εξίσωση δίνεται από την χ2 - Sx + Ρ = Ο δηλαδή είναι η

χ2 -�α2 - 4β · χ - β = Ο .

8. Δίνεται η εξίσωση - χ2 - 2αχ + α2 + β2 + 2 = Ο , α, β Ε � .

ί . Να δείξετε ότι η εξίσωση έχει ρίζες χ

ι , χ2 πραγματικές και άνισες για κάθε α,β Ε � .

ί ί . Να βρεθεί η εξίσωση 2ου βαθμού, η ο­ποία έχει ρίζες διπλάσιες της αρχικής.

ί ί ί . Να βρεθούν οι α, β Ε � ώστε η εξίσωση

να έχει ρίζα τον αριθμό 1 . Λύση

ί. Η δοθείσα εξίσωση έχει διακρίνουσα Δ = (-2α)2 - 4(- 1)(α2 + β2 + 2) = = 4α 2 + 4α 2 + 4β 2 + 8 = 8α 2 + 4β 2 + 8 > Ο , για κάθε α, β Ε � . Άρα έχει ρίζες πραγματικές και άνισες.

i i . Αφού χ1 , χ2 είναι -χ2 - 2αχ + α2 + β2 + 2 = 0 Χ ι + χ2 = -2α ( 1 )

οι ρίζες της θα ισχύουν

α2 + β2 + 2 2 2 και χ · χ = = -α - β - 2 (2) I 2 _ 1 Η ζητούμενη εξίσωση θα έχει ρίζες ρ1 = 2χι , ρ2 = 2χ2 και δίνονται από τη χ2 - S · χ + Ρ = Ο

( I ) Με S = ρι + ρ2 = 2χ ι + 2χ 2 = 2(χ ι + χ2 ) =- 4α

ΕΥΚΛΕΙΔΗΣ Β ' λθ ' τ.2/28

Page 31: Ευκλειδης Β 58

Μαθηματικά για την Α ' Λυκείου

( 2 ) και Ρ = ρ, · ρ2 = 2χ 1 • 2χ 2 = 4χ 1 • Χ 2 = = 4(-α2 - β2 - 2) = -4α2 - 4β2 - 8 . Οπότε προκύπτει η εξίσωση χ 2 + 4αχ - 4α 2 - 4β 2 - 8 = Ο .

i i i . Για να έχει η αρχική εξίσωση ρίζα ρ = 1 πρέπει - 12 - 2α · 1 + α2 + β2 + 2 = Ο <=> α2 - 2α + 1 + β2 = Ο <=> (α - 1)2 + β2 = Ο <=> (α - 1)2 = 0 και β2 = 0 <=> α = 1 και β = Ο.

9. Δίνονται οι εξισώσεις χ2 + λχ - 5 = Ο (1) και 2χ2 - 4χ - 2λ + 2 = 0 (2), λ Ε � . i . Να δείξετε ότι η εξίσωση (1) tχει ρίζες πραγ­

ματικές και άνισες χ,, χ2 για κάθε λ Ε � . ί ί . Να βρεθεί ο λ Ε � ώστε η εξίσωση (2) να

έχει ρίζες ρ,, ρ2 πραγματικές και άνισες. i i i . Να βρεθεί ο λ Ε � ώστε μια ρίζα της ε­

ξίσωσης (1) να είναι αντίθετη μιας ρίζας της εξίσωσης (2).

Λύση

ί . Δ, = λ2 - 4 · 1(-5) = λ2 + 20 > 0 για κάθε λ ε � . Άρα η εξίσωση ( 1 ) έχει ρίζες πραγματικές και άνισες.

ίί. Πρέπει Δ2 > Ο <=> (-4)2 - 4 · 2 · (-2λ + 2) > Ο <=> 1 6 - 8( -2λ + 2) > ο <=> 1 6 + 1 6λ - 1 6 > ο <=> 1 6λ > ο <=> λ > ο

ί ί ί . Αφού Χ ι είναι ρίζα της εξίσωσης ( 1 ) και ρ 1 ρίζα της εξίσωσης (2) και χ , = -ρ, προκύπτει τό σύστημα {:� : �:: - 5 = ο <=>

{:� : �;,1- 5 = ο

2ρ� - 4ρ, - 2λ + 2 = ο ρ� - 2ρ, - λ + 1 = ο {χ , = -ρ, {χ , = -ρ, (3)

ρ� + λρl - 5 = ο <=> ρ

� - (ρ, - 1 )2 ρ, - 5 = ο ( 4)

(ρ, - 1 )2 = λ λ = (ρ, - 1 )2 (5)

από την (4) έχουμε ρi - 3ρ

� + ρ, + 5 = Ο

<=> ρ� - 4ρ� + ρ� + 5ρ, - 4ρ, + 5 = ο

<=> ρ� (ρ1 + 1 ) - 4ρ1 (ρ, + 1 ) + 5 (ρ , + 1 ) = Ο

<=> (ρ, + 1 ) {ρ� - 4ρ, + 5 ) = 0 <=> ρ1 = - 1 αφού το

τριώνυμο έχει Δ < Ο. Έτσι από την (5) έχουμε λ = 4.

I Ο. Έμπορος αγόρασε κάποια μέτρα ενός υφά­σματος και πλήρωσε συνολικά 1 80 €. Αν με τα ίδια χρήματα αγόραζε 10 μέτρα περισ­σότερα, τότε κάθε μέτρο υφάσματος θα του κόστιζε 0,2 λιγότερα. Να βρεθούν: i . Πόσα μέτρα υφάσματος αγόρασε. i i . Πόσα ευρώ του κόστισε το κάθε μέτρο

υφάσματος. Λύση

ί. Έστω ότι αγόρασε χ μέτρα του υφάσματος. Τό-' θ , , 1 80 € Α , τε το κα ε μετρο του κοστισε - . ν αγο-

χ ραζε χ + 1 Ο μέτρα του υφάσματος, τότε το κάθε

μέτρο θα του κόστιζε 180 - Ο, 2 €. χ

Επομένως έχουμε:

(χ + 1 0) { 1:0 - 0, 2) = 1 80 <=>

1 800 )-86 - 0, 2x +-- 2 =J-86 <=> χ

-0, 2χ2 + 1 800 - 2χ = ο <=> 2χ 2 + 20χ - 1 8000 = Ο <=> χ 2 + 1 Οχ - 9000 = Ο που έχει Δ = 36 1 00,

- 1 0 ±J36ϊ00

χ\ ,2 = 2 - 1 0 ± 1 90

2 1 80 90 ' -200 1 00 ' χ , =Τ= η χ

2 = -

2- = - απορριπεται.

Άρα αγόρασε 90 μέτρα του υφάσματος. ί ί . Το κάθε μέτρο υφάσματος του κόστισε

1 80 = 2 €. 90

I I . Δίνονται δύο άνισοι κύκλοι με κοινό κέντρο ένα σημείο Ο. Α ν η ακτίνα του μεyάλου κύ­κλου είναι αυξημένη κατά 3 μέτρα του δι­πλάσιου της ακτίνας του μικρού κύκλου και το εμβαδόν του δακτυλίου που σχηματίζουν οι δύο ομόκεντροι κύκλοι ισούται με 24π τ.μ. να βρεθεί η ακτίνα του μικρού κύκλου.

Λύση

Α ν η ακτίνα του μικρού κύκλου είναι ρ > Ο τότε η ακτίνα του μεγάλου κύκλου θα είναι 2ρ + 3 .

ΕΥΚΛΕΙΔΗΣ Β ' λθ ' τ.2/29

Page 32: Ευκλειδης Β 58

Μαθηματικά για την Α ' Λυκείου

Για το εμβαδόν του δακτυλίου που σχηματίζουν οι δύο ομόκεντροι κύκλοι θα ισχύει: Εδακnιλίου = Εμεyάλου κύκλου - Εμικρού κύκλου �

� 24π = π(2ρ + 3)2 - πρ2 �

� 24π = π[(2ρ + 3)2 - ρ2 ] �

� ρ2 + 4ρ -5 = Ο η οποία έχει διακρίνουσα την Δ = 42 - 4 · ( -5) = 36 και επομένως έχει ρίζες τις

-4 ± .J36 ρι ,2 = 2 �

ή

-4 + 6 ρι = -- � ρι = 1 2 -4 - 6

· ρ2 = -- � ρ2 = -5 < Ο απορρίπτεται. 2

Έτσι θα ισχύει ότι ρ = 1 .

Σύνολα. fενι«ό nεpί συναρτήσεων. Η συνάρτηση f(x} = αχ + fJ

1 . Να γράψετε με αναγραφή των στοιχείων του το σύνολο : { I

ν2 + 2ν + 6 *} Α = χ ε Ν χ = ν , ν ε Ν •

Λύση

Έχουμε:

χ = ν2 + 2ν + 6 = ν + 2 + �

ν ν

Ε δ ' �τ , ' 6 �τ Ά ' πει η χ ε 1� , επεται οτι - ε 1� . ρα, ο ν ειναι ν

ένας θετικός διαιρέτης του 6. Συνεπώς: ν = I ή ν = 2 ή ν = 3 ή ν = 6. Έτσι, έχουμε αντίστοιχα: χ = 9 ή χ = 7 ή χ = 7 ή χ = 9. Άρα: Α = {7, 9 } . 2. Να δείξετε ότι είναι κενό το σύνολο:

Α = {χ ε ΊRi x4 + 2 lxl � ι χ - 21 - 3} .

Λύση

Έστω ότι Α "* 0 . Θεωρούμε ένα στοιχείο χ του Α. Τότε χ ε JR και

χ4 + 2 l x l � l x - 2 1 - 3 . ( 1 ) Επειδή :

l x - 2 1 - 3 � l x l + 2 - 3 = l x l - 1,

της Χρυσταλλένης Κυβερνήτου

από την ( 1 ) έπεται ότι: χ4 + 2 lx l � l x l - 1 :::::> χ4 + l x l + 1 � Ο , άτοπο.

Άρα: Α = 0.

3. Να δείξετε ότι Α ς Β , όπου: Α = {χ ε ΊR i x3 - 2χ2 + 2χ + 5 � ο} και

Β = {χ ε ΊRI χ3 - χ2 + 6 � ο} .

Λύση

Με χ ε JR , έχουμε: χ Ε Α :::::> χ3 - 2χ2 + 2χ + 5 � ο

:::::> χ3 - χ 2 + 6 � χ 2 - 2χ + 1 :::::> χ 3 - χ 2 + 6 � (χ - 1)2 � ο :::::> χ 3 - χ2 + 6 � ο =:> χ ε Β.

Άρα, κάθε στοιχείο του Α είναι και στοιχείο του Β . Συνεπώς Α ς Β. 4. Να βρείτε τα σύνολα ορισμού των συναρ­

τήσεων: 1 . f(x) = �3 - l2x - l l

2. g(x) = Jt - lxl . l + x

Λύση

Ι . Ένας αριθμός χ ε JR ανήκει στο σύνολο ορι­σμού της συνάρτησης f αν, και μόνο αν:

ΕΥΚΛΕΙΔΗΣ Β ' λθ ' τ.2/30

Page 33: Ευκλειδης Β 58

Μαθηματικά για την Α ' Λυκείου

3 -l2x - 1

1 � Ο � l2x - 1

1 � 3 � -3 � 2χ - 1 � 3 � -2 � 2χ � 4 � -1 � χ :::; 2 . Άρα, το σύνολο ορισμού Α της f είναι:

Α = [-1 , 2] . 2. Ένας αριθμός χ Ε � ανήκει στο σύνολο ορι-σμού της συνάρτησης g αν και μόνο αν: {:: xl x:;t.l Ο � {x :;t. - 1 ( 1 )

-� 0 <lxl - 1)(x + 1) � 0

1 + χ • Έστω ότι χ � Ο . Τότε, επειδή χ =F -1 ,

lxl = χ και

χ + 1 > Ο , έχουμε: (l) � x - 1 � 0 � x � 1 .

Άρα, τότε: χ Ε [0, 1] . • Έστω ότι χ < Ο. Τότε:

(l) <c> {�-:�1l)(x + I ) � O � {�χ*+�;, � ο � χ * - 1 .

Άρα, τότε: χ Ε (-οο, - 1) υ (-1 , 0). Συνεπώς, το σύνολο ορισμού Β της g είναι: Β=( -οο, - 1) υ ( -1, 0) υ [Ο, 1]=( -οο, - 1) υ ( -1, 1] .

5. Έστω η συνάρτηση :

f(x) = {3χ -

2, -2χ + 1,

Να λυθεί η εξίσωση :

αν χ � 1 αν χ < ι

3 lf(x)l - 4χ + ι = ο (1)

Λύση

1 . Έστω ότι χ � 1 . Τότε f(x) = 3χ - 2 και επειδή 3χ - 2 � 3 - 2 = 1 > Ο, έχουμε: (1) � 3 l3x - 2

l - 4x + 1 = 0 �3(3χ - 2) - 4χ + Ι = 0� 9χ -6- 4χ + 1 = 0 �χ= 1, δεκτή (γιατί χ � 1 ).

2 . Έστω ότι χ < 1 . Τότε f(x) = -2x + 1 . Έτσι έχουμε: (1) � 3 l-2x + 1

1 - 4χ + 1 = Ο �

� 3l2x - 1

1 - 4χ + 1 = Ο (2)

• 'Εστω ότι _!_ � χ < 1 . Τότε 2χ - 1 � Ο και άρα 2 l2x - 1

1 = 2χ - 1 . Έτσι, έχουμε: (2) � 3(2χ - 1) - 4χ + 1 = ο� χ = 1 '

απορρίπεται (γιατί χ < 1 ) . 'Ε ' 1 Τ ' • στω οτι χ < - . οτε

2 2χ - 1 < Ο και άρα

l2x - 11 = -2χ + 1 . Έτσι, έχουμε:

2 (2) � 3(-2χ + 1) - 4χ + 1 = 0 � χ = - , 5 δεκτή (γιατί χ <

_!_).

2 Άρα, οι ρίζες της εξίσωσης ( 1 ) είναι: χ = 1 και

2 Χ = - . 5 6. Να δείξετε ότι τα σημεία Α(2, -3), Β(6, -ι),

Γ( 4, 3) και Δ(Ο, ι) είναι κορυφές τετραγώ­νου.

Λύση

Έχουμε:

(ΑΒ) = �(6 - 2)2 + (- Ι + 3)2 = .J1 6 + 4 = J20. Όμοια βρίσκουμε ότι:

(ΒΓ) = (ΓΔ) = (ΔΑ) = J20. Άρα, το τετράπλευρο ΑΒΓΔ είναι ρόμβος. Εξάλ­λου, έχουμε:

(ΑΒ)2 = 20, (ΒΓ)2 = 20 και (ΑΓ)2 = (4 - 2)2 + (3 + 3)2 = 40.

Άρα: (ΑΒ)2 + (ΒΓ)2 = (ΑΓ)2 • Συνεπώς το τρίγωνο ΑΒΓ είναι ορθογώνιο στο Β. Άρα το τετράπλευρο ΑΒΓ Δ είναι τετράγωνο.

7. Να βρείτε τους αριθμούς λ, για τους οποί­ους η απόσταση των σημείων Α(λ, -3) και Β(6, λ - ι) είναι 4J2 .

Λύση

Έχουμε: (ΑΒ) = 4

.fi � (ΑΒ)2 = 32

� (λ - 6)2 + ( -3 - λ + 1)2 = 32 � (λ - 6)2 + (-λ - 2)2 = 32 � λ2 - 1 2λ + 36 + λ2 + 4λ + 4 = 32 � 2λ2 - 8λ + 8 = ο � λ2 - 4λ + 4 = ο

� (λ - 2)2 = 0 � λ - 2 = 0 � λ = 2.

8. Να βρείτε τα σημεία Μ τον άξονα χ 'χ, για τα οποία το άθροισμα των τετραγώνων των

ΕΥΚΛΕΙΔΗΣ Β' λθ ' τ.2/3 1

Page 34: Ευκλειδης Β 58

Μαθηματικά για την Α ' Λυκείου

αποστάσεων του Μ από τα σημεία Α(1 , 3) • f(2) = 23 - 2 · 2 + 2 = 6 -::f:. 5 . Άρα, η Cr δεν και Β(-1 , 1) είναι το ελάχιστο δυνατό.

ΛίJση

Έστω ότι Μ( χ, 0). Έχουμε: (ΜΑ/ + (ΜΒ)2 = = (χ - 1)2 + (0 - 3)2 + (χ + 1)2 + (0 - 1)2

= 2χ2 + 1 2 2 1 2, με το = για χ = Ο. Άρα, το ζητούμενο σημείο είναι το Μ(Ο, 0), δηλα­δή η αρχή των αξόνων. 9. Ένα σημείο Μ(χ, y) κινείται στο ορθογώνιο

ΑΒΓΔ (παρακάτω σχήμα). Να δείξετε ότι:

Υ.

Δ 3

lx - y

l� 4 .

Α�, ��--------�

Έχουμε:

- 1 ο

ΛίJση

{-1 � χ � 4 {- 1 � χ � 4 1 � y � 3 � -3 � -y � - 1

� -4 � x - y :::;; 3 :::;; 4 � -4 :::;; x - y :::;; 4

�ιx - y

l :::;; 4 .

1 Ο. Η γραφική παράσταση Cr της συνάρτησης f(x) = χ3 + αχ + β διέρχεται από τα σημεία Α(1 , 1) και Β(-1 , 3). Να εξετάσετε αν η Cr διέρχεται και από τα σημεία Γ(2, 5) και Δ(-2, -2).

Η Cr διέρχεται από τα σημεία Α( 1 , 1 ) και Β( -1 , 3) αν, και μόνο αν:

{f(l) = 1 { 1 + α + β = 1 {α + β = Ο f(- 1) = 3 <:::::> - 1 - α + β = 3 <:::::> α - β = -4

<=> {α = -2 . β = 2 Άρα: f(x) = χ3 - 2χ + 2 . Έχουμε:

διέρχεται από το σημείο Γ(2, 5) . • f(-2) = (-2)3 - 2(-2) + 2 = -2 . Άρα, η Cr

διέρχεται από το σημείο Δ(-2, -2). ι 1 . Έστω η συνάρτηση : {Χ2 - χ + 2λ - 3

f(x) = ...Γχ - 3μ - 2'

'

χ + 1

αν χ � Ο

αν χ > Ο

Να βρείτε τους αριθμούς λ και μ, για τους οποίους η γραφική παράσταση της f διέρ­χεται από τα σημεία Α(-1 ,μ) και Β(1 ,λ+1).

Λύση

Για να συμβαίνει το ζητούμενο πρέπει και αρκεί:

{f( - 1) = μ {1 + 1 + 2λ - 3 = μ f(l) = λ + 1 <:::::> 1 - 3μ - 2 = λ + 1 1 + 1

{2λ - 1 = μ <:::::> - 1 - 3μ = 2λ + 2

{2λ - μ = 1 {-4μ = 4 <:::::> -2λ - 3μ = 3 <:::::> 2λ - μ = 1

<:::::> { �; +-/ = 1 <:::::> { � : � 1 ι 2 . Θεωρούμε τα σημεία:

Α(1 ,2), Β(-1,-4) και Γ(2,5). ι . Α ν y=αχ+β είναι η εξίσωση της ευθείας

ΑΒ να βρείτε τους αριθμούς α και β. 2. Να δείξετε ότι τα τρία αυτά σημεία είναι

συνευθειακά. 3. Να εξετάσετε αν τα σημεία Α,Β και

Δ(-2,-5) είναι συνευθειακά. Λύση

ι . Επειδή η ευθεία ΑΒ διέρχεται από τα σημεία Α( 1 ,2) και Β(-1 ,-4), η εξίσωσή της:y = αχ + β θα επαληθεύεται από τις συντεταγμένες των σημείων αυτών. Έτσι, θα έχουμε:

{2 = α + β {2β = -2 {β = - 1 -4 = -α + β <:::::> α + β = 2 <:::::> α = 3

Άρα, η εξίσωση της ευθείας ΑΒ είναι: y=3x-1 . 2 . Οι συντεταγμένες του σημείου Γ(2,5) επαλη­

θεύουν την εξίσωση : y = 3χ - 1 της ευθείας ΑΒ, γιατί: 5 = 3 · 2 - 1 . Άρα, το σημείο Γ ανήκει

ΕΥΚΛΕΙΔΗΣ Β' λθ ' τ.2/32

Page 35: Ευκλειδης Β 58

Μαθηματικά για την Α ' Λυκέίου

στην ευθεία ΑΒ και συνεπώς τα σημεία Α,Β και Γ είναι συνευθειακά.

3. Οι συντεταγμένες του σημείου Δ(-2,-5) δεν επαληθεύουν την εξίσωση : y = 3χ-1 της ευθεί­ας ΑΒ, γιατί: -5 -:f::. 3 · ( -2) - 1 . Άρα, τα σημεία Α, Β και Δ δεν είναι συνευθειακά.

1 3 . Να βρείτε τους αριθμούς λ Ε JR , για τους οποίους οι ευθείες:

ε1 : y = (λ3 + 3λ)χ + λ + 1 και ε2 : y = (3λ2 + l)x + λ - 3

να είναι παράλληλες. Στη συνέχεια, για τις τιμές του λ που θα βρείτε, να σχεδιάσετε τις ευθείες αυτές στο ίδιο σύστημα αξόνων.

Λύση

Θεωρούμε τις ευθείες:

ε : y = λχ + κ και ε ' : y = λ' χ + κ ' Ισχύουν ο ι εξής ισοδυναμίες:

• ε // ε ' � λ = λ '

• ε _l_ ε ' � λ · λ' = - 1

α. Έχουμε: ε , // ε2 � ΊC + 3λ = 3λ2 + 1 � '){ - 3Λ! + 3λ - 1 = ο � (λ - 1)3 = ο � λ = 1 ο

Άρα, υπάρχει μία μόνο ζητούμενη τιμή του λ, η λ = 1 .

Υ

β. Με λ= 1 , βρίσκουμε: ε1 : y = 4x + 2 ( 1 ) και ε2 : y = 4χ - 2 (2)

χ

Από την ( 1 ) με χ=Ο βρίσκουμε y = 2 και με χ = - 1 , βρίσκουμε y = -2 . Άρα, η ευθεία ε1

διέρχεται από τα σημεία Α(Ο,2) και Β(-1 ,-2). Έτσι, σχεδιάζουμε την ε 1 • Όμοια σχεδιάζουμε και την ε2 (παραπάνω σχήμα).

1 4 . Να βρείτε τους αριθμούς λ Ε JR , για τους οποίους οι ευθείες:

ε, : y = (λ + 2)χ - 3λ + 1 και ε2 : y = (λ + 4)χ + 2λ - 3

να είναι κάθετες.

Έχουμε: Λύση

ε, _l_ ε2 � (λ + 2)(λ + 4) = - 1

λ2 + 4λ + 2λ + 8 + 1 = ο �

λ2 + 6λ + 9 = 0 � (λ + 3)2 = 0 �

λ+ 3 = 0 � λ = -3 . Άρα, υπάρχει μία μόνο ζητούμενη τιμή του λ, η λ = -3 .

1 5 . Μία συνάρτηση f είναι ορισμένη στο JR και για κάθε χ Ε JR ισχύει:

f(x) + 2f(3 - χ) = -2χ + 6 ( 1 )

Α ν y = αχ+ β είναι η εξίσωση της ευθείας, η οποία διέρχεται από τα σημεία A(O,f(O)) και B(3,f(3)), να βρείτε τους αριθμούς α και β.

Λύση

Καταρχήν θα βρούμε τους αριθμούς f(O) και f(3) . Από την ( 1 ) με χ = Ο και χ = 3 βρίσκουμε αντι­στοίχως:

{f(O) + 2f(3) = 6 � {f(O) + 2f(3) = 6 �

f(3) + 2f(O) = Ο -4f(O) - 2f(3) = Ο {-3f(O) = 6 {f(O) = -2 f(3) + 2f(O) = Ο

� f(3) = 4

Συνεπώς Α(Ο, -2) και 8(3 ,4). Επειδή η ευθεία ΑΒ διέρχεται από τα σημεία αυτά η εξίσωσή της: y = αχ + β θα επαληθεύεται από τις συντεταγμένες των σημείων αυτών. Έτσι, έχουμε:

{-2 = α · Ο + β � {β = -2

4 = 3α + β α = 2

ΕΥΚΛΕΙΔΗΣ Β ' λθ ' τ.2/33

Page 36: Ευκλειδης Β 58

a Παpαλληλόγpαμμα

Λ ΥΜΕΝΕΣ ΑΣΚΗΣΕΙΣ 1 . Δίνεται το παραλληλόγραμμο ΑΒΓΔ, στο ε­

σωτερικό της πλευράς του Γ Δ παίρνουμε τυ­χαίο σημείο Ε και ονομάζουμε Η την τομή της ΒΕ με την προέκταση της ΑΔ. Προεκτεί­νουμε την πλευρά Γ Δ προς το μέρος του Δ και παίρνουμε σημείο Ζ ώστε ΔΖ = ΓΕ. Από το σημείο Ζ φέρνουμε παράλληλη ευθεία προς ΑΗ που τέμνει την προέκταση του ευ­θυγράμμου τμήματος ΒΗ στο σημείο Ι. ί. Να αποδειχθεί ότι τα τετράπλευρα Α­

ΖΙΗ και ΑΖΕΒ είναι παραλληλόγραμμα. ίί. Να αποδειχθεί ότι ΒΕ = ΙΗ.

Λύση

Γ

Τα τρίγωνα ΑΔΖ και ΒΓΕ είναι ίσα επειδή : --- -ΔΖ = ΓΕ (από υπόθεση), ΑΔΖ = ΒΓΕ (εντός - ε-

κτός και επί τα αυτά) και ΑΔ = ΒΓ (απέναντι --- -πλευρές παραλληλογράμμου) άρα : ΔΖΑ = ΓΕΒ

- -( 1 ) και ΑΖ = ΒΕ (2). Επίσης ισχύει: ΔΕΗ = ΓΕΒ (3) . Από τις σχέσεις ( 1 ) και (3) προκύπτει ότι

---ΑΖΔ = ΔΕΗ και επειδή είναι και γωνίες εντός ε-ναλλάξ τα ευθύγραμμα τμήματα ΑΖ και ΗΙ θα εί­ναι παράλληλα. Το τετράπλευρο ΑΗΙΖ είναι πα­ραλληλόγραμμο γιατί ΑΗ//ΖΙ και ΑΖ//ΗΙ οπότε ΑΖ = ΙΗ (4). Από τη σχέση ( 1 ) συμπεραίνουμε ότι ΑΖ//ΒΕ και επειδή ΑΒ//ΕΖ το ΑΒΕΖ είναι παραλ­ληλόγραμμο και ισχύει ότι: ΑΖ = ΒΕ (5) . Από τις σχέσεις (4) και (5) έχουμε ότι ΒΕ = Hl.

2. Δίνεται το παραλληλόγραμμο ΑΒΓ Δ. Από την κορυφή Β γράφουμε τυχαία ευθεία (ε)

Ν. Αναστασίου, Κ. Ανεστόπουλος

και έστω Ε, Ζ και Η οι προβολές των σημείων Α, Δ και Γ αντίστοιχα πάνω στην ευθεία (ε). Να αποδειχθεί ότι: ΔΖ=ΑΕ+ΓΗ.

Λύση

Α ' Τρόπος Δ

Έχουμε ΑΕ//ΔΖ//ΓΗ επειδή είναι κάθετες στην ί­δια ευθεία (ε) . Φέρνουμε ΑΚ .l ΔΖ και τα τρίγω­να ΑΔΚ και ΒΓΗ είναι ίσα επειδή Κ = Η = 90° , ΑΔ = ΒΓ (απέναντι πλευρές παρ/μου) και

- -ΑΔΚ = ΒΓΗ (παράλληλες πλευρές), άρα θα έχουν και ΔΚ = ΓΗ ( 1 ) . Από το ορθογώνιο ΑΕΖΚ έχω ότι ΑΕ = ΖΚ (2), άρα ΔΖ = ΔΚ+ΚΖ = ΓΗ+ΑΕ. Β ' Τρόπος

Δ

Έστω Ο το κέντρο του παραλληλόγραμμου ΑΒΓ Δ. Τότε Ο είναι μέσο της διαγωνίου ΑΓ. Αν φέρουμε ΟΚ κάθετη στην ΕΗ το ΟΚ είναι διάμεσος του τραπεζίου ΑΕΗΓ. Έτσι ΑΕ+ΓΗ = 20Κ ( 1 ). Παρα­τηρούμε ότι στο τρίγωνο ΒΔΖ το Ο είναι μέσο του ΒΔ και ΟΚ//ΔΖ τότε ΔΖ = 20Κ (2), από τις ( 1 ),(2) έχουμε ΔΖ = ΑΕ+ΓΗ. 3. Θεωρούμε το παραλληλόγραμμο ΑΒΓ Δ

ώστε ΑΒ = 2ΒΓ. Στις προεκτάσεις του ευ­θυγράμμου τμήματος ΑΔ προς τα σημεία Α και Δ θεωρούμε αντίστοιχα τα σημεία Ε

ΕΥΚΛΕΙΔΗΣ Β' λθ ' τ.2/34

Page 37: Ευκλειδης Β 58

Μαθηματικά για την Α· Λυκείου

και Ζ ώστε ΑΔ = ΑΕ = ΔΖ. Να αποδειχθεί ότι: ΒΖ .l ΓΕ .

Λύση z

Έστω Λ, Κ τα σημεία τομής των ΕΓ και ΒΖ με τα τμήματα ΑΒ και ΔΓ αντίστοιχα. Από την υπόθεση έχουμε ΒΓ=//ΑΕ και ΒΓ=//ΔΖ άρα ΑΓΒΕ και ΒΓΖΔ είναι παραλληλόγραμμα που έχουν κέντρα τα σημεία

Ι Λ και Κ αντίστοιχα. Επομένως ΛΒ = -ΑΒ = ΒΓ 2 και ΚΓ = ..!_ ΔΓ = ΒΓ . Δηλαδή το τετράπλευρο 2 ΑΒΓ Δ είναι ρόμβος οπότε θα έχει διαγώνιες κάθετες δηλαδή ΒΚ .l ΛΓ άρα ΒΖ .l ΓΕ .

Επειδή ισχύει ΒΜ = ΓΝ συμπεραίνουμε ότι ΔΖ = ΔΜ, δηλαδή το τρίγωνο ΔΖΗ είναι ισοσκελές.

ίί. Από το παρ/μο ΒΖΔΜ έχουμε ΜΔ=//ΒΖ (3) και από παρ/μο ΓΝΔΗ έχουμε ΔΝ=//ΗΓ (4). Από (3) και (4) προκύπτει ΒΖ=//ΗΓ άρα ΒΖΓΗ παρ/μο και Ε το κέντρο του, οπότε το ισοσκε­λές τρίγωνο ΖΔΗ η διάμεσος ΔΕ θα είναι και -διχοτόμος της ΖΔΗ . -Οι γωνίες ΒΑΓ και ΖΔΗ είναι ίσες επειδή οι πλευρές τους είναι παράλληλες άρα και οι διχο­τόμοι τους θα είναι παράλληλες. (Γιατί;).

5. Δίνεται τετράγωνο ΑΒΓ Δ, στην προέκταση της πλευράς ΑΒ προς το μέρος του Β θεω­ρούμε το σημείο Ε και στην προέκτασή της πλευράς ΒΓ θεωρούμε το σημείο Ζ ώστε ΒΕ = ΓΖ. Να αποδειχθεί ότι: ί . ΑΖ = ΔΕ και ίί. ΑΖ .l ΔΕ .

Λύση

4. Δινεται τυχαίο τρίγωνο ΑΒΓ. Στις πλευρές i. Τα τρίγωνα ΑΒΖ και ΑΔΕ είναι ίσα επειδή ΑΒ και ΑΓ θεωρούμε αντίστοιχα σημεία Μ ΑΒ = ΑΔ (από υπόθεση), ΒΖ = ΑΕ ( αθροίσμα-και Ν ώστε ΒΜ = ΓΝ. Αν Δ είναι το μέσο του ΜΝ και Ε το μέσο του ΒΓ και φέρουμε ΔΖ=//ΜΒ, ΔΗ=//ΓΝ, τότε να αποδειχθεί ότι: i. Το τρίγωνο ΔΖΗ είναι ισοσκελές, i i . το ΔΕ είναι παράλληλο με την διχοτόμο

της γωνίας Α του τριγώνου ΑΒΓ. Λύση

Α

Γ

ί. Από το παρ/μο ΒΖΔΜ έχουμε ΜΒ=//ΔΖ ( 1 ) και από το παρ/μο ΓΝΔΗ έχουμε ΓΝ=//ΗΔ (2).

τα ίσων ευθυγράμμων τμημάτων) και Α = Β = 90° άρα ΔΕ = ΑΖ και Ε = Ζ .

z

κ

Alf---------08-_____::, E

ίί. Από το ΑΒΖ τρίγωνο προκύπτει ΖΑΒ + Ζ = 90° , όμως Ε = Ζ οπότε ΖΑΒ + Ε = 90° . Δηλαδή το τρίγωνο ΑΚΕ θα είναι ορθογώνιο στο Κ άρα ΑΖ .l ΔΕ .

6. Δίνεται τυχαίο τετράπλευρο ΑΒΓ Δ, θεω­ρούμε τα σημεία Ε και Ζ που είναι τα μέσα των διαγωνίων ΒΔ και ΑΓ αντίστοιχα. Γράφουμε ευθύγραμμο τμήμα ΔΗ ώστε ΔΗ//ΑΒ και ΔΗ = ΑΒ. Να αποδειχθεί ότι

ΗΓ//ΕΖ και ΕΖ = !_ ΗΓ . 2

ΕΥΚΛΕΙΔΗΣ Β ' λθ ' τ.2/35

Page 38: Ευκλειδης Β 58

Μαθηματικά για την Α' Λυκείου

Λύση

Α

Το τετράπλευρο ΑΒΗΔ είναι παραλληλόγραμμο επειδή ΑΒ=//ΔΗ άρα οι διαγώνιες ΒΔ και ΑΗ δι­χοτομούνται στο Ε. Έτσι στο τρίγωνο ΑΓΗ τα Ε και Ζ είναι μέσα των πλευρών ΑΗ και ΑΓ αντί-στοιχα, άρα ΗΓ//ΕΖ και EZ = !Hr . 2 7. Δίνεται τυχαίο ΑΒΓ και Δ είναι ένα εσωτε­

ρικό σημείο της πλευράς ΒΓ. Από το ση­μείο Δ φέρνουμε παράλληλες ευθείες προς τις πλευρές ΑΓ και ΑΒ που τις τέμνουν στα σημεία Ζ και Ε αντίστοιχα. Α ν Η είναι το συμμετρικό σημείο Δ ως προς την ΕΖ, να αποδειχθεί ότι: ί. ΑΗ//ΕΖ και ί ί . το ΑΕΖΗ είναι ισοσκελές τραπέζιο.

Λύση Α

Ε

B�------------�------� r Δ

ί. Το τετράπλευρο ΑΕΔΖ είναι παραλληλόγραμ­μο αφού οι απέναντι πλευρές του είναι παράλ­ληλες. Φέρνουμε από την ΑΘ .1 ΕΖ . Τα ορθο-γώνια τρίγωνα ΑΕΘ και ΔΖΟ είναι ίσα επειδή ΑΕ = ΔΖ και οι οξείες γωνίες ΕΑΘ και ΖΔΟ εί­ναι ίσες αφού έχουν πλευρές παράλληλες, επο­μένως ΑΘ = ΔΟ ( 1 ) . Από την υπόθεση ισχύει ότι ΔΟ = ΟΗ (2). Από την ( 1 ) και (2)έχουμε ότι ΑΘ = ΗΟ, όμως ΑΘ//ΗΟ (κάθετες στην ίδια ευθεία), άρα το ΑΘΟΗ είναι ορθογώνιο, δηλα­δή ΑΗ//ΕΖ.

ί ί . Από την υπόθεση προκύπτει ότι το ΗΖ είναι συμμετρικό του ΔΖ ως προς την ΕΖ. Έτσι ΗΖ = ΔΖ άρα και ΗΖ = ΑΕ. Δηλαδή το τραπέζιο ΑΕΖΗ είναι ισοσκελές.

22ο ΠΑΝΕΛΛΗΝΙΟ ΣΥΝΕΔΡΙΟ ΜΑΘΗΜΑ ΤΙ ΚΗΣ ΠΑΙΔΕΙΑΣ ΛΑΜΙΑ: 1 8 / 1 9 I 20 Νοεμβρίου 2005

Στη Λαμία, από 1 8-20 Νοεμβρίου, στο χώρο του T.E.I . πραγματοποιήθηκε το 22° συνέδριο της ΕΜΕ με με­γάλη επιτυχία. Θέμα του συνεδρίου «Οι σύγχρονες εφαρμογές των Μαθηματικών και η αξιοποίησή τους στη Εκπαίδευση».

Κάθε χρόνο το συνέδριο της ΕΜΕ είναι ένα κοσμικό πλέον γεγονός και ιδιαίτερα για την περιοχή του πα­ραρτήματος στο οποίο φιλοξενείται κάθε φορά.

Μεγάλη υπήρξε και η συμμετοχή με εργασίες (66) που ανακοινώθηκαν στο συνέδριο αλλά και των συνέ­δρων που το παρακολούθησαν (850) . Οι κεντρικές ομιλίες και το στρογγυλό τραπέζι τράβηξαν το ενδιαφέρον των συνέδρων.

23ο ΠΑΝΕΛΛΉΝΙΟ ΣΥΝΕΔΡΙΟ ΜΑΘΗΜΑ ΤΙ ΚΗΣ ΠΑΙΔΕΙΑΣ ΠΑ ΤΡΑ:24 125 I 26 Νοεμβρίου 2006

Η επιτυχημένη πορεία της ΕΜΕ και ο θεσμός των συνεδρίων θα συνεχιστεί το 2006 με το 23° συνέδριο που θα διεξαχθεί στην Πάτρα στις 24-26 Νοέμβρη με θέμα: «Τα Μαθηματικά ως πολιτισμός στο σύγχρονο κό­σμο: προεκτάσεις στην κριτική σκέψη, την επιχειρηματολογία και την αισθητική».

Σκοπός του συνεδρίου θα είναι να αναδειχθεί ότι τα Μαθηματικά είναι ένα από τα πιο σημαντικά πολιτιστι­κά συστατικά στο σύγχρονο κόσμο.

ΕΥΚΛΕΙΔΗΣ Β ' λθ ' τ.2/36

Page 39: Ευκλειδης Β 58

ΞJaΙιιιι•"�••• ,_ .. � w .. �,., .. _, Αιι--ι•

ΑΛfΕΒΡΑ ΠΟΛ ΥΩΝΥΜΑ - ΕΞΙΣΩ:ΣΕΙΣ- Α.ΝΙΣΩΣΕΙΣ

Λ ΥΜΕΝΕΣ ΑΣΚΗΣΕΙΣ 1. Ένα πολυώνυμο Ρ(χ) διαιρούμενο με χ - 2

αφήνει υπόλοιπο 3 και διαιρούμενο με χ+ 1 αφήνει υπόλοιπο -3. Τι υπόλοιπο θα αφήσει αν διαιρεθεί με το γινόμενο (χ-2)(χ+1) ;

Λύση

Το υπόλοιπο υ(χ) θα είναι το πολύ ένα πολυώνυμο 1 ου βαθμού δηλ. υ( χ) = αχ + β , αφού ο διαιρέτης (χ-2)(χ+ 1 ) είναι 2ου βαθμού. Τότε, από την ταυτότητα της διαίρεσης θα είναι:

Ρ( χ) = (χ - 2)(χ + 1)π(χ) + αχ + β , όπου π(χ) το πηλίκο αυτής της διαίρεσης. Για χ = 2 η ταυτότητα δίνει: Ρ(2) = 2α + β = 3 και για χ = - 1 είναι: Ρ( -1 ) = -α + β = -3 .

2α + β = 3 Λύνουμε το σύστημα -α + β = -3 και βρίσκουμε α = 2, β = - 1 . Άρα το υπόλοιπο που ζητάμε είναι: υ( χ) = 2 χ - 1 . 2. Δίνεται το πολυώνυμο

Ρ( χ) = χ5 - 2αχ3 + βχ2 + 3γ - 6 . Να βρεθούν οι πραγματικοί αριθμοί α, β, γ ώστε το Ρ(χ) να έχει παράγοντα το πολυώνυμο χ3 -χ.

Λύση

Είναι χ3 - χ = χ(χ + 1)(χ - 1) . Επειδή το χ είναι παράγοντας του Ρ(χ) θα είναι Ρ( Ο) = Ο δηλ. 3γ - 6 = Ο <=> γ = 2 . τότε το Ρ( χ) γί-νεται Ρ( χ) = χ5 - 2αχ3 + βχ2 = χ ( χ4 - 2αχ2 + βχ ) άρα το πηλίκο της διαίρεσης Ρ(χ) :χ είναι χ4 - 2αχ2 + βχ .

Γιώργος Αποστολόπουλος

Το χ+ 1 είναι παράγοντας του πηλίκου χ4 - 2αχ2 + βχ , με το σχήμα HORNER έχουμε:

ο -2α β ο

Θ � - 1 2α- 1 -2α-β+1 - 1 -2α+1 2α+β- 1 I -2α-β+l=Ο

Επίσης το χ- 1 επειδή είναι παράγοντας του νέου πηλίκου χ3 - χ2 + ( -2α + 1)χ + 2α + β - 1 θα έχουμε πάλι το σχήμα HORNER.

1 - 1 -2α+Ι 2α+β- Ι

CD --�--���--�o--_,_-2_α+_I __ _

. 1 οποτε α = - . 2

Ο -2α+1 β=Ο

Άρα α = .!. , β = Ο, γ = 2 . 2 3 . Για το πολυώνυμο Ρ( χ) ισχύουν:

Ρ(2) = 2 και Ρ(χ) = Ρ(2-χ), για κάθε χ ε JR .

Να βρεθεί το υπόλοιπο της διαίρεσης: Ρ( χ) : ( χ2 - 2χ) .

Λύση

Το υπόλοιπο θα είναι το πολύ 1 ου βαθμού δηλ. υ( χ) = αχ + β . Τότε θα έχουμε:

Ρ( χ) = { χ2 - 2χ )π( χ) + αχ + β . Για χ = 2 έχουμε: Ρ(2) = 2α + β = 2 . Αλλά από την ταυτότητα Ρ( χ) =. Ρ(2 - χ) έχουμε για χ = 2, Ρ(2) = Ρ(Ο) δηλ. Ρ(Ο) = 2 .

ΕΥΚΛΕΙΔΗΣ Β ' λθ ' τ.2/37

Page 40: Ευκλειδης Β 58

Μαθηματικά Β' Λυκείου

Στην ταυτότητα της διαίρεσης για χ = Ο έχομε Ρ( Ο) = β = 2 , οπότε 2α + 2 = 2 <::::::> α = Ο . Δηλαδή το υπόλοιπο της διαίρεσης που ζητάμε εί­ναι υ(χ) = Ο · χ + 2 = 2 . 5. Δίνεται το πολυώνυμο

Ρ(χ) = χ2• + χ• - 2 , ν ε f{ .

ί. Αν το Ρ(χ) έχει ρίζα το -1 , να δειχθεί ότι ο φυσικός ν είναι άρτος αριθμός.

ί ί . Αν ισχύει Ρ(2) = 70, να βρεθεί ο βαθμός του Ρ(χ) και μετά να παραγοντοποιηθεί το Ρ(χ).

Λύση ί. Αφού το - I είναι ρίζα του Ρ(χ) θα είναι: Ρ(- 1 ) = Ο δηλ. (- 1)2v + (- l)v - 2 = 0 <::::::> 1 + (- l)v - 2 = 0 δηλ. (- Ι γ = I άρα ο ν είναι άρτιος. ίί . Αφού Ρ(2) = 70 θα είναι:

22v + 2v - 2 = 70 <::::::> (2v )2 + 2v - 72 = Ο θέτου-με 2v = ω > Ο , οπότε είναι: ω2 + ω - 72 = Ο που έχει ρίζες ω = 8 (δεκτή) ή ω = -9 ( απορρίπτε­ται) . Άρα 2v = 8 <::::::> ν = 3 Τότε το Ρ( χ) = χ6 + χ3 - 2 έχει βαθμό 6. Παρατηρούμε ότι για χ = I το Ρ( χ) μηδενίζεται. Κάνοντας σχήμα Homer έχουμε:

ι ο ο ι ο ο -2

ω � 2 2 2

2 2 2 [Ξi Άρα Ρ( χ) = (χ - I) ( xs + χ4 + χ3 + 2χ2 + 2χ + 2 ) = = (χ - I) [ χ2 ( χ3 + 2 ) + χ ( χ3 + 2 ) + ( χ3 + 2 ) J = = (χ - 2) ( χ3 + 2 ) ( χ2 + χ + I ) .

6. Αν το πολυώνυμο Ρ(χ) διαιρούμενο με χ-α, χ-β, χ-γ αφήνει υπόλοιπα α, β, γ αντίστοιχα, με α =ι:. β =ι:. γ =ι:. α , τότε το υπόλοιπο της διαίρεσης Ρ(χ) : (χ - α)(χ - β)(χ - γ) θα εί­ναι το πολυώνυμο υ( χ) = χ .

Λύση Αν π(χ) είναι το πηλίκο αυτής της διαίρεσης, το υπόλοιπο υ(χ) θα είναι το πολύ 2ου βαθμού πο-

λυώνυμο, δηλ. υ( χ) = κχ2 + λχ + μ . Από την ταυ­τότητα της διαίρεσης

Ρ( χ) = (χ - α)(χ - β)(χ - γ)π(χ) + κχ2 + λχ + μ για χ = α, χ = β, χ = γ προκύπτουν:

α2κ + αλ + μ = α β2κ + βλ + μ = β γ2κ + γλ + μ = γ

Αφαιρώντας τις δύο πρώτες εξισώσεις κατά μέλη και τις δύο τελευταίες, θα έχουμε

(α2 - β2 ) κ + (α - β)λ = α - β

και (β2 - γ2 ) κ + (β - γ)λ = β - γ επειδή α =ι:. β και β =ι:. γ θα έχουμε:

(α + β)κ + λ = Ι και (β + γ)κ + λ = Ι από τις οποίες με αφαίρεση κατά μέλη προκύπτει (α - γ)κ = Ο και επειδή α ;t γ θα είναι κ = Ο. Αντικαθιστούμε στην εξίσωση (β + γ)κ + λ = I το κ = Ο απ' όπου προκύπτει λ = I . Οπότε η προηγούμενη σχέση α2κ + αλ + μ = α δί-νει για κ = Ο, λ = I ότι: μ = Ο. Άρα το υπόλοιπο εί­ναι το πολυώνυμο: υ( χ) = χ . 7. Δίνονται τα πολυώνυμα

Ρ( χ) = χ5 - 4χ4 + 3χ3 - 2χ2 + 8χ - 6 και Q(x) = χ2 - 4χ + 3 . Χωρίς να εκτελέσετε τη διαίρεση Ρ( χ) : Q(x) , να δείξετε ότι αυτή είναι τέλεια.

Λύση Είναι Ρ(χ) = χ3 (χ2 - 4χ + 3 ) - 2 (χ2 - 4χ + 3 ) = = ( χ2 - 4χ + 3 ) ( χ3 - 2 ) = Q(x) · ( χ3 - 2 ) . Δηλαδή Ρ( χ) = Q(x) · ( χ3 - 2 ) οπότε η διαίρεση Ρ( χ) : Q( χ) είναι τέλεια. 8. Αν Ρ(χ) = χ1 5 - 4χ14 + 2χ13 + 2006 , να υπο­

λογιστεί η τιμή της παράστασης: Ρ ( 2 +

J2) + Ρ ( 2 -J2)

2 Λύση Είναι Ρ(χ) = χ 1 3 (χ2 - 4χ + 2 ) + 2006 . Οι ρίζες του τριωνύμου ·χ 2 - 4χ + 2 είναι

χ 1 = 2 +.fi

, χ2 = 2 -.fi

. ΕΥΚΛΕΙΔΗΣ Β ' λθ ' τ.2/38

Page 41: Ευκλειδης Β 58

Μαθηματικά Β ' Λυκείου

Άρα P (2 +J2) = 2006 , P (2 -

J2) = 2006 και η

τιμή της παράστασης θα είναι 2006 + 2006 = 2006 .

2 9. Για το πολυώνυμο

Ρ( χ) = χ6 + χ4 + 4χ3 - 2χ2 + 5 '

να δειχθεί ότι Ρ( χ) > χ5 - χ3 + 2χ2 - 2 , για κάθε x E JR .

Λύση Είναι Ρ(χ) = (χ6 + 4χ3 + 4) + ( χ4 - 2χ2 + 1 ) =

= (χ3 + 2)2 + (χ2 - 1 )2 .

Είναι γνωστό ότι α2 + β2 > αβ ( 1 ) όταν α -::f:. Ο ή β -::f:. Ο . Εδώ είναι α = χ3 + 2 , β = χ2 - Ι αλλά για χ = ± I είναι α -::f:. Ο , οπότε από: Ρ( χ) > ( χ3 + 2 ) ( χ2 - I ) <=>

Ρ( χ) > χ5 - χ3 + 2χ2 - 2 για κάθε χ Ε JR . 1 Ο. Δίνεται το πολυώνυμο

Ρ( χ) = χ3 + χ2 + λ2χ - ι , όπου λ Ε JR .

ί. Δείξετε ότι για κάθε λ Ε JR , το Ρ( χ) δεν έχει ακέραιες ρίζες.

ίί. Αν ισχύει Ρ ( .Ji) = 3.J2 + 1 , να βρείτε

Λύση

τις τιμές του λ και την τιμή της παρά­στασης Ρ(α)+Ρ(β), όπου α, β είναι οι ρί­ζες της εξίσωσης χ2 + 2χ - 1 = Ο .

ί. Αν είχε ακέραια ρίζα το Ρ(χ) αυτή θα ήταν διαι­ρέτης - I , δηλ. το + I ή το - 1 . Έχουμε: P(l) = Ο <=> I + Ι + λ2 - 1 = Ο <=> λ2 = -1 άτοπο και: Ρ( -1) = Ο <=> -1 + 1 - λ2 - 1 = Ο <::> λ2 = - 1 άτοπο. Άρα το Ρ(χ) δεν έχει ακέραια ρίζα για κάθε λ E JR .

ίί. Ρ ( J2) = 3

.)2 + 1 <=>

<=> .)23 +

J22 + J2λ2 - 1 = 3

.)2 + 1 <=>

λ2 = 1 <=> λ = -1 ή λ = 1 . Το Ρ(χ) γράφεται Ρ( χ) = χ3 + χ 2 + χ - 1 . Ρ( α) + Ρ(β) = α3 + α2 + α - 1 + β3 + β2 + β - 1 =

= (α3 + β3 ) + (α2 + β2) + (α + β ) - 2 =

= (α + β)3 - 3αβ(α + β) + (α + β)2 --2αβ + (α + β) - 2 αλλά από τους τύπους VIET Α έχουμε α + β = -2 και αβ = - 1 , οπότε: Ρ(α) + Ρ(β) = - 12 .

1 1 . Δίνεται το πολυώνυμο Ρ(χ) = χ• + 2χν-ι + 3χ + ν2 + 2ν - 20 ,

ν φυσικός αριθμός, που έχει το -1 ρίζα του. ί. Να δείξετε ότι το Ρ( χ) είναι 4ου βαθμού. ίί. Να δείξτε ότι το Ρ( χ) δεν έχει άλλες ακέραι­

ες ρίζες. ίίί. Να λυθεί η aνίσωση : Ρ( χ) + Ρ(-χ) < 16 . iv. Να λυθεί η εξίσωση : Ρ(χ) · Ρ(-χ) + 84 = 0

στο σύνολο των ακεραίων αριθμών. Λύση ί. Θα είναι

Ρ(-1) = 0 <::> <::> ( - 1)v + 2( - 1)v-

l- 3 + ν2 + 2ν - 20 = Ο .

Α ν ο ν είναι περιττός φυσικός αριθμός, τότε - 1 + 2 - 3 + ν2 + 2ν - 20 = Ο <=> ν2 + 2ν - 22 = Ο αλλά η διακρίνουσα Δ = 92 δεν είναι τέλειο τετράγωνο φυσικού αριθμού δηλ. ο ν δεν είναι φυσικός αριθμός πράγμα άτοπο. Άρα ο ν είναι άρτιος φυσικός αριθμός. Τότε η εξίσωση Ρ(- 1 ) = Ο γίνεται 1 - 2 - 3 + ν2 + 2ν - 20 = Ο <=> ν2 + 2ν - 24 = Ο που έχει ρίζες ν = 4 ή ν = -6 (η οποία απορρί­πτεται) . Άρα ν = 4, δηλαδή το Ρ(χ) είναι 4ου βαθμού.

Ρ( χ) = χ4 + 2χ3 + 3χ + 4

ίί. Οι διαιρέτες του +4 είναι: ±1 ,± 2, ± 4 . Οι θετικοί διαιρέτες απορρίπτονται, Ρ( -2) -::f:. Ο , P(-4) -::f:. O , Ρ(-1) = 0 , δηλ. το Ρ(χ) εκτός του -1 δεν έχει άλλη ακέραια ρίζα.

iii. Ρ(χ) + Ρ(-χ) < 1 6 <::> . . . <::> χ4 < 4 <::> (χ2 + 2) (χ2 - 2) < 0 <::> χ2 - 2 < 0 ,

γιατί χ2 + 2 > Ο για κάθε χ Ε JR , άρα -

J2 < χ <

J2 .

iv. Ρ(χ)Ρ( -χ) + 84 = Ο <=> ( χ4 + 2χ3 + 3χ + 4) . ( χ4 - 2χ3 - 3χ + 4) + 84 = ο

ΕΥΚΛΕΙΔΗΣ Β ' λθ ' τ.2/39

Page 42: Ευκλειδης Β 58

Μαθηματικά Β' Λυκείου

<=> . . . <=> χ8 - 4χ6 - 4χ4 - 9χ2 + 1 00 = ο θέτουμε χ 2 = ω > Ο , θα είναι

ω4 - 4ω3 - 4ω2 - 9ω + 1 00 = Ο . Επειδή θέλουμε τις ακέραιες λύσεις, θα πρέπει να πάρουμε τους θετικούς διαιρέτες που είναι τέλεια τετράγωνα, δηλ. τους 1 , 4, 25 , 1 00. Με σχήμα HORNER, για ω = 4 έχουμε:

1 -4 -4 -9 1 00

� �� ____ 4 ____ 0 ___ -_1 6 __ �- 1_0_0 __ _

Ε ο -4 -25

Άρα ισοδύναμα έχουμε: (ω - 4)(ω3 - 4ω - 25) = 0 <=:> ω = 4 ή ω3 - 4ω - 25 = Ο . Εύκολα διαπιστώνουμε ότι η εξίσωση αυτή δεν έχει ακέραια ρίζα, δηλ. ω = 4 οπότε

χ 2 = 4 <=> χ = 2 ή χ = -2 . 1 2 . Έστω το πολυώνυμο

Ρ( χ) = αχ3 + βχ2 + γχ + 2005 ,

όπου α,β,γ είναι ακέραιοι αριθμοί με α =ι:. Ο . Α ν ο αριθμός P(l) είναι περιττός ακέραιος, να δειχθεί ότι η εξίσωση Ρ( χ) = Ο δεν έχει ακέραιες ρίζες.

Λύση Α ν η εξίσωση Ρ( χ) = Ο είχε ακέραια ρίζα άρτιο αριθμό, τότε θα έπρεπε να ήταν διαιρέτης του στα­θερού όρου 2005 . Αυτό όμως δεν γίνεται, γιατί εί-

ναι γνωστό ότι ένας άρτιος αριθμός ποτέ δεν διαι­ρεί έναν περιττό. Αν η Ρ( χ) = Ο έχει ρίζα ρ περιττό ακέραιο αριθμό, τότε Ρ(ρ) = Ο . Σχηματίζουμε την διαφορά Ρ(ρ) - P(l) = = αρ3 + βρ2 + γ ρ + 2005 - (α + β + γ + 2005) <:::> Ο - P(l) = α (ρ3 - 1 ) + β (ρ2 - 1 ) + γ(ρ - 1) <:::> -P(l) = α (ρ3 - 1 ) + β (ρ2 - 1 ) + γ(ρ - 1) ( 1 ) . Οι ρ, ρ2 και ρ3 είναι περιττοί αριθμοί οπότε ρ- 1 , ρ2-1 και ρ3 - 1 θα είναι άρτιοι και λόγω της ( 1 ) ο -P( l ) είναι άρτιος οπότε και ο Ρ( 1 ) είναι άρτιος αριθμός, που είναι άτοπο. Άρα η εξίσωση Ρ(χ) = Ο δεν έχει ακέραιες ρίζες. 1 3 . Να δειχθεί ότι ο αριθμός

9 · 1 2ν - 2 · 5ν , ν Ε .Ν

είναι πολλαπλάσιο του 7. Λύση Θεωρούμε το πολυώνυμο Ρ( χ) = 9 . χ ν - 2 . 5ν , έ-χουμε Ρ(5) = 7 · 5v . Η διαίρεση Ρ( χ) : (χ - 5) δίνει κάποιο πηλίκο π( χ) και υπόλοιπο το Ρ(5) άρα είναι:

Ρ(χ) = (χ - 5)π(χ) + 7 · 5ν ( 1 ) ( I ) οπότε Ρ(1 2) = 9 · 1 2v - 2 · 5v = (1 2 - 5)π(1 2) + 7 · 5v

δηλαδή : 9 · 1 2ν - 2 · 5ν = 7 {π(1 2) + 5ν } = πολ/σιο του 7 .

ΓΕΩΜΕΤΡΙΑ: Εμβαδά Αργυρόπουλος Ηλίας, Καβαδίας Κων/νος, Καραγιάννης Παναγιώτης

Το εμβαδόν ενός πολυγωνικού χωρίου είναι βασική και δύσκολη έννοια.

Στην προσπάθειά μας να βοηθήσουμε τους μαθητές για την καλύτερη κατανόησή της δίνουμε ορισμένες

αντιπροσωπευτικές ασκήσεις.

Σε κάθε άσκηση ακολουθεί σχόλιο σχετικό με τη σκέψη - κλειδί για την λύση της.

Λ ΥΜ Ε Ν ΕΣ ΑΣΚΗΣΕΙΣ

Άσκηση Ι η Δίνεται τρίγωνο ΑΒΓ και δύο τυχαία σημεία Κ, Λ της πλευράς του ΒΓ. Α ν Μ, Ν είναι τα μέσα

των πλευρών ΑΒ, ΑΓ αντίστοιχα να αποδείξετε ότι: ί . (ΑΜΚΝ) = ji(ABΓ) .

ί ί . (ΚΜΝ) = (ΛΜΝ).

ΕΥΚΛΕΙΔΗΣ Β' λθ ' τ.2/40

Page 43: Ευκλειδης Β 58

Μαθηματικά Β ' Λυκείου

Λύση Α

ί . Φέρουμε την ΑΚ. Στο τρίγωνο ΑΚΒ η ΚΜ εί­ναι διάμεσος, επομένως έχουμε (ΚΜΑ) =

_!_(ΑΚΒ) . 2

Αλλ , ΘΕ 1 ( . ) , (ΕΘΖ) 1 , α - = - γιατι; οποτε -- = - αρα ΒΕ 3 (ΕΒΓ) 9

(ΕΘΖ) = _!_

(ΕΒΓ) ( 1 ) . 9 Επειδή ΒΕ διάμεσος στο ΑΒΓ είναι

(ΒΕΓ) = _!_ (ΑΒΓ) (2). 2 Από ( 1 ), (2) προκύπτει:

1 1 1 (ΕΘΖ) = - · - (ΑΒΓ) ή (ΕΘΖ) = -36 τ.μ. 9 2 1 8 Όμοια η ΚΝ είναι διάμεσος στο τρίγωνο ΑΚΓ δηλαδή (ΕΘΖ) = 2 τ.μ. οπότε έχουμε (ΚΝΑ) = � (ΑΚΓ) . Προσθέτο- . Οδηγός σκέψης.

ί. Στα εμβαδά η ομοιότητα αξιοποιείται με την ντας τις δύο σχέσεις κατά μέλη έχουμε: πρόταση : ο λόγος των εμβαδών δύο πολυγώ-(ΑΜΚΝ) = _!_( (ΑΚΒ) + (ΑΓΚ) ] = _!_ (ΑΒΓ) . 2 2

ί ί . Επειδή ΜΝ//ΒΓ (γιατί Μ, Ν μέσα των πλευρών ΑΒ, ΑΓ αντίστοιχα) τα τρίγωνα ΚΜΗ και ΛΜΝ έχουν την ίδια βάση ΜΝ και ίσα τα αντί­στοιχα προς αυτή ύψη, άρα θα είναι

(ΚΜΝ) = (ΛΜΝ). Οδηγός σκέψης.

ί . Η διάμεσος ενός τριγώνου χωρίζει αυτό σε δύο ισοδύναμα τρίγωνα.

ί ί . Όταν δύο τρίγωνα έχουν κοινή βάση και ίσα ύψη είναι ισοδύναμα.

Άσκη ση 2η Δίνεται τρίγωνο ΑΒΓ και οι διάμεσοι του ΑΔ και ΒΕ. Από το κέντρο βάρους του Θ φέρουμε

νων ισούνται με το τετράγωνο του λόγου ο­μοιότητας.

ίί. Το κέντρο βάρους του τριγώνου χωρίζει κάθε διάμεσο σε τμήματα με λόγω 2 : 1 .

Άσκηση 3η Δίνεται τρίγωνο ΑΒΓ και σημείο Δ της πλευράς

ΒΓ ώστε ΒΔ = .!. ΒΓ . Α ν Ε, Ζ και Η είναι ση-3

μεία των ΑΔ, ΒΕ και ΖΓ αντίστοιχα ώστε: ι ι ι ΑΕ = -ΑΔ , ΒΖ = -ΒΕ και ΓΗ = -ΓΖ . Να 3 3 3

αποδείξετε ότι:

ί . (ΕΖΗ) = �(ΖΕΓ) 3

ί ί ί . (ΕΖΗ) = _!_(ΑΒΓ) 27

ί ί . (ΖΕΓ) = �(ΒΕΓ) 3

παράλληλη προς τη ΒΓ που τέμνει την ΑΓ στο Λύση

Ζ. Αν (ΑΒΓ) = 36 τ.μ. να υπολογίσετε το εμβα- Α

δόν του τριγώνου ΘΕΖ. Λύση

Α

ί . Τα τρίγωνα ΕΖΗ και ΕΖΓ έχουνε κοινό το ύψος από την κορυφή Ε, οπότε

(ΕΖΗ) ΖΗ 2 (ΕΖΓ) =

zr = 3 , Επειδή ΘΖ//ΒΓ τα τρίγωνα ΘΕΖ και ΒΕΓ είναι αφού ΗΓ = Jj ΖΓ , άρα (ΕΖΗ) = χ (ΕΖΓ) .

( )2 ΕΘΖ ΘΕ όμοια, τότε _( __

) = -(ΕΒΓ) ΒΕ

ΕΥΚΛΕΙΔΗΣ Β ' λθ ' τ2/41

Page 44: Ευκλειδης Β 58

Μαθηματικά Β' Λυκείου

i i . Τα τρίγωνα ΖΕΓ και ΒΕΓ έχουν κοινό ύψος , , Γ , (ΖΕΓ) ΖΕ 2 απο την κορυφη οποτε -- = - = - , (ΒΕΓ) ΒΕ 3

αφού ΒΖ = .!_ ΒΕ , άρα (ΖΕΓ) = � (ΒΕΓ) . 3 3 ί ί ί . Από τις ί) και ίί) έχουμε (ΕΖΗ) = i (ΒΕΓ) ( 1 ) . 9

Αλλά (ΒΕΓ) = (ΒΔΕ) + (Γ ΔΕ) (2). Επίσης τα τρίγωνα ΒΔΕ και ΒΔΑ έχουν κοινό το ύψος από την κορυφή Β και τα τρίγωνα Γ ΔΕ και ΑΔΓ έχουν κοινό ύψος από την κορυφή Γ. οπότε (ΒΔΕ) = � (ΒΑΔ) (3) 3

(ΓΔΕ) = � (ΑΔΓ) (4) 3 Από τις ( 1 ), (2), (3 ) και (4) έχουμε διαδοχικά (ΕΖΗ) = i (ΒΕΓ) = i [� (ΒΑΔ) + � (ΑΔΓ)] 9 9 3 3

= _!_ (ΑΒΓ) 27 Οδηγός σκέψης.

Όταν δύο τρίγωνα έχουν ίσα ύψη, ο λόγος των εμ­βαδών τους ισούται με τον λόγο των αντίστοιχων βάσεων. Άσκηση 4η Δίνεται τρίγωνο ΑΒΓ και Δ, Ε, Ζ τα μέσα των πλευρών του ΑΒ, ΒΓ, Γ Α αντίστοιχα. Αν είναι ΑΗ το ύψος του, ΕΖ = 5cm, ΒΓ = 16cm και η α­πόσταση του Δ από το ΑΗ ισούται με 3 cm, τότε: i. Να αποδείξετε ότι ΗΕ = 2cm. ii. Να υπολογίσετε το εμβαδόν του ΗΔΖΕ. Λύση

Α

Η ΖΕ 1 1= ΒΑ (2) αφού Ζ, Ε μέσα πλευρών. 2 Από ( 1 ) και (2) έχουμε ΗΔ = ΕΖ = 5cm. Επειδή τα Δ, Ζ μέσα πλευρών, η ΔΖ//ΒΓ οπότε το ΗΔΖΕ είναι ισοσκελές τραπέζιο. Από το Ε φέρουμε την ΕΛ κάθετο στην ΔΖ, τότε τα τρίγωνα ΗΔΚ και ΕΛΖ είναι ίσα (γιατί;) οπότε ΛΖ = ΔΚ = 3cm. Το ΚΛΕΗ είναι ορθογώνιο οπότε ΗΕ = ΚΛ. Είναι ΔΖ = ΔΚ+ΚΛ+ΛΖ ή ΒΓ = 3 + ΚΛ + +3 ή 2 � = 6 + ΚΛ ή ΚΛ = 2cm άρα και ΗΕ = 2cm. 2

ii. (ΗΔΖΕ) = ΗΕ + ΔΖ · ΗΚ ή 2 (ΗΔΖΕ) = 2 + 8 · ΗΚ ή (ΗΔΖΕ) = 5 · ΗΚ . 2 Είναι όμως ΗΚ = -JΗΔ2 - ΔΚ 2 ή ΗΚ = 4cm. Τελικά (ΗΔΖΕ) = 5 · 4 = 20cm2

Οδηγός σκέψης.

i. Το ευθύγραμμο τμήμα που ορίζουν τα μέσα δύο πλευρών ενός τριγώνου είναι παράλληλο προς την τρίτη και ίσο με το μισό της.

ίί. Η διάμεσος ορθογωνίου τριγώνου που αντιστοιχεί στην υποτείνουσα ισούται με το μισό της.

Άσκηση 5η Δίνεται παραλληλόγραμμο ΑΒΓ Δ με Α < 90° και περίμετρο 12 cm. Α ν η διχοτόμος της γωνί­ας Α τέμνει την διαγώνιο ΔΒ σε τμήματα με μή­κη 1 cm και 2cm, να υπολογίσετε το εμβαδόν: i. του παραλληλόγραμμου ΑΒΓ Δ και ίί. του τραπεζίου ΑΔΕΒ, όπου Ε το σημείο που

τέμνει η διχοτόμος την πλευρά ΒΓ του πα­ραλληλόγραμμου.

Λύση

k<VΓ .Ν.. Υ Δ i. Το τρίγωνο ΑΒΗ είναι ορθογώνιο στο Η και η i. Θέτουμε ΑΒ = χ και ΑΔ = y. Από το θεώρημα

ΗΔ διάμεσος οπότε ΗΔ = ΒΑ ( 1 ) . εσωτερικής διχοτόμου στο ΑΒΔ έχουμε 2

ΕΥΚΛΕΙΔΗΣ Β' λθ ' τ2/42

ι(

Page 45: Ευκλειδης Β 58

Μαθηματικά Β ' Λυκείου

ΚΒ ΑΒ 1 χ , - = - <:::::> - = - αρα ΚΔ ΑΔ 2 y L = � = χ + Υ = � = 2cm . 2 1 1 + 2 3 Επομένως y = 4cm και χ = 2cm. Επειδή ΒΔ διαγώνιος στο ΑΒΓΔ θα είναι (ΑΒΓΔ) 2(ΑΒΔ) και από τον τύπο του Ήρωνα έχουμε (ΑΒΔ) = �τ( τ - α)( τ - β)( τ - γ) όπου

4 + 2 + 3 ' 9 τ = η τ = - . 2 2 Επομένως

--���--��--�

(ΑΒΔ) = �(� - 4) (� - 3 )(� - 2) (ΑΒΔ) = 3.Jl5 cm2 . 4 Άρα (ΑΒΓΔ) = 2 . 3.J15 = 3.J15 cm2 • 4 2

ή

ίί . Για τον υπολογισμό του εμβαδού του τραπεζίου αρκεί να βρούμε τη βάση του ΒΕ και το ύψος του ΒΖ. Το τρίγωνο ΑΒΕ είναι ισοσκελές (για­τί;) άρα ΒΕ = ΑΒ = 2cm. 'Εχουμε (ΑΒΔ) = .!_ΑΔ · ΒΖ ή 3.Jl5 = .!_4ΒΖ ή 2 4 2 ΒΖ = 3.Jl5 cm . Τότε (ΑΒΕΔ) = ΒΕ + ΑΔ . ΒΖ

8 z

ή (ΑΒΕΔ) = 2 + 4 . 3Μ . 2 8

Άρα (ΑΒΕΔ) = 9.Jl5 cm2 . 8

Οδηγός σκέψης.

Η εσωτερική διχοτόμος μιας γωνίας ενός τριγώνου χωρίζει την απέναντι πλευρά σε μέρη ανάλογα των προκείμενων πλευρών.

Άσκηση 6η

Δίνεται ορθογώνιο τρίγωνο ΑΒΓ {Α = 90° ) . Εκτός του τριγώνου κατασκευάζουμε τα τετρά­γωνα ΒΓ ΔΕ και ΑΓΖΗ. Α ν Κ τυχαίο σημείο της ΑΓ να δείξετε ότι: ί. ( ΔΚΓ) = (ΖΓΚ)

i i. (ΑΔΓ) + (ΑΕΒ) = .!_ΒΓ2 • 2

Λύση

ί. Τα τρίγωνα ΔΚΓ και ΖΚΓ έχουν κοινή βάση την πλευρά ΚΓ. Αν ΔΛ και ΖΓ τα ύψη τους εί­ναι ΔΛ = ΖΓ διότι τα τρίγωνα ΔΛΓ, ΑΒΓ είναι ίσα (γιατί;) . Άρα (ΔΚΓ) = (ΖΓΚ).

ίί . Φέρουμε το ύψος ΕΜ του τριγώνου ΑΒΕ. Τότε από την ισότητα των τριγώνων ΑΓΒ και ΕΒΜ προκύπτει ΕΜ = ΑΒ. Έτσι έχουμε:

1 1 (ΑΔΓ) + (ΑΕΒ) = -ΑΓ · ΔΛ + -ΑΒ · ΕΜ 2 2 = .!_ΑΓ · ΑΓ + .!_ΑΒ · ΑΒ = .!_ (ΑΓ2 + ΑΒ2 )

2 2 2 = .!..ΒΓ2 • 2

Άλλος τρόπος

ί. Είναι (ΖΓΚ) = .!_ΚΓ · ΖΓ και 2 (ΔΚΓ) = .!.. κΓ · ΓΔημ(90° +Γ)=.!.. κΓ · ΓΔσυνΓ 2 2

1 , (ΔΚΓ) - ΚΓ · Γ Δ · συν Γ , (ΔΚΓ) αρα -- = 2 η -- = 1 (ΖΚΓ) .!.. κΓ · ΖΓ (ΖΚΓ)

2 , ΑΓ ΖΓ γιατι συνΓ = - = - . ΒΓ ΓΔ

Επομένως (ΔΚΓ) = (ΖΓΚ). ii. Είναι ( ΑΔΓ) = .!_ ΑΓ · Γ Δ · ημ(90° + Γ) 2

= .!_ΑΒ · ΒΓ · συνΓ 2 και (ΑΒΕ) = .!_ΑΒ · ΒΕ · ημ(90ο + Β) 2 = .!_ ΑΒ · ΒΓ · συν Β . 2 Επομένως (ΑΔΓ) + (ΑΒΕ) = = .!_ΑΒ · ΒΓσυνΓ + .!_ΑΒ · ΒΓ · συνΒ 2 2

ΕΥΚΛΕΙΔΗΣ Β ' λθ ' τ2/43

Page 46: Ευκλειδης Β 58

Μαθηματικά Β' Λυκείου

ι = -ΒΓ(ΑΓσυνΓ + ΑΒσυνΒ) 2 =_!_

ΒΓ · (ΑΓ · ΑΓ + ΑΒ · ΑΒ ) 2 ΒΓ ΒΓ = �ΒΓ (ΑΓ2 + ΑΒ2 ) = �ΒΓ2 •

Οδηγός σκέψης.

i . Δύο τρίγωνα με κοινή βάση και ίσα ύψη είναι ισοδύναμα.

i i . Ο τριγωνομετρικός τύπος υπολογισμού του εμ-βαδού τριγώνου Ε =

_!_βγημΑ . 2

Άσκηση 7η Δίνεται ορθογώνιο ΑΒΓ Δ με πλευρές ΑΒ = 6cm και ΑΔ = 4cm. Α ν Ε σημείο της ΑΒ με

ΒΕ = .!.. ΑΒ και Ζ το σημείο τομής της ΓΕ με 3

την ΒΔ, να υπολογίσετε το εμβαδόν του τετρα­πλεύρου ΑΕΖΔ. Λύση

Τα τρίγωνα ΖΔΓ και ΖΕΒ είναι όμοια (ΔΓ//ΕΒ), οπότε και για τα ύψη τους θα ισχύει

υ2 ΕΒ . υ2 2 Ι - = - η - = - = - . υ1 ΔΓ υ1 6 3

τ . υ2 1 . υ2 1 • 1 οτε = -- η - = - η υ2 = cm. υ1 + υ2 3 + 1 4 4 Η ισότητα (ΑΕΖΔ) = (ΑΒΔ) - (ΖΕΒ) δίνει το γι-νόμενο (ΑΕΖΔ) =

_!_6 · 4 -

_!_ 2 · 1 ή 2 2

(ΑΕΖΔ) = l lcm2 . Οδηγός σκέψης.

Α ν δύο τρίγωνα είναι όμοια τότε ο λόγος των ομό­λογων υψών είναι ίσος με τον λόγο ομοιότητας. Άσκηση 8η Έστω τρίγωνο ΑΒΓ και ευθεία παράλληλη προς τη ΒΓ που τέμνει τις πλευρές ΑΒ και ΑΓ στο Δ και Ε αντί

dτοιχα. Να αποδείξετε ότι

(ΑΒΕ)2 = (ΑΔΕ) · (ΑΒΓ) .

Λύση Α

Τα τρίγωνα ΑΒΕ και ΑΔΕ έχουν κοινή γωνία των Α, επομένως θα είναι

(ΑΒΕ) = ΑΒ · ΑΕ . (ΑΒΕ) = ΑΒ ( Ι ) . (ΑΔΕ) ΑΔ · ΑΕ η (ΑΔΕ) ΑΔ Για τον ίδιο λόγο για τα τρίγωνα ΑΒΓ και ΑΒΕ θα ισχύει

(ΑΒΓ) ΑΒ · ΑΓ . (ΑΒΓ) ΑΓ (ΑΒΕ) = ΑΒ · ΑΕ η (ΑΒΕ) = ΑΕ

Επειδή όμως ΔΕ//ΒΓ θα ισχύει ΑΒ = ΑΓ (3). ΑΔ ΑΕ

(2).

Από τις σχέσεις (1 ), (2) και (3) καταλήγουμε στην (ΑΒΕ) = (ΑΒΓ) ' (ΑΒΕ)2 = (ΑΒΓ) · (ΑΔΕ) . (ΑΔΕ) (ΑΒΕ) η

Οδηγός σκέψης.

Σε δύο τρίγωνα με κοινή γωνία ο λόγος των εμβα­δών ισούται με τον λόγο των γινομένων των πλευ­ρών που περιέχουν την γωνία. Άσκηση 9η . Δίνεται τρίγωνο ΑΒΓ. Από το μέσο Μ της ΑΒ φέρουμε ευθεία (ε) που τέμνει τις ΑΓ και ΓΒ στα Δ και Ε αντίστοιχα. Η παράλληλη από το Γ προς της (ε) τέμνει την προέκταση της ΑΒ στο Ζ. Ν α αποδείξετε ότι: i . (ΕΒΖ) = (ΜΒΓ) ίίί . (ΖΒΕ) = (ΖΑΔ) Λύση

i i . (ΜΔΖ) = (ΜΔΓ)

i . Επειδή τα τρίγωνα ΖΜΕ και ΓΜΕ έχουν κοινή βάση ΕΜ και οι κορυφές τους Ζ και Γ βρίσκο­νται σε ευθεία παράλληλη προς την κοινή βάση είναι ισοδύναμα, δηλαδή (ΖΕΜ) = (ΓΜΕ) ή (ΖΕΜ) - (ΒΕΜ) = (ΓΜΕ) - (ΒΕΜ) ή (ΖΕΒ) = (ΓΒΜ) ( 1 ).

ΕΥΚΛΕΙΔΗΣ Β ' λθ ' τ2/44

Page 47: Ευκλειδης Β 58

Μαθηματικά Β' Λυκείου

ΚrΕ + rn = ΚrE + Δzr = 90° . Άρα ΕΚΓ = 90° .

i i . Στο ορθογώνιο τρίγωνο ΕΓΒ το ΓΚ είναι ύψος, . θ . Ι Ι 1 . αρα α ισχυει ΓΚ2 = ΕΓ2 + ΓΒ2 η Ι 1 1 . 2 2 - = - + - η χ = 500cm 1 00 χ2 χ2

4 ή (ΑΒΓ Δ) = 500cm2 όπου χ η πλευρά του τετραγώνου.

ii. Για τον ίδιο λόγο τα τρίγωνα ΖΜΔ και ΓΜΔ εί- Άλλος τρόπος.

ναι ισοδύναμα άρα Επειδή τα τρίγωνα ΓΚΕ και Γ ΔΖ έχουν κοινή γω-(ΖΜΔ) = (ΓΜΔ) (2) .

i i i . Επειδή ΓΜ διάμεσος θα είναι (ΓΒΜ) = (Γ ΑΜ) οπότε η ( 1 ) γίνεται (ΖΕΒ) = (Γ ΑΜ) ή (ΖΕΒ) = (ΓΜΔ) + (ΜΑΔ) ή

(2 ) (ΖΕΒ) = (ΖΜΔ) + (ΜΑΔ) ή (ΖΕΔ) = (ΖΑΔ) .

Οδηγός σκέψης. Δύο τρίγωνα που έχουν κοινή βάση και οι κορυφές τους είναι σε ευθεία παράλληλη προς τη βάση εί­ναι ισοδύναμα. Άσκηση Ι Οη Δίνεται τετράγωνο ΑΒΓ Δ και Ε, Ζ τα μέσα των πλευρών ΔΓ, ΑΔ αντίστοιχα. Αν Κ είναι το ση­μείο τομής των τμημάτων ΒΕ και ΓΖ με ΚΓ = 10 cm τότε: i. Να αποδείξετε ότι ΒκΓ = 90° ii . Να υπολογίσετε το εμβαδόν του ΑΒΓΔ. Λύση

ί. Τα τρίγωνα ΓΖΔ και ΒΕΓ είναι ίσα γιατί Δ = Γ = 90°, ΖΔ = ΕΓ ως μισά ίσων πλευρών ΔΓ = ΒΓ άρα και οι αντίστοιχες γωνίες τους θα είναι ί---- --- --- ---σες, δηλαδή ΕΒΓ = ΖΓ Δ και ΒΕΓ = ΓΖΔ . Στο τρίγωνο ΓΕΚ είναι

νία θα ισχύει: χ

(ΓΚΕ) ΓΚ ΤΕ . (ΓΚΕ) 1 Ο . 2

(ΓΔΖ) = ΓΖ · ΓΔ η (ΓΔΖ) = x-J5 -- · Χ 2

. (ΓΚΕ) = � ( 1 ) . η (ΓΔΖ) x-J5

(γιατί;)

Τα τρίγωνα αυτά όμως είναι και όμοια, άρα θα ισ . ει: (ΓΚΕ) = ( ΓΚ )2 • (ΓΚΕ) = 1 00

χυ (ΓΔΖ) ΓΔ η (ΓΔΖ) χ2 (2) .

Α . ( 1 ) (2) . 1 ο 1 00 . πο και παιρνουμε r; = -2 η χν5 χ χ = 1 0-JS ή χ2 = 500cm2 ή (ΑΒΓΔ) = 500cm2 . Οδηγός σκέψης.

Σ ' θ θ . . . Ι Ι Ι ε κα ε ο ρ ογωνιο τριγωνο ισχυει -2 = 2 + -2 . υα β γ

Άσκηση t l η Θεωρούμε δύο οξυγώνια τρίγωνα ΑΒΓ και ΜΝΡ με παράλληλες πλευρές, όπως στο σχήμα. Συμβολίζουμε με Ει, Ε2 και Ε3 τα εμβαδά των τριγώνων ΑΔΖ, ΒΚΙ και ΓΗΘ αντίστοιχα και με S1 , S2 και S3 τα εμβαδά των ΡΘΙ, ΝΖΗ και ΜΔΚ αντίστοιχα. Α ν Ε και S τα εμβαδά των ΑΒΓ και ΜΝΡ αντίστοιχα να αποδείξετε ότι: i. Τα τρίγωνα ΑΔΖ και ΜΝΡ είναι όμοια, ο­

μοίως και ΑΒΓ και ΡΙΘ. JS ΜΝ JE ΒΓ ii . .JE: = ΔΖ

και JS: = ΙΘ

iii . JS = JS: + JE. + ,Js; και

JE = JE: + Fι + JE: .

Ε ΥΚ.\ΕΒ.ΗΣ Β ' ί.θ ' τ2/45

Page 48: Ευκλειδης Β 58

Μαθηματικά Β' Λυκείου

ίν . .JS + JE =

= JS: + $: + $: + JE. + .JE: + JE:. Λύση

Α

Ρ

ί . Τα τρίγωνα ΑΔΖ και ΜΝΡ έχουν πλευρές πα­ράλληλες άρα είναι όμοια. Όμοια αποδεικνύε­ται και η δεύτερη ομοιότητα.

ίί . Επειδή τα τρίγωνα ΑΔΖ και ΜΝΡ είναι όμοια θα έχουμε

S (ΜΝ )2 • JS ΜΝ Ε = ΔΖ η JE: = ΔΖ .

Όμοια επειδή τα τρίγωνα ΑΒΓ και ΡΙΘ είναι όμοια προκύπτει η δεύτερη ισότητα.

iii . Από το i i) έχουμε διαδοχικά: .JE = ΒΓ = ΒΙ + ΙΘ + ΘΓ = ΒΙ + Ι + ΘΓ ( 1 ) .jS; ΙΘ ΙΘ ΙΘ ΙΘ Επειδή το τρίγωνο ΒΚΙ είναι όμοιο με το ΡΙΘ και το ΡΙΘ όμοιο προς το ΘΗΓ έχουν αντίστοιχα

ΒΙ .jE; ΘΓ JF:; - = -- και - = -- (2). ΙΘ .jS; ΙΘ .jS; Από ( 1 ) και (2) προκύπτει: .JE .JE; + .JS: + JE; . - η .JS: .JS: .JE = .JE; + .JS: + JE; . Για την απόδειξη της άλλης ισότητας έχουμε αντίστοιχα. JS ΜΝ ΜΔ + ΔΖ + ΖΝ Jf; = ΔΖ = ΔΖ

= ΜΔ + 1 + ΖΝ = .jS; + 1 + Fz ΔΖ ΔΖ Jf; Jf; από την οποία προκύπτει ότι

JS = .}S; + JE: + Fz . ίν. Προκύπτει με πρόσθεση των σχέσεων του ερω­

τήματος iii) κατά μέλη .

Οδηγός σκέψης.

Α ν δύο τρίγωνα είναι όμοια ο λόγος των εμβαδών τους ισούται με το τετράγωνο του λόγου ομοιότητας.

Προτεινόμενες ασκήσεις 1. Δίνεται παραλληλόγραμμο ΑΒΓ Δ και Ο τυ­

χαίο σημείο στο εσωτερικό του. Να δείξετε τα παρακάτω: ί. (ΑΟΒ)+(ΓΟΔ) = (ΑΟΔ)+(ΒΟΓ) ίί. (ΒΟΔ) = ICAOB) - (ΒΟΓ)I ίίί. Α ν Ο είναι εξωτερικό του παραλληλο­

γράμμου να αποδείξετε ότι (ΟΒΔ) = (ΟΒΓ)+(ΟΑΒ). -

2. Δίνεται ορθή γωνία ΒΑΓ με μήκη πλευρά ΑΒ = 9cm και ΑΓ = I 4cm. Στην ΑΓ θεωρού­με σημείο Ε ώστε ΑΕ = 6cm. Από το Γ φέρ­νουμε παράλληλη προς την ΒΕ που τέμνει την παράλληλη προς την ΑΒ από το Ε στο Δ. Να βρείτε το εμβαδόν του ΒΕΔ .

(Απαντ. 36cm2) •

3. Δίνεται παραλληλόγραμμο ΑΒΓ Δ και Ε, Ζ τα μέσα των ΒΓ και Γ Δ αντίστοιχα. Α ν Ο το κέ­ντρο του παραλληλογράμμου και Λ το σημείο τομής της ΔΒ με την ΑΕ, να βρεθεί το εμβα­δόν του ΑΒΓ Δ αν είναι (ΑΟΛ) = 3 τ.μ.

(Απαντ. 36 τ.μ.) 4. Δίνεται ορθογώνιο ΑΒΓ Δ με ΑΔ = Ι Ocm και

ΑΒ> ΑΔ. Πάνω στην ΑΒ παίρνουμε τμήμα ΝΜ = 4cm και στην ΓΔ τμήμα ΚΛ = 6cm. Αν Ρ το σημείο τομής των ΚΜ και ΝΛ (Ρ εσωτε­ρικό του ΑΒΓΔ) να υπολογίσετε το (ΝΡΜ).

(Απαντ. 8 cm2) . 5. Δίνεται το τετράγωνο ΑΒΓ Δ πλευράς 8 cm.

Α ν Ε σημείο της ΑΒ με ΑΕ = ΑΒ και Ζ ση-4 μείο της ΕΓ με ΕΖ = ΕΓ να βρείτε το εμβα-

8 δόν του ΑΕΖΔ.

(Απαντ. Ι 2 cm2) .

ΕΥΚΛΕΙΔΗΣ Β ' λθ ' τ2/46

Page 49: Ευκλειδης Β 58

Μαθηματικά Β ' Λυκείου

r Ασκήσεις στην εuθεία και στον κύκλο

Άσκηση 1 Δίνονται τα μη συγγραμμικά και μη μηδενικά

διανύσματα α, β με (α: β) = π το διάνυσμα 4

γ = λii + (λ - 1 )β, λ Ε IR και η εξίσωση.

(ε) : α rx + βγχ + (α + β)y = ο

Να αποδείξετε ότι: ί) η (ε) παριστάνει ένα σύνολο ευθειών. ίί) Όλες οι ευθείες (ε) διέρχονται από ένα στα­

θερό σημείο το οποίο να βρεθεί. Λύση i) Η εξίσωση (ε) παριστά ευθείες όταν ίi · γ και

β · γ δεν μηδενίζονται συγχρόνως. Αν δεχθούμε ότι είναι ίi · γ = Ο και β · γ = Ο τότε: 1) Αν γ :;t: Ο είναι ίi .l γ και β .l γ οπότε β ιι α .

Άτοπο, αφού τα διανύσματα ίi, β είναι μη συγγραμμικά.

2) Αν γ = Ο δηλ. : λίi + (λ - l)β = Ο , επειδή εί-ναι ίi 11 β , ισχύει λ = Ο και λ - Ι = Ο, άτοπο. Άρα, οι ποσότητες ίi · γ και β · γ δεν μηδε­νίζονται ταυτόχρονα οπότε η (ε) παριστά ένα σύνολο ευθειών.

ii) Θέλουμε να δείξουμε ότι οι ευθείες της μορφής (ε) διέρχονται από σταθερό σημείο. Δίνουμε στο λ δύο τυχαίες τιμές, έστω λ = Ο και λ = Ι . Τότε α) για λ = Ο είναι γ = -β και η (ε) γράφεται

ίi( -β)χ + β( -β)y + (ίi + β)( -β) = ο - - -2 -- -2 α · β · χ + β · y = -αβ - β ( 1 )

β) για λ = Ι είναι γ = ίi . και η (ε) γράφεται ίi . ίiχ + β . ίiy + (ίi + β)ίi = ο -2 - -β -β- -2 α χ + α · y = -α - α (2)

Λύνουμε το σύστημα των ( 1 ), (2) και έχουμε: D - ίi . β β2 - (- . β-)2 - - 2 β-2 -- - α α · --2 - β-α α ·

Αθ. Καραγεώργου

( ια ι · Ι β iσυβ�)2 - ιαι2 Ιβ 12 = Ξιαι2 · Ιβ 12 - ιαι2 · Ιβ 12 = -Ξ ιαι2 · Ιβ 12 * ο

-αβ - β2 β2 - - 1 , _ ,2 ι- ι2 Dx = - 2 - • . • - - α . β -ίiβ - ίi ίi . β 2

ο = α · β -�·! --β22 = . . . = .!_ιαι2 · Ιβ 12 Υ ίi2 -αβ - α 2

Ά _ Dx 1 Dy Ι δ λαδ ' ρα χ-- = - και y = - = - η η το D D κοινό σημείο των δύο ευθειών είναι το Κ(- Ι , - Ι ). Θα εξετάσουμε αν το σημείο αυτό επα­ληθεύει τις ευθείες (ε) για κάθε χ Ε IR . Έχουμε: α · γ( - Ι) + β · ΎC - 1) + (ίi + β)γ = = -ίi . γ - β . γ + ίi . γ + β . γ = ο Τελικά, όλες οι ευθείες (ε) διέρχονται από το σημείο Κ(- Ι , - Ι )

Άσκηση 2

Δίνονται τα διανύσματα α, β και οι ευθείες:

(ε, ) : α · βχ + (α · β - t)y = 2α . β

( ε2 ) : (α . β + 1 )χ + α . βy = 2α . β + 1

Ι . Να αποδείξετε ότι οι ευθείες (ε1), (ε2) τέ­μνονται σε κάθε περίπτωση.

2. Να βρείτε τον γεωμετρικό τόπο των σημεί­ων τομής τους.

3. Αν είναι α :;t: Ο, β :;t: ο ' να αποδείξετε την ι-

σοδυναμία

Λύση 1 . Λύνουμε το σύστημα των (ε1 ), (ε2) .

Έχουμε α . β D = ίi · β + 1

ίi · β - 1 = α . β = (ίi · β )2 - (α · β + Ι ) (α . β - Ι ) =

ΕΥΚΛΕΙΔΗΣ Β ' λθ ' τ2/47

Page 50: Ευκλειδης Β 58

Μαθηματικά Β' Λυκείου

2<i · β a . β - Ι -D = χ -2<i · β + Ι - = <i · β + Ι

<i · β a . β 2a . β _ -

DY

= _ _ = -α · β a . β + Ι 2a . β + Ι

Έτσι, έχουμε για οποιαδήποτε διανύσματα a, β ότι

Dx _ - DY _ -χ = 0 = α · β + 1 , y = 0 = -α · β

οπότε οι ευθείες (ε1 ), (ε2) τέμνονται σε κάθε περί­πτωση, στο σημείο Ρ ( a · β + Ι , -a · β)

2 . Για τον γεωμετρικό τόπο των σημείων τομής Ρ έχουμε

Yp = -<i · β άρα: Xp = -yp + 1 <=> X p + ΥΡ - 1 = Ο οπότε ο γ.τ. των σημείων τομής Ρ είναι η ευθεία

(ε) : χ + y - 1 = 0 3. Θεωρούμε τα παράλληλα διανύσματα h1 , fi2 προς τις ευθείες (ε Ι ), (ε2) αντίστοιχα

Είναι h 1 = (<i · β - 1 , -a . β) και

h2 = (<i · β, -a . β - Ι) , οπότε ε1 .l ε2 <=> h1 .l h2 <=> h1 · h2 = Ο <=> <=> ( a . β - Ι ) a . _β + ( -a . β) ( -a . β - Ι ) = ο

<=> (<i · β )z - <i · β + (<i · β)z + (<i · β) = Ο

<=> 2 (a · β)2 = ο<=> a . β = ο <=> a .l β Άσκηση 3

Δίνονται οι ευθείες (ε. ) : α . β χ + β . γy + a . γ = ο

(ε2 ) : « · προΡαίiχ + ii · προΡμΎΥ + r . προβ1« + ι = ο

όπου a, β ,γ είναι διανύσματα μη μηδενικά και ανα δύο μη συγγραμμικά. Ι . Να αποδείξετε ότι ει // ε2• 2 . Να υπολογίσετε την απόσταση των ε1 , ε2•

3. Αν τα διανύσματα α, β ,γ είναι μοναδιαία, να εξετάσετε αν μπορεί η απόσταση των

δ ' θ ' ' ' J2 υο ευ ειων να ειναι ιση με Τ Λύση I . Γνωρίζουμε ότι ισχύουν:

a · β = a · προ β α β β · γ = β · προββγ a . γ = γ . προβγ<i Έτσι, η εξίσωση (ε2) γράφεται ισοδύναμα

a . β χ + β . γy + γ . a + Ι = ο Παρατηρούμε ότι οι συντελεστές των χ, y στις εξισώσεις των ε1 ,ε2 είναι ίσοι, οπότε οι ευθείες ε1 , ε2 είναι παράλληλες.

2 . Το σημείο κ[ο, - � ·�) ανήκει στην ε1 . Συνε­β · γ

�( a . β)z + (β . γ )2 3. Είναι: ιa ι = j β j = lγl = I

Λ

( 1 )

Λ

υποθέτουμε ότι (<i, β) = φ και (β, γ) = ω . Δεχό­μαστε ότι η απόσταση των δύο ευθειών είναι ' J2 I Α ' . ( Ι ) ' ιση με Τ = J2 . πο την σχεση , εχουμε:

Ι Ι �(<i · β)2 + (β . γ)2

J2

<=> (α . β )2 + (β . γ )z = 2 <=>

<=> ( l<i l · jβ jσυνφ)2 + ( j β j . Ιγl συνω)2 = 2 <=> συν2φ + συν2ω = 2 (2) Η σχέση (2) ισχύει μόνο όταν συν2φ=συν2ω= Ι δηλαδή όταν φ = Ο ή φ = π και ω = Ο ή ω = π οπότε θα είναι α ιι β και β ;ι γ ' άτοπο, αφου τα a, β, γ είναι μη συγγραμμικά ανά δύο.

Άσκηση 4 ΕΥΚΛΕΙΔΗΣ Β ' λθ ' τ2/48

Page 51: Ευκλειδης Β 58

Μαθηματικά Β · Λυκείου

Δίνεται ορθογώνιο και ισοσκελές τρίγωνο ΟΑΒ Α

με ΑΟΒ = 90° .Θεωρούμε τυχαίο σημείο Μ της πλευράς ΑΒ και ονομάζουμε Μ1, Μ2 τις ορθές προβολές του Μ πάνω στις ΟΑ, ΟΒ αντίστοιχα. Να βρείτε τον γεωμετρικό τόπο του μέσου Κ του ευθύγραμμου τμήματος Μ1Μ2. Λύση

Υ

Β(Ο,α)

- - - - - - - - - Μ(χο,Υο)

Α( α, Ο) χ

Τοποθετούμε το τρίγωνο ΟΑΒ σε ορθοκανονι­κό σύστημα αξόνων με αρχή την κορυφή Ο, την ΟΑ στον θετικό ημιάξονα των χ και την ΟΒ στον θετικό ημιάξονα των y. Αν Α(α, 0), επειδή το τρίγωνο είναι ισοσκελές, έχουμε Β(Ο, α) . Αν Μ(χο, Υο) τότε Μ1 ( χ0 , 0) , M2 (0, y0 ) . Επειδή το σημείο Μ ανήκει στην ευθεία ΑΒ, που έχει εξίσωση χ + y = α, ισχύει

Χο + Υο = α <=> Υο = α - χσ ( I ) Λόγω της ( 1 ), είναι Μ 2 ( Ο, α - χ0 ) το μέσο Κ του ΜιΜz είναι κ( Χ2ο ' α -

2Χο )

για τον γ.τ. του Κ έχουμε: χ0 α - χ0 Χ κ = 2 και Υκ = -2-

α - 2χκ δηλ: χ0 = 2χκ και Υ κ = ----'-'-2 οπότε: 2yκ = α - 2χκ <::> 2χκ + 2yκ = α <=> Χ κ + Υ κ = � και το μέσο Κ του ΜιΜz ικα-2

. ξ' θ , α νοποιει την ε ισωση της ευ ειας χ + y = - , χ > 2 Ο, y > Ο. Τελικά, ο ζητούμενος γ.τ. είναι το ευ­θύγραμμο τμήμα που συνδέει τα μέσα Ρ, Σ των ΟΑ, ΟΒ αντίστοιχα.

Υ

Β

Α χ

Άσκηση 5 Δίνονται τα μη συγγραμμικά ανά δύο διανύσμα­τα α,β ,γ και οι ευθείες: (ε, ) : χ + y - 2 + α · β (y - χ) = ο ( ε2 ) : χ + Υ + β · γ (χ - Υ - 1 ) = Ο

Α ν οι ευθείες ταυτίζονται να αποδείξετε ότι β .L {α + γ) .

Λύση Γ ράφουμε τις εξισώσεις των ευθειών στην μορφή Αχ + By + Γ = Ο . Έχουμε (ε , ) : ( ι - α - β ) χ + ( ι + α . β ) y - 2 = ο ( ε2 ) : ( ι + β · γ ) χ + ( ι - βγ ) y - βγ = Ο Επειδή οι ευθείες ταυτίζονται υπάρχει ρ ε IR τέτοιο, ώστε 1 - α - β = ρ ( ι + β - γ) 1 - α - β = ρ ( 1 - β - γ) -2 = -ρβ - γ

( 1 ) (2) (3)

Με πρόσθεση των ( 1 ), (2) παίρνουμε: 2 = 2ρ <=> ρ = 1

για ρ = ι η (3) δίνει: β · γ = 2 και η ( 1 ), λόγω της ( 4 ), γίνεται

(4)

1 - α . β = 1 + β . γ <=> ι - α . β = ι + 2 <=> α . β = -2 (5) Με πρόσθεση των (4), (5) έχουμε α . β + β · γ = ο <=> β (α + γ) = ο <=> β .L (α + γ)

Άσκηση 6 Δίνονται τα διανύσματα

α, β,γ με ια ι = ιγ ι = 2 και ι β ι = 1 . Αν η εξίσωση

c : χ2 + y 2 + α . β . χ + β . γy + α . γ = ο

ΕΥΚΛΕΙΔΗΣ Β ' λθ' τ2/49

Page 52: Ευκλειδης Β 58

Μαθηματικά Β' Λυκείου

παριστάνει κύκλο, να αποδείξετε ότι η γωνία των διανυσμάτων ii, γ βρίσκεται στο διάστημα

Λύση Επειδή η εξίσωση (C) παριστάνει κύκλο, ισχύει

(ii . β)2 + (β . y)2 - 4ii . γ > ο

<=> ( Ιa Ι Ιβ l συν(a]))2 + ( Ιβ1 Ιr l συν(�))2

-4 lii l lr l συν([.?) > Ο <=> 2συν2 (a]) +

+2συν2 (�) - 4 · J2 · J2συν([.?) > Ο

<=> 8συν([.?) < 2συν2 (a)) + 2συν2 (�)

� 2 · Ι + 2 · Ι = 4 <::> συν([.?) < _!_ (Ι) 2 -Επειδή (ii, Ύ) Ε [Ο, π) , από την ( Ι ), προκύπτει ότι

([.?) Ε (� , Π) . ,

ο π

Άσκηση 7 Η ευθεία (ε) : y = 2χ τέμνει ένα κύκλο (C) κέ-ντρου Κ(3,1) στα σημεία Α, Β ώστε το εμβαδόν

του τριγώνου ΑΚΒ θα είναι το 1 r,; του εμβα-

3ν5 δού του κυκλικού δίσκου. Να βρείτε την εξίσω­ση του κύκλου. Λί>ση Η ζητούμενη εξίσωση είναι της μορφής:

(C) : (x - 3)2 + (y - I)2 = R2 ( 1 ) Αρκεί να βρεθεί η ακτίνα R του κύκλου.

χ

Το ύψος ΚΜ του τριγώνου ΑΚΒ είναι ίσο με την απόσταση του κέντρου Κ από την ευθεία (ε).

1 1 - 2 · 3 + Ι Ι · 5 � Δηλ. (ΚΜ) = d(Κ, ε) = = - = ν5 . J( -2)2 + Ι2 J5 Από το πυθαγόρειο θεώρημα στο τρίγωνο ΚΜΒ παίρνουμε (ΜΒ) = JR 2 - (ΚΜ)2 = JR 2 - 5 οπότε (ΑΒ) = 2(ΜΒ) = 2JR 2 - 5 και το εμβαδόν του τριγώνου είναι: (ΑΚΒ) = _!_ · (ΑΒ) · (ΚΜ) = 2 = _!_ · 2JR2 - 5 . JS = J5 .JR2 - 5 . 2 Επειδή το εμβαδόν του κυκλικού δίσκου είναι ίσο με πR 2 , σύμφωνα με το πρόβλημα, έχουμε: J5 .JR2 - 5 = -Ι-πR2 <::> I 5JR2 - 5 = πR2

3J5 <::> 225(R2 - 5) = π2R 4

<::> π2R 4 - 225R 2 + 5 · 225 = Ο (2) για χ = R 2 > 5 η (2) γράφεται

π2χ2 - 225χ + 5 · 225 = 0 (3) με Δχ = ( -225)2 - 4 · π2 · 5 · 225 = 225(225 - 20π2 ) = (1 5J 225 - 20π2 )2 > ο και ρίζες

225 ± I 5J225 - 20π2 Χ ι .2 =

που είναι δέκτες και οι δύο. Άρα, υπάρχουν δύο κύκλοι με την ιδιότητα του προβλήματος, με ακτίνες

R 1 2 = I ι;: JΓ22_5_±_1_5�r:2=2=5 =_ =20=π=2

· πν2

Άσκηση 8

Η ευθεία (ε) : χ - y - 3 = Ο τέμνει τον κύκλο (C) : χ2 + y2 - 2αχ - 4αy = Ο στα σημεία Α, Β. Να βρείτε την τιμή του α Ε IR ώστε να ισχύει - -ΟΑ · ΟΒ = Ο , όπου Ο η αρχή των αξόνων.

ΕΥΚΛΕΙΔΗΣ Β' λθ ' τ2/50

Page 53: Ευκλειδης Β 58

Μαθηματικά Β ' Λυκείου

Αίιση Επειδή το 0(0, Ο) επαληθεύει την εξίσωση του κύ­κλου, ο κύκλος (C) διέρχεται από την αρχή των αξόνων. Επειδή ΟΑ · ΟΒ = Ο είναι ΟΑ .l ΟΒ και -συνεπώς ΑΟΒ = 90' . Άρα η ΑΒ είναι διάμετρος του κύκλου οπότε διέρχεται από το κέντρο του Κ( α, 2α) . Επειδή η ΑΒ Ε (ε) και Κ Ε ΑΒ είναι και Κ Ε (ε) οπότε οι συντεταγμένες του Κ επαληθεύ­ουν την εξίσωση της (ε). Δηλ. α - 2α - 3 = Ο και τελικά: α = - 3 .

Υ.

-3 .fJ" (ε) χ

Άσκη ση 9 Δίνεται η εξίωση εκ: (1 + κ )χ + (Κ - ι )y + ι + κ = ο, κ Ε JR. •

Ι . Ν α αποδείξετε ότι: α. Η εκ παριστάνει ευθεία για κάθε κ Ε IR. •

β. Οι ευθείες εκ διέρχονται από σταθερό σημείο Ρ το οποίο να βρείτε.

2. Να προσδιορίσετε την εξίσωση του κύκλου C που εφάπτεται στον άξονα y Ύ και έχει κέ­ντρο το σημείο Ρ.

Λύση I . α. Επειδή οι συντελεστές 1 + κ, κ - 1 των χ, y

αντίστοιχα δεν μηδενίζονται ταυτόχρονα η εκ παριστάνει ευθεία για κάθε κ Ε IR. .

β. Στην εξίσωση εκ δίνουμε στον κ δύο τυχαίες τιμές έστω κ = 1 και κ = -1 . Έχουμε: ε1 : 2χ + 2 = Ο <::::::> χ + 1 = Ο <::::::> χ = - 1 ε2 : -2y = O <:=:> y = O Άρα, το κοινό σημείο των ε1 , ε2 είναι το Ρ( -1 , Ο) θα δείξουμε ότι από το Ρ διέρχονται όλες οι ευθείες εκ. Πράγματι, έχουμε:

(1 + κ) · ( - 1) + (κ - 1) · Ο + 1 + κ = = - 1 - Κ + 1 + Κ = Ο Τελικά, όλες οι ευθείες εκ διέρχονται από το σταθερό σημείο Ρ( - 1 , 0).

2. Το κέντρο του ζητούμενου κύκλου C είναι το Ρ(-1 , 0). Επειδή ο κύκλος εφάπτεται στον y 'y είναι R = d(P, y'y) = l- 1 1 = 1 . Τελικά, η ζητούμενη εξίσωση είναι:

(χ + 1 )2 + y2 = 1 . Άσκηση 1 0 Να βρείτε την μικρότερη απόσταση του σημείου Α(4, 4) από τον κύκλο C: xz + yz - 2χ = Ο .

ΑίJση Το κέντρο του κύκλου είναι το σημείο Κ (-�2 , -% J = (1, Ο) και η ακτίνα του είναι ίση με

1 Ι 2 2 1 R = 2ν (-2) + Ο - 4 · 0 = 2 · 2 = 1 .

Επειδή (KA) = J(xA - Χ κ )2 + (yA - yκ )2 =

= Jc 4 - 1)2 + c 4 - ο)2 = J9 + 1 6 = 5 > 1 = R

Το σημείο Α είναι εξωτερικό του κύκλου C. Φέ­ρουμε την ΑΚ και ονομάζουμε Μ το πρώτο σημείο τομής της με τον κύκλο. Α ν Λ είναι τυχαίο σημείο του κύκλου τότε από την τριγωνική ανισότητα στο τρίγωνο ΑΚΛ έχουμε:

(ΑΚ) ::; (ΑΛ) + (ΚΛ) <::::::> (ΑΜ) + (ΜΚ) ::; (ΑΛ) + (ΚΛ) <::::::> (ΑΜ) + R ::; (ΑΛ) + R <::::::> (ΑΜ) ::; (ΑΛ)

Υ. - - - - - - - - - - - - - - - - - - - - - , Α(4,4)

' ' ' ' ' ' ' ' ' ' ' ' ' ' ' ' ' ' '

χ

Άρα, η μικρότερη απόσταση του σημείου Α από τον κύκλο είναι η (ΑΜ), οπότε

(ΑΜ) = (ΑΚ) - (ΚΜ) = (ΑΚ) - R = 5 - 1 = 4

ΕΥΚΛΕΙΔΗΣ Β ' λθ ' τ2/51

Page 54: Ευκλειδης Β 58

� ......... . ,_ �μ r ��lι �-ιι Αιιrι-1•

ΣΥΝΑΡ1ΗΣΕΙΣ Ευάγγελος Ευσταθίου, Δημήτρης Αργυράκης, Μεντής Κώστας

Χ Ρ Η Σ Ι Μ Ε Σ Θ Ε Ω Ρ Η τ Ι ΚΕΣ ΓΝΩΣΕ Ι Σ

Ι . Για μια συνάρτηση f: Α � 1R ισχύουν: α. Αν η f είναι « 1 - 1 » τότε f(α) = f(β) ς::} α = β ,

α, β ε Α . β. Η f είναι αντιστρέψιμη 6 Η f είναι « 1 - 1 » . γ. Αν η f είναι αντιστρέψιμη τότε:

• Πεδίο ορισμού της f- 1 είναι το f(A). • Σύνολο τιμών της f- 1 είναι το Α. • Για κάθε χ ε Α ισχύει:

f(x) = y ς::} f- 1 (y) = χ οπότε f(f-1 (y)) = y 'v'y ε f(A) και

2. Π ροτάσεις

f-1 (f(x)) = χ 'v'x ε Α

Έστω συνάρτηση f: Α � 1R . α. Α ν η f είναι γνησίως μονότονη στο Α, τότε

η f είναι «1-1 >>. Σημείωση :

Το αντίστροφο δεν ισχύει. Για παράδειγμα η συνάρτηση f (χ) = ..!._ είναι « 1 - 1 », αλλά χ δεν είναι γνησίως μονότονη στο 1R * .

β. Α ν η f είναι γνησίως μονότονη στο Α, τότε και η συνάρτηση Γ1 είναι γνησίως μονό­τονη στο f(A) με το ίδιο είδος μονοτονίας. (Απόδειξη για άσκηση). Για παράδειγμα η συνάρτηση f(x) = ex είναι γνησίως αύξου-σα στο 1R , οπότε και η f- 1 (χ) = ln χ είναι γνησίως αύξουσα στο f(JR) = (0, +οο) .

γ. • Α ν η f είναι άρτια, τότε η f δεν είναι 1 - 1 . • Α ν η f είναι περιττή, τότε η f μπορεί να

είναι « 1 - 1 » (π. χ. f (χ) = ..!._ ), αλλά μπορεί χ

και να μην είναι « 1 - 1 » (π.χ. f(x) = ημχ , π 5π 1 αφου ημ- = ημ- = - ). 6 6 2

δ. Α ν η f είναι αντιστρέψιμη και περιττή, τότε και η Γ1 είναι περιττή. Απόδειξη

Από την υπόθεση η f είναι αντιστρέψιμη τό­τε για κάθε y ε f(A) υπάρχει μοναδικό χ ε Α ώστε f(x) = y (1 ) . Επίσης η f είναι περιττή, τότε για κάθε χ ε Α θα είναι και το

( I ) -χ ε Α , f( -χ) = -f(x) = - y (2). Επειδή f( -χ) ε f(A) . Από την (2) προκύπτει -y ε f(A) . Έτσι Γ1 ( -y) = r-l (f( -χ)) = (Γ1 ο f)( -χ) = -χ (3) Αλλά ισχύει: f(x) = y ς::} χ = f- 1 (y) τότε από την (3) προκύπτει ότι r-l ( -y) = -f- 1 (y). Πράγματι η f- 1 είναι περιττή .

Άμεσες συνέπειες των προηγούμενων προτά­σεων είναι: • Α ν η f δεν είναι αντιστρέψιμη τότε η f δεν

είναι «1-1». • Α ν η f δεν είναι αντιστρέψιμη, τότε η f

δεν είναι γνησίως μονότονη. • Άρα η f είναι «1-1» τότε η f δεν είναι άρτια. • Α ν η f είναι λ στο Α, τότε και η Γ1 είναι

λ στο f(A) και αν η f είναι � στο Α, τότε

και η Γ1 είναι � στο f(A). 3. Γραφικές παραστάσεις των f και Γ1

Α ν η συνάρτηση f είναι αντιστρέψιμη τότε α. Cr , CΓ, είναι συμμετρικές ως προς τη δι-

χοτόμο y = χ. Άρα: Μ(α, β) ε Cr ς::} Ν(β, α) ε Cr-,

ΕΥΚΛΕΙΔΗΣ Β' λθ' τ.2/52

Page 55: Ευκλειδης Β 58

Μαθηματικά για την Γ Λυκείου

β . Τα κοινά σημεία των Cr , CΓ, εφόσον υ­

πάρχουν έχουν συντεταγμένες που προσ­διορίζονται από τις λύσεις του συστήμα-{Υ = f(x) τος 1 (Σ). Υ = Γ (χ)

Το σύστημα (Σ) είναι ισοδύναμο με τα συ­στήματα {Υ = f(x) {χ = f- ι (y) (Σ ι ) και (Σz) x = f(y) y = f- ι (x) Αν ξέρουμε τον τύπο της f επιλύουμε το (Σ 1 ) ενώ αν ξέρουμε τον τύπο της f- ι επιλύουμε το (Σz).

Π ΡΟΣΟΧ Η !

Τα κοινά σημεία των Cr , CΓ, δεν βρίσκονται πά­ντοτε στη διχοτόμο y = χ. Για παράδειγμα, η συ-νάρτηση f (χ) = _!_ έχει αντίστροφη την f- ι (χ) = _!_ , χ χ οπότε όλα τα σημεία των C�" , CΓ, είναι κοινά αλλά μόνο δύο από αυτά τα ( 1 , 1 ) και (-1 , -1) ανήκουν

Ε ' f( ) - 1 ' ' στην y = χ. πισης η χ = - εχει αντιστροφη χ

f- ι ( ) - 1 ' c c ' ' λ την χ = - οποτε Γ , Γ' εχουν ο α τους τα χ σημεία κοινά αλλά κανένα δεν ανήκει στην y = χ. Α ν όμως η συνάρτηση f είναι γνησίως αύξουσα τότε αποδεικνύεται ότι οι C r , C Γ' εφόσον έχουν κοινά σημεία, θα βρίσκονται στη διχοτόμο y = χ. Δηλαδή ισχύει: Πρόταση

Α ν η f είναι γνήσια αύξουσα συνάρτηση στο Α τότε ισχύει ισοδυναμία:

f(x) = Γ1 (χ) <=> f(x) = χ, χ Ε Β = Α n f(A) .

Απόδειξη Επειδή η f είναι γνήσια αύξουσα στο Α θα είναι 1 - 1 άρα θα υπάρχει η συνάρτηση 11 η οποία θα εί­ναι και αυτή γνήσια αύξουσα στο f(A). • Αν χ Ε Β και f(x) = χ προκύπτει χ = f- 1 (x)

άρα f(x) = f- ι (χ) . • Αν χ Ε Β και f- ι (χ) = f(x) θα δείξουμε ότι

f(x) = χ . Έστω f(χ) > χ τότε f- ι (f(x) ) > f- ι (x) ή

χ > Γι (χ) ή χ > f(x) άτοπο. Έστω f(χ) < χ τότε f- ι (f(x) ) < f- ι (x) ή χ < f- ι (χ) ή χ < f(x) άτοπο. Άρα f(x) = χ. Σχόλιο

Α ν μία συνάρτηση f : Α � IR. είναι 1 - 1 τότε ορίζε­ται η f- ι : f(A) � IR. , αλλά πολλές φορές ενώ γνωρίζουμε ότι υπάρχει η Γι δεν μπορούμε να προσδιορίσουμε τον τύπο της. Αυτό συμβαίνει γιατί η εξίσωση y = f(x) δεν μπορεί να λυθεί αλγε­βρικά πάντα ως προς χ. Για παράδειγμα μπορούμε να αναφέρουμε τη συνάρτηση f(x) = 2χ + ημχ, χ Ε IR. η οποία είναι γνήσια αύξουσα άρα είναι και « 1 - 1 » οπότε ορίζεται η αντίστροφη της συνάρτη­σης f- ι : f(A) � IR. . Όμως δεν μπορούμε να βρού-με τον τύπο της, αφού η εξίσωση y = 2χ + ημχ δεν λύνεται αλγεβρικά ως προς χ. Επομένως, για να βρούμε τα κοινά σημεία των Cr και CΓ, χρησιμο-ποιούμε την προηγούμενη πρόταση . Έτσι έχουμε: Γ1 (χ) = f(x) <=> f(x) = χ <=> 2χ + ημχ = χ <=> <=> ημχ = -χ <=> χ = Ο (αφού είναι γνωστό ότι lημχ l = lx l <=> χ = Ο ). Δηλαδή είναι πολύ σημαντικό στην ανάλυση να γνωρίζουμε ότι υπάρχει η αντίστροφη f- ι μιας συ­νάρτησης f1 έστω και αν δεν γνωρίζουμε το τύπο της. Π αρατήρηση I . Η προηγούμενη πρόταση μας δίνει διέξοδο στο πρόβλημα εύρεσης των κοινών σημείων των Cr , CΓ, εφόσον υπάρχουν αλλά μερικές φορές η εξί-σωση f(x) = χ δεν επιλύεται αλγεβρικά. Π.χ. f(x) = 2χ - συνχ, Α = IR. είναι γνήσια αύξουσα στο IR. άρα αντιστρέφεται, όμως δεν μπορούμε να βρούμε την Γι και αν χρησιμοποιήσαμε την προη­γούμενη πρόταση έχουμε

f- ι (χ) = f(x) <=> f(x) = χ <=> 2χ - συνχ = χ <=> <=> συνχ = χ ( 1 ) η οποία δεν επιλύεται αλγεβρικά. Η λύση της ( 1 ) είναι η τετμημένη του σημείου το­μής των y = χ και y = συνχ. Π αρατήρηση 2. Εύκολα μπορούμε να διαπιστώσουμε ότι:

ΕΥΚΛΕΙΔΗΣ Β ' λθ ' τ.2/53

Page 56: Ευκλειδης Β 58

Μαθηματικά για την Γ Λυκείου

• Αν οι Cr , CΓ, έχουν ένα μόνο κοινό σημείο

τότε αυτό βρίσκεται στη διχοτόμο y =χ. Πράγματι, αν υπήρχε κοινό σημείο των Cr , CΓ, το Μ( α, β) και Μ ε δ : y = χ τότε θα ήταν α ::F β οπότε οι Cr , CΓ, θα είχαν και άλλο κοινό σημείο Ν(β, α) άτοπο.

• Α ν ένα σημείο ανήκει στην Cr και στη διχοτόμο y = χ τότε το σημείο αυτό ανήκει και στη C Γ' •

Πράγματι αν Μ( α, β) κοινό σημείο των Cf και y = χ τότε είναι β == f(α) και β = α., οπότε

α=β f- 1 (β) = α<::::> f- 1 (α) = β <::::> Μ( α, β) Ε CΓ, .

ΑΣ Κ Η Σ Ε Ι Σ

I . Έστω η συνάρτηση f ορισμένη στο IR η οποία έχει σύνολο τιμών το IR και για κάθε χ Ε IR ικανοποιεί τη σχέση f(f(x)) + χ = Ο . Ν α αποδείξετε ότι: α. η f είναι "1-1". β. Γ1 (χ) = -f(x) . γ. η f δεν είναι γνησίως μονότονη. δ. η f είναι περιττή συνάρτηση.

Απόδειξη α. Έστω χ 1 , χ 2 Ε IR με f(x 1 ) = f(x 2 ) τότε

f(f(x 1 )) = f(f(x 2 )) <::::> -χ 1 = -Χ2 <=> Χ 1 = Χ2 άρα η συνάρτηση f είναι ' ' 1 -1 " .

β. Αφού η f είναι " 1-1 " αντιστρέφεται, οπότε ο­ρίζεται η Γ1 • Αν όπου χ θέσουμε f- 1 (x) έχου­με f(f(f- 1 (χ))) + f- 1 (χ) = Ο <::::> f(x) + f- 1 (χ) = Ο <=> Γ1 (χ) = -f(x) .

γ. Υποθέτουμε ότι η συνάρτηση f είναι γνησίως μονότονη . Έστω χ 1 , χ 2 Ε IR με χ 1 < χ 2 • Δια-κρίνουμε περιπτώσεις: • Έστω ότι η f είναι γνησίως αύξουσα, τότε

fλ είναι f(x 1 ) < f(x2 ) <=> f(f(x 1 )) < f(f(x 2 ))

( I )

δ. Η σχέση ( 1 ) ισχύει για κάθε χ Ε IR , άρα θα ι­σχύει και για f (χ) Ε IR , οπότε έχουμε

( I ) f(f(f(x))) + f(x) = Ο<::> f( -χ) + f(x) = Ο <::::> <::::> f( -χ) = -f(x), άρα η f είναι περιττή συνάρ-τηση .

2 . Δίνεται η συνάρτηση f(x) = e' + χ - 1 .

α. Να αποδείξετε ότι η f είναι 1-1 . β. Να λύσετε την εξίσωση

eημχ + ημχ = Fe + .!. . 2

γ. Να αποδείξετε ότι e• + e < eπ + π . δ. Να λύσετε την εξίσωση (f ο f)(x) = Ο .

Απόδειξη

Η συνάρτηση f ορίζεται Α = IR . α. Έστω χ 1 , χ 2 E IR με χ 1 < χ2 γνωρίζουμε ότι

ex ' < ex' ο

Τότε προσθέτουμε κατά μέλη τις ανισότητες έχουμε ex ' + Χ 1 < ex' + Χ2 <::::> <::> ex ' + χ 1 - l < ex' + χ 2 - l <::> f(x 1 ) < f(x 2 ) άρα η συνάρτηση f είναι γνησίως αύξουσα, ο­πότε είναι και " Ι- Ι " .

β. Είναι eημχ + ημχ = Fe + .!_ <::::> eημχ + ημχ - Ι = 2 = e1 1 2 + - - I <::> f(ημx) = f - <::::> ημχ = - <=> Ι ( Ι ) r:·· ι - ι ·· Ι

2 2 2 π <::::> ημχ = η μ- <::::> 6

π χ = 2κπ + -6

<::::> ή π 5π χ = 2κπ + π - - = 2κπ + -6 6

η γ. Είναι e < π<::> f( e) < f(π) <::::>

κ Ε Ζ.

<::::> e• + e - Ι < eπ + π - Ι <::::> e• + e < eπ + π . <::::>- χ 1 < -χ2 <::::> χ 1 > χ2 που είναι άτοπο. δ. Παρατηρούμε ότι f(O) = Ο (1 ), οπότε έχουμε:

( I ) f:J - 1 ( I ) • Έστω ότι η f είναι γνησίως φθίνουσα, τότε (f ο f)(x) = O<::> f(f(x)) = f(O) <::::> f(x) = Ο<::>

rY r:ι - ι είναι f(x 1 ) > f(x2 ) <::::> f(f(x 1 )) < f(f(x 2 )) f(x) = f(O) <::::> χ = Ο .

( I ) <::::>- χ 1 < -χ2 <::::> χ 1 > χ2 που είναι άτοπο. 3 . α. Δίνεται η γνησίως μονότονη συνάρτηση

Καταλήξαμε σ' άτοπο διότι υποθέσαμε ότι η f f : Α � IR . Να αποδείξετε ότι η εξίσωση είναι γνησίως μονότονη, άρα η f δεν είναι γνη- f(x) = Ο έχει το πολύ μια ρίζα στο Α. σίως μονότονη . β. Να λύσετε την εξίσωση 3' + χ = 1 1 .

ΕΥΚΛΕΙΔΗΣ Β ' λθ ' τ. Ι/54

Page 57: Ευκλειδης Β 58

Μαθηματικά για την Γ Λυκείου

γ. Αν α, β, γ είναι τα μήκη των πλευρών ορθογωνίου τριγώνου ΑΒΓ με Α = 90• , να αποδείξετε ότι η εξίσωση β' + γ' = α' έχει μοναδική ρίζα.

Απόδειξη

α. Έστω ότι η εξίσωση f(x) = Ο έχει δύο πραγμα­τικές ρίζες τις ρ , , ρ2 με ρ , < ρ2 τότε είναι f(ρ, ) = f(ρ2 ) = ο ( 1 ) . Έστω ότι η f είναι γνησίως αύξουσα τότε από

( I ) ρ, < ρ2 � f(ρ, ) < f(ρ2 )�0 < Ο άτοπο. Έστω ότι η f είναι γνησίως φθίνουσα τότε από

( Ι ) ρι < ρ2 � f (ρι ) > f (ρ2 )� Ο > Ο άτοπο. Άρα η εξίσωση f(x) = Ο έχει το πολύ μια πραγ­ματική ρίζα.

β. Η χ = 2 είναι ρίζα της εξίσωσης 3' + χ = 1 1 , αφού 32 + 2 = 1 1 . Θα δείξουμε τώρα ότι η ρίζα χ = 2 είναι μοναδική . Θεωρούμε τη συνάρτηση

f ( Χ ) = 3χ + Χ - 1 1 , Χ Ε JR. .

Για κάθε Χ ι , χ2 Ε IR. με Χ ι < χ 2 είναι 3χ , < 3χ' . τότε προσθέτοντας κατά μέλη έχουμε 3 χ ' + Χ ι < 3 χ' + χ 2 � 3χ ' + Χ ι - 1 1 < 3χ' + χ2 - 1 1 � f(χ ι ) < f(x 2 ) , άρα η f είναι γνησίως αύξου­σα στο IR. . Αφού η f είναι γνησίως μονότονη συνάρτηση, βάσει του (α) ερωτήματος, η εξί­σωση f(x) = Ο έχει το πολύ μια ρίζα στο IR. . Όμως είχαμε αποδείξει ότι η εξίσωση f(x) = Ο έχει τουλάχιστον μια ρίζα, την χ = 2, επομένως η παραπάνω εξίσωση έχει μοναδική ρίζα την χ = 2 .

γ. Από το Πυθαγόρειο Θεώρημα ισχύει β2 + γ2 = α2 , επομένως η εκθετική εξίσωση β χ + γχ = α' έχει μια τουλάχιστον ρίζα, την χ = 2 . Θεωρούμε τη συνάρτηση f(x) = (�)x + (�) χ - 1, X E IR. . Επειδή β < α

και γ < α είναι Ο < Q. < 1 και Ο < 1. < 1 επομέ-α α νως οι εκθετικές συναρτήσεις (�τ και (�)χ είναι γνησίως φθίνουσες. Για κάθε χ ι , χ2 Ε IR. με χ , < χ2 είναι

(+ ) ( β ) χ , ( γ )χ , � - + - > α α

ως φθίνουσα στο IR. . Αφού η f είναι γνησίως μονότονη συνάρτηση, βάσει του α. ερωτήμα-τος, η εξίσωση f (χ) = Ο � (�)χ + (�)χ - 1 = Ο

� β χ + γχ = αχ θα έχει το πολύ μια ρίζα στο IR. . Όμως είχαμε αποδείξει ότι η εξίσωση αυτή έχει τουλάχιστον μια ρίζα, την χ = 2 , επομένως η χ = 2 είναι η μοναδική ρίζα της παραπάνω ε­ξίσωσης.

4. α. Θεωρούμε τις συναρτήσεις f, g : Α � IR. . Α ν η f είναι γνησίως αύξουσα στο Α, η g είναι γνησίως φθίνουσα στο Α και για κάθε χ Ε Α είναι f(x) > Ο και g(x) > Ο, ' δ 'ξ ' ' f τοτε να απο ει ετε οτι η συναρτηση -

g είναι γνησίως αύξουσα στο Α.

β. Να αποδείξετε ότι η συνάρτηση

F( ) ln χ , , ' ξ χ = -- ειναι γνησιως αυ ουσα στο συν χ

π π γ. Αν - < χ < χ < - να αποδείξετε ότι 3 I 2 2

Χσυνχ2 < Χσυνχ1 I 2

Απόδειξη α. Για κάθε χ ι , χ 2 Ε Α με Χ ι < χ 2 είναι

0 < f(x ι ) < f(x2 ) } 0 < f(x , ) < f(x2 ) } ,

� 1 1 τοτε g(x 1 ) > g(x2 ) > 0 0 < -- < --g(x, ) g(x2 ) f(x ι ) · -1 -<f(x2 ) · -1 - � f(x , ) < f(x2 ) � g(χ ι ) g(x 2 ) g(χ ι ) g(x2 ) � (�) cx 1 ) < (�) cx 2 ) , άρα η συνάρτηση � είναι γνησίως αύξουσα στο Α.

β. Θεωρούμε τις συναρτήσεις f(x) = lnx και g(x) = συνχ. Η συνάρτηση f είναι γνησίως αύξουσα

ΕΥΚΛΕΙΔΗΣ Β' λθ ' τ.2/55

Page 58: Ευκλειδης Β 58

Μαθηματικά για την Γ Λυκείου

στο ( �,%) και η συνάρτηση g είναι γνησίως

φθίνουσα στο ( � ·%) . Επίσης για κάθε

χ ε ( �·%) είναι f(x) > Ο και g(x) > Ο, άρα λό­γω της α. έχουμε ότι η συνάρτηση F είναι γνη-, ' ξ ( π π ) σιως αυ ουσα στο "3, "2 .

Π Π Fλ γ. Έχουμε - < χ ι < Χ2 < -=> F(x ι ) < F(x2 ) <=> 3 2

Ιη χι ln x2 <=>--<--<=>συνχ2 · ln x ι <συνχι · ln x2 συνχι συνχ2 αφού συνχ ι > Ο και συνχ 2 > Ο

5. α. Δίνονται οι συναρτήσεις f, g : Α � IR . Α ν η f είναι γνησίως αύξουσα στο IR και η g είναι γνησίως φθίνουσα στο IR , τότε να αποδείξετε ότι: i) για κάθε h > Ο και χ ε IR ισχύει

g(x + h) - g(x) < f(x + h) - f(x) . i i ) οι γραφικές παραστάσεις των συναρ­

τήσεων f, g έχουν το πολύ ένα κοινό σημείο.

β. Να αποδείξετε ότι οι γραφικές παρα­στάσεις των συναρτήσεων f(x) = e' + χ3

και g(x) = e-• - 2χ έχουν μοναδικό κοινό σημείο.

ΑΠόδειξη α. i ) Θεωρούμε τη συνάρτηση F(x) = f(x) - g(x) ,

χ ε !R . Για κάθε χ ι , χ 2 ε !R με χ ι < χ2 είναι f(χ ι ) < f(x2 ) } f(χ ι ) <f(x2 ) } , <=> τοτε g(χ ι ) > g(x 2 ) -g(χ ι ) < -g(x2 ) f(χ ι ) - g(χ ι ) < f(x2 ) - g(x 2 ) <=> (f - g)(χ ι ) < (f - g)(x 2 ) , άρα η συνάρτηση F είναι γνησίως αύξουσα στο IR . Αφού h > Ο είναι χ < χ + h και F λ θα ισχύει F(x) < F(x + h) <=> f(x) - g(x) < f(x + h) - g(x + h) <=> g(x + h) - g(x) < f(x + h) - f(x) .

i i ) Αφού η συνάρτηση F είναι γνησίως μονότο­νη (αύξουσα), σύμφωνα με την άσκηση 3 .α. , η εξίσωση F(x) = Ο <=> f(x) = g(x) θα έχει το πολύ μια ρίζα, δηλαδή θα υπάρχει το πολύ ένα σημείο χ0 ε IR τέτοιο ώστε:

F( χ0 ) = Ο <=> f (χ0 ) = g( χ0 ) που σημαίνει ότι οι γραφικές παραστάσεις των συναρτήσεων f και g έχουν το πολύ ένα κοινό σημείο Μ(χσ, Υο) με Υ ο = f(x0 ) = g(x0 ).

β. Παρατηρούμε ότι f(O) = g(O) = I άρα οι Cr, Cg έχουν κοινό σημείο το Μ( Ο, 1 ) . Επίσης έχουμε τη συνάρτηση f(x) = e ' + χ3 είναι γνησίως αύ-ξουσα στο IR , γιατί e' λ και χ3 λ ενώ η συ­νάρτηση g(x) = e- x - 2χ είναι γνησίως φθί-νουσα στο IR γιατί e- x 1 και -2χ 1 . Βάσει όμως του ερωτήματος (αii) το σημείο Μ(Ο, 1 ) είναι το μοναδικό κοινό σημείο των C r και Cg

6. Δίνεται η συνάρτηση f(x) = χ + .Jx2 + 1 . Να

αποδείξετε ότι: α. f(x) > Ο για κάθε χ ε IR . β. η f αντιστρέφεται και να ορίσετε την

συνάρτηση Γ1 •

Απόδειξη α. Για κάθε χ ε IR είναι χ2 + I > Ο , οπότε η f ορί­

ζεται στο IR . Είναι f(x) = x +

� > χ +

Ν =

= χ + l xl � χ + (-χ) = Ο, χ ε IR .

Άρα f(x) > Ο για κάθε χ ε IR . β. Έστω χ ι , χ 2 ε !R με f(x ι ) = f(x 2 ) <=>

Χ ι +�

= Χ2 +�χ; + I <=>

(χ ι + �Χ� + 1 )(χ ι -

�Χ � + 1 ) <=> =

Χ ι -�Χ� + 1

(χ 2 +*'

)(χ2 - *') χ2 -

�χ; + 1

- 1 - 1 <=> = <=> Χ ι -

�Χ� + 1 χ2 -

�χ; + 1

<=> Χ ι -�Χ� + 1 = χ2 -

�χ; + 1

και επειδή ισχύει χ ι + �χ� + 1 = χ2 +

�χ; + 1

προσθέτοντας κατά μέλη έχουμε: 2χ ι = 2χ2 <=> Χ ι = Χ2 .

Άρα η f είναι 1- 1 οπότε η f αντιστρέφεται. Για να βρούμε την f- ι λύνουμε την y = f(x) ως προς χ. 'Ετσι y = χ + � , y > Ο <=>

ΕΥΚΛΕΙΔΗΣ Β ' λθ' τ. l /56

Page 59: Ευκλειδης Β 58

Μαθηματικά για την Γ Λυκείου

<=> y = (χ + J;2;ϊχχ - J;2;ϊ) χ - J;2;ϊ

<::> y = � <=> _!._ = J;2;ϊ - χ χ - νχ2 + 1 Υ

1 � <=> - - y = νx - + 1 - x - y Υ <=> _!._ - y = J;2;ϊ - χ - χ - J;2;ϊ Υ

1 1 1 <=> - - Υ = -2χ <=> Υ - - = 2χ <=> χ = -y - -Υ Υ 2 2y

y2 - 1 χ2 - 1 <::::> χ = -- τότε f- ι ( χ ) = --, χ > Ο 2y 2χ 7. Δίνεται η συνάρτηση f(x) = vΓx+J - J;. .

α. Να αποδείξετε ότι f(x) > Ο για κάθε χ Ε [Ο,+οο) .

β . Να αποδείξετε ότι η f αντιστρέφεται και να ορίσετε τη συνάρτηση Γ1 •

γ. Να λύσετε την εξίσωση <Μ - Ji.) . <-lx3 +x - 7+-./χ3 +χ - s )=t .

Απόδειξη {χ + 1 2:: ο

α. Για να ορίζεται η f πρέπει <::::> χ 2:: Ο , χ 2:: 0 άρα Ar = [Ο, +οο] . Για κάθε χ Ε [Ο, +οο) είναι f(x) = .Jx+l - Γχ > fx - Γχ = Ο . Επομένως f(x) > Ο για κάθε χ Ε [0, +οο) .

β. Έστω Χ ι , Χ 2 Ε [Ο, +οο) με f(x ι ) = f(x 2 ) <=> F:+ϊ - F: = ..;χ;+ϊ - ;;:; <=> F:+ϊ - .,Γχ;+ϊ = F: - ;;:; <=> (χ ι + 1) - (χ2 + 1) Χ ι - χ2

--;:::'===----;=== = <=> F:+ϊ + .,Γχ;+ϊ F: + ;;:; Χι - Χ2 = Χ ι - Χ2 <=> F:+ϊ + .,Γχ;+ϊ F: + ;;:;

(Χι -χ2 )(Jχι + I +Jx2 + l ) = (χι -χ2 ) (Fι +F;) <=> <=> (χ ι - χ2 ) ( F:+l-F: + .,Γχ;+ί - F;) = Ο <=> (χ ι - x 2 )[f(x ι ) + f(x 2 )] = 0 <=>

'--ν------' >0 <::::> χ ι - χ 2 = Ο <::::> χ ι = χ2 , άρα η f είναι « 1-1 », οπότε αντιστρέφεται. Για να βρούμε την Γι λύνουμε την y = f(x) ως προς χ. Είναι

y = f(x)} <=> y = .Jx+l - fx} <=> y > O y > O Υ + j;. = .Jx+l} <=> (y + fx/ = χ + 1} <=> y > O y > O / + 2yf;. + χ = χ + 1} <=> 2Γχy = 1 - y2 } <=> y > O y > O fx = 1 - / } ι -y'�ο χ = (1 - y2 )2 }

2y <=> 4y2 <=> y > O O < y ::; I

f- ι (y) = y4 -

4�2 + 1 }

0 < y ::; 1 ι χ4 - 2χ2 + 1 Άρα Γι : (Ο, 1] � 1R με Γ (χ ) = 2 · 4χ

γ. Είναι: ( .Jx+l - Γχ )( -./χ3 + χ - 7 + -./χ3 + χ - 8 ) = 1 <=> .Jx+l - Γχ =

1 <=> -./χ3 + χ - 7 + -./χ3 + χ - 8

� Γ -./χ3 + χ - 7 - -./χ3 + χ - 8 ν χ + ι - ν χ = <=> (χ3 + χ - 7) - (χ3 + χ - 8) .Jx+l - Γχ = -./χ3 + χ - 7 - -./χ3 + χ - 8 <=>

f: ι - ι f(x) = f(x3 + χ - 8) <::::> χ 3 + χ - 8 = χ <::::> Χ3 - 8 = 0 <::::> Χ 3 = 8 <::::> Χ = 2

8. Έστω η συνάρτηση f η οποία για κάθε χ Ε IR. ικανοποιεί τη σχέση f3 (x) + 5f(x) + x = O . α. Να αποδείξετε ότι η f αντιστρέφεται και

να ορίσετε τη συνάρτηση Γ1 • β. Να βρείτε τα κοινά σημεία των Cr και C Γ' •

Απόδειξη α. Έστω Χ ι , χ 2 Ε IR με f(χ ι ) = f(xz) τότε είναι

f\χ ι ) = f3 (χ 2 ) , επίσης είναι 5f(x 1 ) = 5f(x2 ) , οπότε έχουμε f\xι ) + 5f(x ι ) = f\x2 ) + 5f(x2 ) (2). Η σχέση ( 1) ισχύει για κάθε χ Ε IR άρα θα ι­σχύει και για: χ = Χι οπότε είναι f\χ ι )+5f(χ ι ) + Χ ι =Ο (3) και χ = χ2 οπότε είναι f\x2 )+5f(x2 ) + x2 =0 (4) Από (3) και (4) έχουμε

ΕΥΚΛΕΙΔΗΣ Β' λθ ' τ.2/57

Page 60: Ευκλειδης Β 58

Μαθηματικά για την Γ Λυκείου

( 2 ) f3 (χ , ) +5f(x 1 ) + χ , = f3 (χ 2 ) + 5f(x2 ) + χ 2 <=> χ 1 = χ2 , άρα η συνάρτηση f είναι « 1 - 1 » οπότε αντιστρέφεται και μάλιστα ισχύει η ισοδυναμία Υ = f(x) <=> χ = r- ' (y) .Για κάθε χ Ε JR έχουμε f3 (χ) + Sf(x) + χ = Ο <=> / + Sy + Γ' (y) = Ο <=> y3 + sy + r- ' (y) = ο <=> r- ' (y) = -y3 - sy για κάθε y Ε JR . Επομένως έχουμε Γ' : JR ---+ JR με r- ' (x) = -χ3 - Sx .

β . Λύνουμε το σύστημα {Υ = r- ' (χ) {Υ = r- ' (χ) y = f(x) <=> r- ' (y) = f- ' (f(x)) <=> {y = -x3 - 5x <=> {x3 + 5x + y = O -/ - Sy = χ y3 + Sy + χ = Ο

Αν αφαιρέσουμε κατά μέλη έχουμε: χ 3 - y3 + 4 χ - 4 y = Ο (χ - y )(χ 2 + xy + y2 ) + 4( χ - y) = Ο <=> (χ - y )(χ 2 + xy + y2 + 4) = Ο <=> χ - y = Ο <=>

>0 <=> y = x . Άρα το σύστημα ισοδύναμα γράφεται:

{χ, + �χ�+χχ � ο "" {χ, : ::� ο "" { Υ = χ {Υ = χ {χ = Ο χ(χ 2 + 6) = 0 <=> χ = 0 <=> y = O

Άρα κοινό σημείο των Cf και C r- ' είναι το (0, 0).

9. Έστω συνάρτηση f η οποία είναι 1-1 και για κάθε χ Ε JR * ικανοποιεί τη σχέση

(f ο f)(x) · f(x) = α, α :;t: Ο . α. Ν α αποδείξετε ότι ( f ο f)( χ) = χ για κάθε

Χ Ε JR* .

β. Να βρείτε τον τύπο της f.

Απόδειξη

α. Για κάθε x E JR * είναι (f o f)(x) · f(x) = α, α :;t: Ο , οπότε (f ο f)(x) :;t: Ο και f(x) :;t: Ο . Αν όπου χ θέσουμε f(x)(:;t: Ο) έχουμε:

(f ο f)(f(x)) · f(f(x)) = α <=> (f ο f)(f(x)) · (f ο f)(x) = α <=>

α (f ο f)(f(x)) · - = α <=> (f ο f)(f(x)) = f(x) <=> f(x) f:\ - 1

f(f(f(x))) = f(x) <=> f(f(x)) = χ <=> (f o f)(x) = x, x E JR * .

β. Για κάθε χ Ε JR * είναι α (f ο f)(x) · f(x) = α <=> xf(x) = α <=> f(x) = - . χ

ι Ο. Έστω συνάρτηση f η οποία για κάθε χ Ε JR ικανοποιεί τη σχέση (f ο f)(x) = χ3 • α. Ν α αποδείξετε ότι

i) η f αντιστρέφεται. ii) f(x3 ) = (f(x))3 για κάθε χ Ε JR .

β. i) Να λύσετε την εξίσωση f(x) = χ. ii) Να αποδείξετε ότι

[f(-1)]3 + [f(1)]3 = f(O) . iii) Α ν f(8) = 64 να υπολογίσετε την τιμή

f(2).

Απόδειξη

α. i) Έστω χ , , χ2 Ε JR με f(x 1 ) = f(x2 ) . Είναι f(f(x 1 )) = f(f(x 2 )) <=> χ� = χ; <=> χ , = χ 2 άρα η f είναι « 1 - 1 » οπότε αντιστρέφεται.

ii) Για κάθε χ Ε JR είναι f(f(x)) = χ3 . Αν όπου χ θέσουμε f(x) έχουμε f(f(f(x))) = f3 (x) <=> f((f ο f)(x)) = f\x) <=>

f(x 3 ) = f\x), χ Ε JR . β. i) Επειδή η f είναι « 1 - 1 » ισχύει η ισοδυναμία

f(x) = χ <=> f(f(x)) = f(x) <=> f ( x )=x

(f ο f)(x) = f(x) <=> χ3 = f(x) <=> χ3 = χ <=> χ(χ2 - 1) = Ο <=> χ(χ - 1)(χ + 1) = Ο <=> χ = Ο ή χ = 1 ή χ = -1 .

ii) Επειδή οι αριθμοί -1 , Ο, 1 είναι ρίζες της ε­ξίσωσης f(x) = χ την επαληθεύουν, οπότε f(-1 ) = -1 , f(Ο) = Ο και f( 1 ) = 1 . Είναι [f( - 1)]3 + [f(1)]3 = ( - 1)3 + 13 = = - 1 + 1 = Ο = f(O).

iii) Είναι f(8) = 64 <=> f(23 ) = 64 <=> [f(2)]3 = 64 <=> f(2) = ψ;;, <=> f(2) = 4 .

1 ι . Έστω συνάρτηση f η οποία για κάθε χ, y Ε JR ικανοποιεί τη σχέση

ΕΥΚΛΕΙΔΗΣ Β ' λθ ' τ.l/58

Page 61: Ευκλειδης Β 58

Μαθηματικά για την Γ Λυκείου

f(x + y) � f(x) · f(y) � e•+Y . α. Να αποδείξετε ότι f(O) = 1 . β. Να αποδείξετε ότι f( -χ) = -1- για κάθε

f(x) Χ Ε IR .

γ. Να βρείτε τον τύπο της f. Απόδειξη

α. Για χ = y = 0 έχουμε f (O ) � f2 (Ο ) � 1 άρα {f (O) � f2 (Ο ) � {f (0) { 1 - f (O) ) � Ο � f (0 ) � 1 f (O ) � l {1 - f (O ) � O {f(0) $ 1 , � αρα f(O) = 1 . f (O ) � l f(O) � l f( O)=I

β. Για y = -χ έχουμε f(O) � f(x)f( -χ) � 1 � l � f(x )f(-x) � l , άρα f(x)f(-x) = l, x E IR οπότε f( -χ) = -1

- , χ Ε IR ( 1 ) . f(x) γ. Για χ Ε IR και y = Ο έχουμε:

f ( O )=I f(x) � f(x)f(O) � e' � f(x) � e' (2)

οπότε f(x) > Ο για κάθε χ Ε IR . Για χ = Ο και y = - χ έχουμε

f ( O )=I ( \ ) f(-x) � f(O)f(-x) � e-x � f(-x) � e-x � 1 1 f( χ )>0

- � - � f(x) $ e' (3 ) f(x) e ' Από (2) και (3 ) έχουμε f(x) = e', χ Ε IR .

1 2 . Έστω συνάρτηση f με πεδίο ορισμού και σύνολο τιμών το IR , η οποία ικανοποιεί τη σχέση f(x + y) = f(x) + f(y) για κάθε χ, y Ε IR . Ν α αποδείξετε ότι: α. η Cr διέρχεται από την αρχή των αξόνων.

β. Η f είναι περιττή. γ. f(x - y) = f(x) - f(y) για κάθε χ, y Ε IR . δ. αν η εξίσωση f(x) = Ο έχει μοναδική ρίζα

την χ = Ο, τότε η f είναι 1-1 . ε. Γ' (χ + y) = Γ1 (χ) + Γ1 (y) για κάθε

x, y E IR • .

Απόδειξη α. Για χ = y = Ο έχουμε f(O) = f(O) + f(O) �

f(O) = Ο , άρα η Cr διέρχεται από την αρχή των αξόνων.

β. Για y = -χ έχουμε f(x + ( -x))=f(x) + f( -χ) � f( O)=O

f(O) = f(x) + f(-x) � O = f(x) + f(-x) � f( -χ) = -f(x), χ Ε IR άρα η f είναι περιττή συ-νάρτηση .

γ. Αν θέσουμε όπου y το -y έχουμε: f περιττή f(x + ( -y)) = f(x) + f( -y) �

f(x - y) = f(x) - f(y), x , y E IR . δ. Έστω Χ 1 , Χ 2 E !R με f(x 1 ) = f(x2 ) �

( γ ) f(x 1 ) - f(x2 ) = 0� f(x 1 - χ2 ) = 0 Επειδή η εξίσωση f(x) = Ο έχει μοναδική ρίζα την χ = Ο συμπεραίνουμε ότι χ , - χ2 = Ο � χ , = χ2 • Άρα η f είναι « 1- 1 ».

ε. Αφού η f « 1 - 1 )) αντιστρέφεται και ισχύει f (α) = χ � α = f- 1 (χ) f(β) = y � β = f- ' (y) Είναι f(α + β) = f(α) + f(β) � f(α + β) = χ + y � α + β = f- ' (x + y) � f-1 (x) + f- ' (y) = f- ' (x + y), x , y E IR .

ΑΣΚΗΣΕΙΣ ΣΤΑ OPIA

I . Να βρείτε το όριο :

L = lim 2 jx + t j - 3 j x - 2j - t . χ--+ 1 χ2 + 3χ - 4

Λύση

Επειδή χ � 1 , θεωρούμε ότι x E (0, 1) u (1, 2) . Το σύνολο (0, 1) υ (1, 2) είναι υποσύνολο του συνόλου

του Βασίλη Δ. Καρακατσάνη

ορισμού της συνάρτησης, της οποίας ζητάμε το ό­ριο (γιατί;) . Έτσι, έχουμε: χ =ι. 1 και

0 < χ < 2 � � {χ + 1 > ο {Ι χ + 1 1 = χ + 1 χ - 2 < Ο jx - 2 j = -χ + 2

Συνεπώς:

ΕΥΚΛΕΙΔΗΣ Β ' λθ ' τ.2/59

Page 62: Ευκλειδης Β 58

Μαθηματικά για την Γ Λυκείου

L = lim 2(x + 1) - 3(-x + 2) - 1 = lim 5χ - 5 = Χ-->1 Χ 2 + 3χ - 4 Χ-->1 Χ 2 + 3χ - 4

2.

= Iim 5(χ - 1) lim-5- = 1 . χ-->Ι (χ - 1)(χ + 4) χ-->Ι χ + 4

Ν β , , L 1 . χ - 2Fx + ι α ρειτε το οριο : = ιm 3 •

Χ--> 1 χ - 3χ + 2 Λύση

Έχουμε: χ3 - 3χ + 2 = χ3 - χ - 2χ + 2 = χ(χ2 - 1) - 2(x - l)= = χ( χ - l)(x + 1) - 2(χ - 1) = (χ - 1)(χ 2 + χ - 2) = = (χ - 1) [ (χ 2 - I) + (χ - I) J = = (χ - 1) [ (χ - l)(x + Ι) + (χ - 1)] = (χ - 1)2 (χ + 2).

Επειδή χ � I θεωρούμε ότι χ ε (O, l) u (1, 2) . Έχουμε: χ - 2....Γχ + I = ( ._.Γχ )2 - 2....Γχ + 1 = ( ._.Γχ - 1)2 . 'Ε '

L -- Ιι"m ( ._.Γχ - l)2 τσι, εχουμε: χ-->1 (χ - 1)2 (χ + 2)

3.

= lim (._.Γχ" - Ι)2 (._.Γχ" + Ι)2 = χ-> Ι (χ - 1)2 (χ + 2)( ._.Γχ + 1)2

= lim (χ - l)2 = χ-->1 (χ - 1)2 (χ + 2)( ._.Γχ + 1)2

= lim 1 = χ--> Ι (χ + 2)(._.Γχ" + 1)2 1 2

Μια συνάρτηση f είναι ορισμένη στο IR και ισχύει:

I . f(x) - 1 3 ιm = (1) ...... . χ - 1

Να βρείτε τα όρια: i . L = lim f(x) ...... .

L, ι· x100f(x) - 1 ί ί . = ιm -----'"--'---...... . χ - 1

Λύση i . Από την ( 1 ) και επειδή Iim(x - 1) = Ο , έπεται χ-->1

ότι: lim f(x) - l _ (x - 1) = 3 · 0 � Χ-->1 Χ - I lim [f(x) - 1] = Ο � lim f(x) = I . χ�ι χ�ι

i i . Έχουμε: , . χ ι οοf(χ) _ χ • οο + χ ι οο - I L = lιm = χ - 1

4.

I . [ 1 00 f(x) - 1 X 1 00 - l ] = ιm χ · + = Χ-->1 Χ - 1 Χ - I

= lim [x ι oo . f(x) - 1 + χ99 + x9s + . . . + x + l] = Χ-->1 Χ - 1 ( Ι ) = 1 1 00 • 3 + (1 + 1 + . . . + I) = I 03 .

� 1 00 φορές

Να βρείτε τους αριθμούς α, β ε IR , για τους οποίους ισχύει:

lim � + αχ + β = � ...... . χ - 1 2 .

Λύση

(1)

I . Έστω ότι για δύο αριθμούς α, β ε IR η ( 1 ) ισχύ-ει. Τότε, επειδή και lim(x - 1) = Ο , έπεται ότι: χ-->1

I . .Jχ2 + 2 + αχ + β ( 1) - 3 Ο ιm χ - - - · � χ-->1 χ - I 2 Iim {.Jx2 + 3 + αχ + β ) = Ο � 2 + α + β = Ο � χ-->1

� β = -α - 2 (2). Αντικαθιστώντας στην ( I ), έχουμε: I . .Jχ2 + 3 + αχ - α + 2 3 ιm = - � χ-->1 χ -1 2

I . (.Jx2 + 3 - 2 J 3 � ιm + α = - � Χ-->1 Χ - I 2

� ιm · + α = - � I . ( (x - l)(x + l) J 3 χ-->Ι (x - l)(.Jx 2 + 3 + 2) 2

2 3 - + α = - � α = Ι . 4 2 Άρα, τότε, λόγω και της (2), έχουμε: α = 1 και β = - 3 .

2 . Αντιστρόφως. Έστω ότι α = 1 και β = -3 . Εξε­τάζουμε αν ισχύει η ( I ) . Έχουμε, τότε: I . .Jx2 + 3 + x - 3 ι · � - 2 + χ - 1 ιm = ιm = Χ-->1 Χ - 1 Χ-->1 Χ - I

I . (.Jx2 + 3 - 2 ι ] ( ' , ) = ιm + οπως παραπανω Χ-->1 Χ - I

= � + I = �. Άρα, η ( 1 ) ισχύει. Συνεπώς, οι ζη-4 2 τούμενοι αριθμοί, είναι: α = I και β = -3 .

5 . Δύο τριώνυμα: α, Χ2 + β1 χ + γ 1 και με πραγματικούς

ΕΥΚΛΕΙΔΗΣ Β' λθ ' τ. Ι/60

Page 63: Ευκλειδης Β 58

Μαθηματικά για την Γ Λυκείου

συντελεστές (α1α2 :ι:. Ο) και πραγματικές

ρίζες, έχουν μία κοινή ρίζα χ0• Ονομάζουμε Δ1 και Δ2 τις διακρίνουσες των τριωνύμων αυτών αντίστοιχα και υποθέτουμε ότι Δ2>0.

Να δείξετε ότι: lim α, χz + β , χ + γ , = f"Δ:.

.... .. αzχ2 + βzχ + γz νΔ: Λύση

Ονομάζουμε χ 1 την άλλη ρίζα του πρώτου τριωνύ­μου και χ2 την άλλη ρίζα του δευτέρου. Επειδή Δz > Ο, έχουμε χ0 :ι:. χ2 . Έτσι, ονομάζοντας L το υπόψη όριο, έχουμε: L = lim αι (χ - χσ )(χ - χ ι ) = lim αι (χ - χ ι ) Ι = χ-+Χο α2 (χ - χ0 )(χ - χ 2 ) Χ->χο α2 (χ - χ 2 )

_ Jαι (χ ο - Χ ι ) J - lα2 Cxo - χ2 ) 1 ·

6 . Μια συνάρτηση f είναι ορισμένη στο διά-

στημα (- ; , ; ) και για κάθε χ Ε (-; , ; ) ισχύουν:

ημ2Χ S f(X) S Χ2 • (1)

Ν α βρείτε τα όρια:

I . L, = lim f(x) . 2. L2 = lim f(x) - f(O) . χ�Ο χ�Ο Χ Λύση

Ι . Έχουμε: Ιίm ημ2χ = ημ2 0 = 0 και lim x2 = 0 χ�ο χ�ο Έτσι, από το θεώρημα ενδιαμέσων τιμών, έ­χουμε: lim f(x) = O. χ�ο

2 . Από την ( 1 ) με χ = Ο, βρίσκουμε: Ο ::;; f(O) ::;; Ο και άρα f(O) = Ο. Έτσι, έχουμε:

L2 = lim f(x) . χ�ο χ Για κάθε χ Ε (Ο,�) , από τις ( 1 ) έχουμε:

ημ2χ f(x) 2 -- s -- s x χ χ

Και επειδή : lim η μ 2χ = lim (ημχ . ημχ ) = χ�ο χ χ�ο χ = 1 · Ο = Ο και lim χ2 = Ο, έπεται ότι: χ�Ο

lim f(x) = Ο. Ομοια βρίσκουμε: lim f(x) = Ο . χ�ο· χ χ�ο- χ Άρα: Lz = Ο.

7. Για μια συνάρτηση f : lR � JR ισχύει: f(χ)ημχ ::;; ημ5χ, για κάθε χ ε JR .

Επίσης, ισχύει: �� f(x) = .e. Ε 1R . Να βρείτε

τον αριθμό .f. . Λύση

Για κάθε χ Ε (Ο ,�) , έχουμε ημχ > Ο και άρα:

f(x) ::;; ημ5χ . ( 1 ) ημχ Για κάθε χ Ε ( -� , 0) , έχουμε ημχ < Ο και άρα:

f(x) 2 ημ5χ . (2) ημχ

5 ημ5χ 'Εχουμε: lim ημ5χ = lim 5χ = 5 . χ�ο ημχ χ�ο ημχ

χ 'Ετσι έχουμε: lim ημ5χ = 5 χ�ο· ημχ και lim ημ5χ = 5 .

χ �ο.-

ημχ Επίσης, έχουμε: lim f(x) = .e. χ-+0'+- και lim f(x) = ι Χ""""+Ο-Από την ( 1) έπεται ότι:

lim f(x) ::;; lim ημ5χ => f ::;; 5 (3) χ�ο· χ�ο· ημχ

Από τη (2) έπεται ότι: lim f(x) 2 lim ημ5χ

=> f 2 5 (4) χ�ο- χ�ο ημχ Από τις (3) και (4) έπεται ότι: f = 5 .

8. Μια συνάρτηση f είναι ορισμένη κοντά στο

ο · ι· f(χ) + ημχ - 1 2 (1) και ισχυει: ιm = . χ�Ο Χ

Ν β , , L . f(x) - 1 α ρειτε το οριο = lιm � . νf(χ) - συνχ

ΕΥΚΛΕΙΔΗΣ Β ' λθ ' τ.2/61

Page 64: Ευκλειδης Β 58

Λύση

'Ε ι · ημχ Ι Α ' ' ( Ι ) ' χουμε ιm- = . πο αυτη και την επεται Χ-+0 Χ ότι: Ιίm [ f(χ) + ημχ - Ι _ ημχ ] = 2 - Ι :::::> Χ-+0 Χ Χ

:::::> lim f(x) - I = I (2). χ -+0 χ Θ , , ( ) f(x) - Ι . εωρουμε τη συναρτηση: g χ = . χ Η g είναι ορισμένη κοντά στο Ο και ισχύει: limg(x) = Ι . χ-+0 Έχουμε (κοντά στο 0): f(x) = xg(x) + Ι . Έτσι, έχου-με: ι = lim xg( χ) + Ι - Ι =

, ... ο .Jxg(x) + Ι - συνχ

. xg(x) [ .Jxg(x) + Ι + συνχ J = lιm = ..... ο xg(x) + Ι - συν2χ

= lim g(x)[ �xg(x) + Ι + συνχ J = Ι(...Γο+Ϊ + Ι) =2. ..... ο g( χ) + ημχ . ημχ Ι + Ι · Ο

χ 9. Να βρείτε το όριο: L = lim x•+t . Χ--++οο

Λύση

Έχουμε χ > Ο και άρα χ '+ ' > Ο , οπότε: ι = lim Χ Χ+ ( = lim e(n XHI = lim e( Χ+ ( ) (Π Χ

Θέτουμε y = (χ + I) ln χ . 'Εχουμε: lim y = lim (χ + Ι) ln χ = ( +οο) · ( +οο) = +οο. Χ--++00 Χ--++00

Έτσι, έχουμε: ι = lim eY = +οο. y--++00

Συνέχεια σε κλειστό διάστημα και _. εφαpμογες της

Π ρόταση I

Α ν μια συνάρτηση f είναι συνεχής σ' ένα διά­στημα Δ και δε μηδενίζεται σ' αυτό, δηλαδή f(x) =F- Ο για κάθε χ Ε Δ , τότε η f διατηρεί στα-θερό πρόσημο στο Δ.

Απόδειξη

Έστω ότι η f δεν διατηρεί σταθερό πρόσημο στο Δ, τότε θα υπάρχουν κ, λ Ε Δ με κ < λ τέτοια ώστε f(κ)f(λ) � Ο . Διακρίνω περιπτώσεις: • Αν f(κ) · f(λ) = Ο � f(κ) = Ο ή f(λ) = Ο άτοπο

γιατί f(x) =F- Ο για κάθε χ Ε Δ . • Αν f(κ)f(λ) < Ο και με δεδομένο ότι η f είναι

συνεχής στο [κ, λ] ς Δ ισχύει θεώρημα Baizano, επομένως θα υπάρχει ένα τουλάχιστον χ0 Ε (κ, λ) τέτοιο, ώστε f(x0 ) = Ο το οποίο εί-ναι άτοπο γιατί f(x) =F- Ο για κάθε χ Ε Δ . Επο-μένως η f διατηρεί σταθερό πρόσημο στο Δ.

Α ν αγνώστου Κώστας του Θεοχάρη

Πρόταση 2 Έστω f : [α, β] --+ IR συνεχής συνάρτηση. Αν για κάθε χ Ε [α, β] είναι f(x) =F- κ όπου κ Ε IR , τότε f(α) < κ και f(β) < κ ή f(α) > κ και f(β) > κ .

Απόδειξη

Άλλη διατύπωση του θεωρήματος των ενδιάμεσων τιμών. Π ρότα ση 3 (Σταθερό σημείο Brouwer)

Έστω συνάρτηση f συνεχής στο [α, β] με α � f(x) � β για κάθε χ Ε [α, β] . Να αποδείξετε ότι υπάρχει χ0 Ε [α, β] , τέτοιο ώστε f(x0 ) = χ0 •

Απόδειξη

Θεωρούμε τη συνάρτηση g(x) = f(x) - χ, χ Ε [α, β] . Παρατηρούμε ότι:

• Η g είναι συνεχής στο [α, β] ως διαφορά συνε­χών

• g(α) · g(β) = [f(α) - α] · [f(β) - β] � Ο '--ν--' �

<=Ο �ο

ΕΥΚΛΕΙΔΗΣ Β ' λθ ' τ. Ι /62

Page 65: Ευκλειδης Β 58

Μαθηματικά για την Γ Λυκείου

Διακρίνουμε περιπτώσεις i) αν g(α) · g(β) = Ο � g(α) = Ο ή g(β) = Ο �

f(α) = α ή f(β) = β ί ί) αν g(α) · g(β) < Ο τότε ισχύει Θ. Bolzano, οπότε

θα υπάρχει χ0 ε (α, β) έτσι ώστε g(x0 ) = Ο � f(x0 ) = χ0 Άρα υπάρχει χ0 ε [α, β] τέτοιο ώστε f(x0 ) = χ0 .

Πρόταση 4

Κάθε πολυώνυμο περιττού βαθμού με πραγμα­τικούς συντελεστές έχει τουλάχιστον μια πραγ­ματική ρίζα.

Απόδειξη Έστω πολυωνυμική συνάρτηση

f( ) ν ν-1 χ = ανχ + αν_1 χ + . . . + α1χ + α0 με ν =2ρ + 1 , αν :;t: Ο . • Αν αν > 0 τότε lim f(x) = lim (ανχν ) = -οο

x -+-<JO x -+-<JO

άρα υπάρχει λ < Ο τέτοιο ώστε f(λ) < Ο, επίσης lim f(x) = lim (ανχν ) = +οο , άρα υπάρχει μ > Ο

χ -+t<JO χ -+t<t::

τέτοιο ώστε f(μ) > Ο . Για τη συνάρτηση f ισχύει το Θ. Bolzano στο [λ, μ] , άρα θα υπάρχει μια τουλάχιστον ρίζα στο (λ, μ), δηλαδή μια τουλάχιστον πραγματική ρίζα.

• Αν αν < Ο τότε lim f(x) = lim (αν χ ν ) = +οο , χ �-οο χ �-οο

άρα υπάρχει κ < Ο τέτοιο ώστε f(κ) > Ο. Επίσης lim f(x) = lim (αν χ ν ) = --οο , άρα υπάρχει ρ > Ο

χ -+t<JO χ -+t<JO

τέτοιο ώστε f(ρ) < Ο . Για τη συνάρτηση f ισχύει το Θ. Bolzano στο [κ, ρ] , άρα θα υπάρχει μια τουλάχιστον ρίζα στο (κ, ρ), δηλαδή μια τουλάχιστον πραγματική ρίζα.

Πρόταση 5

Έστω συνάρτηση f : [α, β ) � JR . Αν η f είναι συ­

νεχής και «1 - 1)) στο [α, β) . Τότε η f είναι γνη­σίως μονότονη στο [α, β).

Απόδειξη Η f « ι - ι )> στο [α, β] άρα f(α) :;t: f(β) . Έστω f(α) < f(β) , θεωρούμε χ 1χ2 ε (α, β) με χ ι < χ2 τότε f(x 1 ) :;t: f(x2 ) . Ισχύει α < Χ ι < Xz < β και για τα: f(α), f(x 1 ), f(x2 ), f(β) θα ισχύει μία από τις σχέσεις. 1 ) f(χ ι ) < f(α) < f(β) 2) f(α) < f(β) < f(x2 )

3) f(x2 ) < f(α) < f(β) 4) f(α) < f(β) < f(χι ) 5) f(α) < f(x2 ) < f(x 1 ) < f(β) 6) f(α) < f(x ι ) < f(x2 ) < f(β) I ) f(x1 ) < f(α) < f(β)

Έστω f : [x ι , β] � JR , η f συνεχής στο [χ ι , β] και f(x 1 ) < f(α) < f(β) άρα από το θεώρημα των ενδιαμέσων τιμών υπάρχει ένα τουλάχιστον ξ ε (χ 1 , β) με f(ξ) = f(α) . Επειδή η f « 1 - ι )> θα ισχύει ξ =α άτοπο γιατί α < χ 1 < ξ < β όμοια για τις περιπτώσεις (2), (3), (4), (5) άρα f(α) < f(x 1 ) < f(x2 ) < f(β) . Επομένως για κάθε χ 1 , χ 2 ε [α, β] με χ 1 < χ 2 � f(x 1 ) < f(x2 ) άρα η Ο στο [α, β] . Όταν f(α) > f(β) τότε η Π στο [α, β] .

Εφαρμογές

1 . Έστω f : JR � JR με f(x) = xν - 2xv-t + 3, ν ε Ν, ν � 2 . Δείξτε ότι υπάρχει ένα τουλάχιστον ξ ε (0,1) τέτοιο ώστε f(ξ + 1) = f(ξ)

Υπόδειξη

Στη συνάρτηση h(x) = f(x + 1) - f(x) εφαρμόζου-με το Θ. Bolzano στο διάστημα [0, ι ] . 2 . Έστω f : JR � JR συνάρτηση συνεχής στο

ι ι - z z 12 R με f\x) + jz1 - z2 j f

2 (x) + 1 2 f(x) = 4

2χ5 + χ4 - 2 για κάθε χ ε JR όπου z1z2 σταθεροί μιγαδικοί οι εικόνες των οποίων είναι εσωτερικά σημεία του κύκλου χ2 + y2

= 1 . Αποδείξτε ότι α) lzι - z2 j < j ι - zιz2 j . β) η εξίσωση f(x) = Ο έχει τουλάχιστον μία

ρίζα στο (0, 1). Απόδειξη

α) Επειδή οι εικόνες των μιγαδικών z1 , z2 είναι εσωτερικά σημεία του κύκλου χ2 + y2 = ι θα ισχύει l zι l < ι και l z2 1 < 1 . Άρα ( ι - Ιz1 1 2 )( 1 - lz2 j2 ) > Ο � (1 - z1z1 ) ( 1 - z2� ) > Ο � 1 - z2� - z1z1 + z1z1z2z2 > Ο �

ΕΥΚΛΕΙΔΗΣ Β ' λθ ' τ.2/63

Page 66: Ευκλειδης Β 58

Μαθηματικά για την Γ Λυκείου

<::::::> 1 + z1 z2 � 'Zz > z1 z1 + z2 'Zz <::::::> <::::::> 1 - z1 'Zz -z1 z2 + z1 z2 z1 'Zz > > z, z, + z2 z2 - z, z2 - z, z2 <::::::> <::::::> 1 - z1z2 - z1 z2 ( 1 - z1z2 ) > > z, (z, - z2 ) - z2 (z, - z2 ) <::::::> <::::::> ( 1 - z, z2 ) ( 1 - z, z2 ) > ( z, - z2 ) ( z, - z2 ) <::::::> <::::::> ( 1 - ΖιΖ2 ) ( 1 - Ζι Ζ2 ) > ( Ζι - Ζ2 ) ( Ζι - Ζ2 ) <::::::> <::::::> 1 1 - z,z2 12 > lzι - z2 12 <::::::> 1 1 - z, z2 1 > l zι - z2 1

β) Για κάθε χ ε JR ισχύει [ r' (x) + lz, - z, Ι r<x) + Ι ι -�z, Ι' }(χ) =

= 2χ5 + χ4 - 2 επειδή, 2 1 ι -�z2 Ι 1 2 I _

1 2 Δ = lz1 - z2 1 - 4 = lz1 - z2 - 1 - Ζιz2 < 0 4 <=> Ιz, - z2 Ι < Ι ι - z, z2 1 ισχύει από το πρώτο ε-ρώτημα.

I - 1 2 1 - z1 z2 Άρα f2 (x) + lz1 - z2 l f(x) + > 0 για 4 κάθε χ ε JR , επομένως f(O) < Ο και f(l) > Ο . Για τη συνάρτηση f εφαρμόζουμε το Θ. Bol­zano στο διάστημα [0, 1 ] • Η fσυνεχής στο [Ο, 1 ] . • f(O)f(l) < Ο άρα υπάρχει ένα τουλάχιστον χ0 ε (Ο, 1) : f(χ0 ) = 0

3. Έστω μια συνάρτηση f : [α, β] � JR συνε­χής στο [α, β] και οι μιγαδικοί z = α2 + if(α), w = f(β) + iβ2 , αβ :;t: Ο .

Αν lz l2 + lw l2 = lw - z l2 τότε η εξίσωση f(x) = Ο έχει μια τουλάχιστον ρίζα στο διά-στημα [α, β] . (lη Δέσμη 1995)

Απόδειξη

l zl 2 + lwl2 = lz - wl2 <:=:> zz + w\V = (z - w)(z - w) <=> <::::::> zz + wW = zz - zW - wz + wW <::::::> zW + wz = ο <::::::> zW + zW = Ο <::::::> 2 Re( zW) = Ο <::::::> Re( zW) = Ο . Είναι zW = [α2 + if(α)] [f(β) - iβ2 ] = = [ α2f(β) + β2f(α) J + [ f(β)f(α) - α2β2}

α2 άρα α2f(β) + β2f(α) = Ο <::::::> f(α) = --2 f(β) β

Αν f(β) = Ο τότε και f(α) = Ο άρα χ = α και χ = β ρίζες της εξίσωσης f(x) = Ο . Αν f(β) :;t: O τότε και f(α) :;t: O . Για τη συνάρτηση f εφαρμόζουμε το Θ. Bolzano στο [α,β] . • Η f συνεχής στο [α, β] .

α2 [ α ]2 • f(α)f(β) = -β2f(β) · f(β) = - βf(β) < Ο

άρα υπάρχει ένα τουλάχιστον χ0 ε (α, β) τέτοιο ώστε f(x0 ) = Ο . Επομένως η εξίσωση f(x) = Ο έ­χει τουλάχιστον μία ρίζα στο [α,β] . 4. Δίνεται η συνεχής συνάρτηση f : JR � JR με

f(O) :;t: Ο και lf(x) l :::;; 2005 για κάθε χ Ε JR •

Δείξτε ότι η εξίσωση f2 (x) = χ2 έχει δύο τουλάχιστον πραγματικές ρίζες.

Απόδειξη

Για κάθε χ Ε JR είναι lf(x) l :::;; 2005 <::::::> -2005 :::;; f(x) :::;; 2005 <::::::> {f(x) + 2005 :?: Ο <::::::> . Θεωρούμε τη συνάρτηση f(x) - 2005 :::;; Ο

g(x) = [ f(x) - χ] · [ f(x) + χ ] , χ ε [ -2005, 2005] Η g είναι συνεχής στο διάστημα [ -2005, 0] g( -2005)g(O) = = [f(-2005) + 2005] [f(-2005) - 2005] . f2 (0) :::;; ο

� <:0 so >0 Διακρίνουμε περιπτώσεις • Αν g( -2005)g(O) = Ο , τότε g( -2005) = Ο άρα

το - 2005 είναι ρίζα της g(x) = Ο. • Αν g( -2005)g(O) < Ο , τότε ισχύει θεώρημα

Bolzano, οπότε η εξίσωση g(x) = Ο έχει μια τουλάχιστον ρίζα στο διάστημα (- 2005 , 0). Άρα η εξίσωση g(x) = Ο <::::::> f2 (χ) = χ2 έχει μία τουλάχιστον ρίζα στο διάστημα [- 2005,0). Ομοίως δείχνουμε ότι η εξίσωση έχει και μια τουλάχιστον ρίζα στο διάστημα ( Ο, 2005] . Ε-πομένως η εξίσωση f2 (x) = χ2 έχει δύο τουλά­χιστον πραγματικές ρίζες.

5. Δίνεται η συνεχής συνάρτηση f : (O,-+«J)�JR.

Αν f(α) = l και f(x)f(y) + f(�)r(�) = 2f(xy)

ΕΥΚΛΕΙΔΗΣ Β ' λθ ' τ.Ι /64

Page 67: Ευκλειδης Β 58

Μαθηματικά για την Γ Λυκείου

για κάθε x, y Ε (Ο,+οο) και α > Ο, να δείξετε ότι η f είναι σταθερή.

Απόδειξη Για χ = y = 1 ισχύει

f2 (1) + f2 (α) = 2f(l) � ( f(l) - 1 )2 = Ο � f(l) = 1 Για y = 1 και χ Ε (0, +οο) έχουμε f(x)f(l) + r (: J r(α) = 2f(x) �

f(x) + f (:J = 2f(x) � f(x) = f (: J ( 1 )

Για y = � και χ Ε (0, +οο) έχουμε χ

f(x)f ( � J + r (: J r(x) = 2f(α) �

2f(x)f (:J = 2 � f(x) (�J = 1 (2)

Από ( 1 ) και (2) έχουμε f2 (χ) = 1 για κάθε χ Ε (Ο, +οο) άρα f(x) :;t: Ο για κάθε χ Ε (Ο, +οο) . Έχουμε λοιπόν f(x) :;t: Ο και f συνεχής στο (Ο, +οο) άρα η f διατηρεί σταθερό πρόσημο στο (Ο, +οο) . Επειδή f(α) = 1 > Ο θα είναι και f(x) > Ο στο (Ο, +οο) . Επομένως f(x) = 1 για κάθε χ Ε (0, +οο) , δηλαδή η f είναι σταθερή συνάρτηση .

6. Έστω συνάρτηση f :IR�IR συνεχής και «1 - 1». Αν υπάρχει Χο E IR με (f o f o f} (:xo) = Xo , a-

ποδείξτε ότι f ( χ0 ) = χ0 •

Απόδειξη

Η f συνεχής και «1 - 1 » στο IR άρα γνήσια μονό­τονη στο IR (πρόταση 5) . Έστω Ο στο IR . Υποθέτουμε ότι f(x0 ) :;t: χ0 • Τότε f(x0 ) > χ0 ή f(x0 ) < χ0 •

η Αν f(x0 ) > χ0 � f(f(x0 )) > f(x0 ) > χ0 άρα f(f(f(x0 ))) > f(x0 ) � χ0 > f(x0 ) άτοπο ό­μοια όταν f(x0 ) < χ0 . Ανάλογα εργαζόμαστε και όταν Π στο 1R . Άρα f(x0 ) = χ0 . Π ροτεινόμενα για λύση θέματα

1 . Έστω η εξίσωση χ2 + χ + t = Ο, t Ε IR και η συ­νάρτηση f : IR � IR με f(t) = l x1 l + lx2 1 όπου χ 1 , χ 2 οι ρίζες της εξίσωσης. α) Να μελετηθεί ως προς τη συνέχεια. β) Να αποδείξετε ότι η εξίσωση

f(x) + ln f(x) = 2 - χ έχει μία τουλάχιστον ρίζα στο ( � , e 1 1 4 J .

2. Δίνονται οι συνεχείς συναρτήσεις f,g : [α, β] � IR με f(β) = g(α) = Ο και f(x) + g(x) :;t: O για κάθε χ Ε [α,β] . Να απο­δείξετε ότι υπάρχει χ0 Ε (α, β) με f(x0 ) = g(x0 ) .

Βιβλιογραφία 1. Απειροστικός λογισμός Ι (Σωτήρη Κ. Ντούγια) . 2. Περιοδικό GAZET Α ΜΑ ΤΗ ΕΜΑ ΠCΑ.

ΜΑΘΗΜΑ Τ/ΚΑ ΓΕΝΙΚΗΣ ΠΑΙΔΕΙΑΣ

Ασκήσεις Στατιστικής των Παναγιώτη Στεφανή, Δημήτρη Βαμβακίδη, Βασίλη Καρκάνη

ΑΣΚΗΣΗ 1 Στον παρακάτω πίνακα έχουμε τη γραπτή βαθ­μολογία που πέτυχαν 30 μαθητές της Γ' Λυκεί­ου σε εξέταση στο μάθημα της Στατιστικής (σε ακέραιες τιμές).

ΕΥΚΛΕΙΔΗΣ Β' λθ ' τ.2/65

Βαθμολογία χ;

Χι 10 13 16 18 Χ6

Συχνότητα ν;

2 5 ν3 10 3 ν6

Page 68: Ευκλειδης Β 58

Μαθηματικά για την Γ Λυκείου

Αν ισχύουν: χ1 < 10, χ6 > 18 , το εύρος της βαθ­

μολογίας είναι R = 1 1 , η μέση τιμή της βαθμο­λογίας είναι χ = 1 4, 2 και η διάμεσος δ = 14,5 τότε να βρεθούν: α. Ο αριθμός των μαθητών που έγραψε 13. β . Ο αριθμός των μαθητών που έγραψε την

ψηλότερη βαθμολογία. γ. Η χαμηλότερη και η ψηλότερη βαθμολογία. δ. Ο αριθμός και το ποσοστό των μαθητών που

έγραψε το πολύ 16. ε. Ο αριθμός και τον ποσοστό των μαθητών

που έγραψε άριστα (τουλάχιστον 18). στ.Να γίνει το κυκλικό διάγραμμα των παραπά­

νω γραπτών βαθμών. Λύση

1 3 + 1 6 α. Εφόσον η διάμεσος είναι δ = 1 4, 5 = 2 και

το πλήθος των παρατηρήσεων 30 συμπεραί­νουμε ότι 1 5 παρατηρήσεις είναι μικρότερες του 14,5 και οι υπόλοιπες 1 5 παρατηρήσεις με­γαλύτερες του 1 4,5 . Έτσι ν, + ν2 + ν3 = 1 5 ή ν 3 = 8 οπότε 8 μαθητές έγραψαν 1 3 .

β . Επίσης: ν 4 + ν 5 + ν 6 = 1 5 ή 1 0 + 3 + ν 6 = 1 5 ή ν 6 = 2 οπότε την ψηλότερη βαθμολογία χ6 έ-γραψαν 2 μαθητές.

γ. Ακόμη R = 1 1 ή χ6 - χ 1 = 1 1 ή Χ 6 = χ 1 + 1 1 ( 1 ) οπότε ο πίνακας που δόθηκε γράφεται: Βαθμολογία χ; Συχνότητα ν; X;V;

Χι 2 2χ, 10 5 50 13 8 104 16 10 160 18 3 54

χ1+1 1 2 2χ,+22 ΣΥΝΟΛΟ 30 4χ, + 390

Σ χίνί - 4χ , + 390 Όμως χ Σν; - 30 Εφόσον χ = 1 4, 2 παίρνουμε: 4χ , + 390 = 14, 2 <=:> . . . <=:> χ, = 9 και από την 30 ( 1 ) : χ6 = 9 + 1 1 = 20 . Δηλαδή η χαμηλότερη βαθμολογία είναι 9 και η ψηλότερη 20.

δ. Σύμφωνα με τα παραπάνω ο πίνακας συχνοτή­των της βαθμολογίας είναι:

Χ; V; Ν; f; F; F;% 9 2 2 0,06 6 6 1 0 5 7 0, 1 7 1 7 23 1 3 8 1 5 0,27 27 50 1 6 1 0 25 0,34 34 84 1 8 3 28 0, 1 0 1 0 94 20 2 30 0,06 6 1 00

ΣΥΝ ΟΛΟ 30 1 1 00 οπότε ο αριθμός των μαθητών που έγραψε το πο­λύ 1 6 είναι 25 και το αντίστοιχο ποσοστό 84%.

ε. Από τον προηγούμενο πίνακα είναι φανερό ότι άρι­στα (τουλάχιστον 1 8) έγραψαν 5 μαθητές ή το 16%.

στ.Για το μέτρο της γωνίας κάθε κυκλικού τομέα στο κυκλικό διάγραμμα ισχύει: α; = 360' f; . Έτσι: α, = 360' · f, = 360' · 0, 06 "" 22'

α2 = 360' · f2 = 360' · Ο, 1 7 "" 6 1' α3 = 360' · f3 = 360' · 0, 27 "" 97' α4 = 360' · f4 = 360' · 0, 34 "" 1 22' α5 = 360' · f5 = 360' · Ο, 1 0 "" 36' α6 = 360' · f6 = 360' · 0, 06 "" 22'

οπότε έχουμε το παρακάτω κυκλικό διάγραμμα στο οποίο κάθε κυκλικός τομέας γωνίας α; α­ντιστοιχεί στο βαθμό χ;, Ι = 1 , 2, . . . , 6 .

χ4= 1 6 χ3= 1 3

ΑΣΚΗΣΗ 2 Δείγμα 50 τηλεθεατών ρωτήθηκε ποιο είναι το αγαπημένο τους κανάλι. Οι απαντήσεις φαίνο­νται στο ν παρακατω πινακα.

Όνομα καναλιού Συχνότητα

κανάλι 1 17 κανάλι 2 6 κανάλι 3 19 κανάλι 4 3 κανάλι 5 5

ΕΥΚΛΕΙΔΗΣ Β ' λθ ' τ.2/66

Page 69: Ευκλειδης Β 58

Μαθηματικά για την Γ Λυκείου

α. Να παραστήσετε γραφικά με ραβδόγραμμα τα δεδομένα αυτά και να βρείτε το ποσοστό των τηλεθεατών που είχε ως αγαπημένο το κανάλι 2.

β. Το ίδw δείγμα ρωτήθηκε για το πόσες ώρες ε­βδομαδιάία παρακολουθεί τηλεόραση. Στο παρακάτω σχήμα δίνεται το πολύγωνο α­θροιστικών σχετικών συχνοτήτων (F;%) της μεταβλητής: "εβδομαδιαίες ώρες τηλεθέασης".

F;%

\ 00

80

46

22

\ 0

ο 2 4 6 8 \ 0 ώρες εβδομαδιαίας τηλεθέασης

ί. Να βρείτε το ποσοστό των τηλεθεατών του δείγματος που παρακολουθούν εβδο­μαδιαία 6 ή περισσότερες ώρες.

ίί. Να βρείτε τον αριθμό των τηλεθεατών του δείγματος που παρακολουθούν εβδο­μαδιαία λιγότερες από 6 ώρες και περισ­σότερες ή ίσες από 4 ώρες.

Λύση

α. Το ραβδόγραμμα συχνοτήτων είναι: ν;

1 9 1 7

6 5 3

� Ω

;:;<:: ;:;<:: :r:: � Ν (...ι :ι,. Vl

Το ζητούμενο ποσοστό είναι � = Ο, Ι 2 ή Ι 2%. 50 β. ί. Το ζητούμενο ποσοστό είναι ίσο με: f4% + f5 % = F5 % - F3% = Ι 00 - 46 = 54 άρα 54%

ii . Ο ζητούμενος αριθμός είναι ίσος με τη συ­ν χνότητα ν3 όμως f3 = 2 <:::::> ν 3 = 50 · f3 ν

και f3 % = F3% - F2% = 46 - 22 = 24 άρα 24%

24 άρα ν = 50 · - = Ι 2 . 3 1 00 ΑΣΚΗΣΗ 3

Να βρεθεί η μέση τιμή των αριθμών 2, 6, 10, 14, . . . , 398.

Λύση

Οι αριθμοί 2, 6, 1 0, Ι 4, . . . , 398 αποτελούν αριθμη­τική πρόοδο με α1 = 2, ω = 4 και αν = 398 . Επίσης αν = αι + (ν - Ι) · ώ οπότε : 398 = 2 + (ν - Ι) · 4 <:::::> ν = 1 00 Για το άθροισμα των Ι 00 όρων της αριθμητικής προόδου 2, 6, 1 0, Ι 4, . . . , 398 έχουμε:

(α1 + αν ) · ν _ (2 + 398) · 1 00 , Σ = ή Σι οο - η ν 2 2 1 00 Σ χί 2οοοο Σ1 00 = 20000 Έτσι: χ = � ή χ = -- = 200 . 1 00 1 00

η ζητούμενη μέση τιμή . ΑΣ Κ Η Σ Η 4

Στο παρακάτω σχήμα φαίνονται το διάγραμμα συχνοτήτων για τη μεταβλητή Χ: ηλικία παι­διών που συμμετείχαν σε μια γιορτή.

Yi 7

;> 6 ·:ΞΙ i 5 � 4

·Ο

� 3 g. 2

ι I I I ο ι 2 3 4 5 6 7 8 9 ι ο ι ι ι2 ι 3 ι4 ι 5

ηλικία παιδιών

Α. α. Να βρείτε το μέγεθος του δείγματος και τη διάμεσο.

β. Να εξετάσετε το είδος της aσυμμετρίας που παρουσιάζει το διάγραμμα.

Β. Θεωρούμε το παραπάνω δείγμα ηλικιών των παιδιών από το οποίο λείπει μόνο το παιδί ηλικίας 15 ετών. Επίσης θεωρούμε ότι η συ­χνότητα κάθε ηλικίας είναι 1 (και όχι αυτή του διαγράμματος που έχει δοθεί). Να εξη­γήσετε ποια επίπτωση θα έχει αυτή η απου­σία στη μέση τιμή της ηλικίας του νέου δείγματος σε σχέση με την αντίστοιχη του αρχικού δείγματος.

ΕΥΚΛΕΙΔΗΣ Β' λθ ' τ.2/67

Page 70: Ευκλειδης Β 58

Μαθηματικά για την Γ Λυκείου

Λύση Α. α. Το μέγεθος του δείγματος είναι:

ι ι ν = Σ ν; = 2 + 4 + 6 + 7 + 6 + 5

i= ι +4 + 2 + 1 + 1 + 1 = 39

Η διάμεσος είναι ίση με την τιμή της 20ης παρατήρησης άρα δ = 6. (γιατί;)

β. Όπως φαίνεται από το διάγραμμα υπάρχει θετική ασυμμετρία.

Β. Η μέση τιμή επηρεάζεται από τις ακραίες τιμές, γι ' αυτό η απουσία της ακραίας τιμής 1 5 , θα έ­χει σαν συνέπεια η μέση τιμή που θα προκύψει να εκφράζει με περισσότερη ακρίβεια την κε­ντρική τιμή των παρατηρήσεων.

ΑΣ Κ Η Σ Η 5 Ο τρόπος εισαγωγής στα A.E.I. σε μια χώρα απαιτεί γραπτές εξετάσεις σε 1 Ο μαθήματα. Ο προφορικός βαθμός στα αντίστοιχα μαθήματα δεν λαμβάνεται υπόψιν. Για να εισαχθεί ένας μαθητής σε μια συγκεκριμένη σχολή χρειάζεται μέσο όρο βαθμολογίας στα γραπτά 17. Ο μαθητής στα πρώτα 7 μαθήματα έχει μέση βαθμολογία 14 και του μένουν για εξέταση ακό­μη 3 μαθήματα. Είναι δυνατό ο μαθητής με κα­τάλληλες επιδόσεις σε αυτά τα 3 μαθήματα που απομένουν, να μπει στη σχολή αυτή;

Λύση Έστω χ ι , χ 2 , χ 3 οι γραπτοί βαθμοί που θα πάρει ο μαθητής στα τρία μαθήματα που του έχουν απο­μείνει ακόμη να εξεταστεί. Έχουμε: χ = 1 7 <=:> ι ο

Σ χί 7

J=L_ = 1 7 <=:> Σ Χ ; + Χ ι + Χ 2 + χ3 = 1 70 ( 1 ) 1 0 i= ι 7

Σχί 7

Όμως 2=L_ = 1 4 <=:> Σ χ ; = 98 . 7 i= ι Έτσι η ( 1 ) γράφεται: 98 + Χ ι + χ2 + χ3 = 1 70 <=:> Χ ι + χ2 + χ3 = 72 . Για το βαθμό χ; κάθε μαθήματος ισχύει: Χ ; � 20 άρα χ ι + χ 2 + χ 3 :ς 60 συνεπώς ο μαθητής ακόμη και αν γράψει τρία 20άρια δεν μπορεί να εισαχθεί στη σχολή αυτή .

ΑΣΚΗΣΗ 6

Η εθνική ομάδα μπάσκετ που κατέκτησε την πρώτη θέση στο Ευρωπαϊκό πρωτάθλημα μπά­σκετ του 2005 είχε στη σύνθεσή της 4 παίχτες που αγωνίζονταν στο ελληνικό πρωτάθλημα, οι οποίοι είχαν μέσο όρο ηλικίας 23,5 έτη, Οι υπό­λοιποι παίχτες αγωνίζονταν σε διάφορες χώρες της Ευρώπης και είχαν μέσο όρο ηλικίας 25 έτη. Επίσης δίνεται ότι ο μέσος όρος ηλικίας όλων των παιχτών της εθνικής ομάδας ήταν 24,5 έτη. α. Να βρείτε πόσοι παίχτες συμμετείχαν στην

εθνική ομάδα. β. Αν υποτεθεί ότι στο αμέσως επόμενο Ευρω­

παϊκό πρωτάθλημα που θα γίνει σε δύο α­κριβώς χρόνια, η Εθνική θα μετέχει με τους ίδιους ακριβώς παίχτες, με εξαίρεση ένα παί­χτη που αποσύρεται ηλικίας τότε 30 ετών και ο οποίος αντικαθίσταται από άλλο παίχτη η­λικίας τότε 20 ετών να βρείτε τη νέα μέση ηλικία της ομάδας.

Λύση

α. Έστω ν ο αριθμός των παικτών της εθνικής ο­μάδας. Είναι:

4 ν

Σχ; + Σ χί χ = 24, 5 <=:> i = ι i=S = 24, 5 ( 1 )

Όμως:

ν

ν

Σ χ . 1 ν

Επίσης: ..i.=L_ = 25 <=:> Σ Χ ; = 25ν - 1 00 . ν - 4 i=s Έτσι η ( 1 ) δίνει:

94 + 25ν - 1 00 = 24, 5 <=:> ν = 1 2 . ν Άρα μετείχαν στην Εθνική 1 2 παίχτες.

β. Η μέση ηλικία των 1 2 παιχτών που μετείχαν στο φετινό πρωτάθλημα σε 2 χρόνια θα είναι: y = x + 2 ή y = 24, 5 + 2 = 26, 5 έτη .

ι 2 Σ Υ; ι 2

Συνεπώς: 2=L_ = 26, 5 <=:> Σ Υ; = 3 1 8 1 2 i=ι Άρα ψ' η νέα μέση ηλικία της ομάδας θα έχουμε:

ΕΥΚΛΕΙΔΗΣ Β' λθ ' τ.2/68

Page 71: Ευκλειδης Β 58

Μαθηματικά για την Γ Λυκείου

1 1 Σ Υ; - 30 + 20

y' = ...!.j=..!.- 1 ____ _

1 2 3 1 8 - 1 0 = 25 67 ' , ετη, 1 2

η ζητούμενη νέα μέση ηλικία της ομάδας. ΑΣ Κ Η Σ Η 7

Στο παρακάτω σχήμα φαίνεται το ιστόγραμμα σχετικών συχνοτήτων f;% του ετήσιου εισοδή­ματος σε χιλιάδες ευρώ, των εργαζομένων σε μια επιχείρηση.

f,%

20 --, 1 0

1 0 1 2 1 4 1 6 1 8 20 εισόδημα σε χιλ. ευρώ

Από το ιστόγραμμα λείπουν τα ορθογώνια που α­ντιστοιχούν σε δύο κλάσεις. Α ν η μέση τιμή του ετήσιου εισοδήματος είναι 15200 ευρώ τότε: α. Να βρείτε τις συχνότητες f3, f4 που λείπουν. β. Η διοίκηση της επιχείρησης αποφάσισε την

χορήγηση επιδόματος θέρμανσης σε όσους από τους εργαζόμενους έχουν έσοδα ετήσια μικρό­τερα από 16.000 ευρώ. Βρείτε το ποσοστό των εργαζομένων που δικαιούνται το επίδομα.

γ. Αν γνωρίζουμε ότι το εμβαδόν του χωρίου που ορίζεται από το πολύγωνο συχνοτήτων και τον οριζόντιο άξονα είναι 40 τότε να βρείτε τον α­ριθμό των οικογενειών που δικαιούνται το επί­δομα του προηγούμενου ερωτήματος.

δ. Α ν η διοίκηση της εταιρείας αποφασίσει να αυξήσει τους μισθούς όλων των εργαζομένων κατά 1 0% από την επόμενη χρονιά. Να βρεί­τε ποια θα είναι η μέση τιμή του ετήσιου ει­σοδήματος, μετά την αύξηση.

Λύση α, Ισχύει: f1 + f2 + f3 + f4 + f5 = 1 <:::::>

Ο, 1 + Ο, 2 + f3 + f4 + Ο, 1 = 1 <:::::> f3 + f4 = Ο, 6 ( 1 ) 5

Ακόμη είναι: χ = 1 5 , 2 <:::::> Σ χ;f; = 1 5 , 2 <:::::> i= l

1 1 · 0, 1 + 1 3 · 0, 2+ 1 5 . f3 + 17 . f4 + 1 9 · 0, 1 = 1 5, 2 =>

1 5f3 + 1 7f4 = 9, 6 (2) Από το σύστημα των ( 1 ) και (2) βρίσκουμε f3 = f4 = 0, 3 .

β. Είναι: f1 + f2 + f3 = Ο, 1 + Ο, 2 + Ο, 3 = Ο, 6 , οπότε το ζητούμενο ποσοστό είναι ίσο με 60%.

γ. Το εμβαδόν του χωρίου αυτού είναι ίσο με το μέγεθος ν του δείγματος, οπότε ν = 40. Έτσι ο ζητούμενος αριθμός είναι: 60 · 40 = 24 οικογένειες. 1 00

δ. Α ν χ; με ί = 1 , 2, . . . , 40 οι μισθοί των εργαζο­μένων πριν την αύξηση τότε οι νέοι μισθοί θα είναι:

1 0 ψ. = χ . + -χ. = 1, 1χ . . ι ι 1 00 ι ι

Συνεπώς η νέα μέση τιμή θα είναι: y = 1 , 1 · χ = 1, 1 · 1 5 , 2 = 1 6, 72

ή 1 6 .720 ευρώ.

ΑΣ Κ Η Σ Η 8

Α. Σε δύο τμήματα της Γ ' Λυκείου που το κα­θένα έχει 20 μαθητές η μέση τιμή του βαθ­μού πρόσβασης είναι 14 στο καθένα και οι τυπικές αποκλίσεις αντίστοιχα 3 και 4,5. α. Να βρείτε σε ποιο τμήμα αναμένεται να

υπάρχουν περισσότερες επιτυχίες σε Α.Ε.Ι. αν θεωρηθεί ότι για την εισαγωγή σε ΑΕΙ χρειάζεται βαθμός μεγαλύτερος από 1 7. Να τεκμηριώσετε την απάντησή σας. Θεωρείστε ότι η κατανομή της βαθ­μολογίας είναι κανονική.

β. Να συγκρίνετε την ομοιογένεια των δύο τμημάτων.

Β. Θεωρούμε το δείγμα που αποτελείται από τους μαθητές και των δύο τμημάτων. Έστω z η μέση τιμή του βαθμού πρόσβασης στο δείγμα αυτό και χ, y οι μέσες τιμές στα αρ-χικά τμήματα, τότε

Ν δ 'ξ , - χ + y α. α απο ει ετε οτι z = -- .

2 β. Να εξετάσετε αν ισχύει πάντα

- x + y ' θ , z = -2- σε κα ε περιπτωση

η σχέση

που δη-

μιουργούμε ένα νέο δείγμα που προέρχε­ται από δύο άλλα δείγματα όπως στην πε­ρίπτωση του ερωτήματος α.

ΕΥΚΛΕΙΔΗΣ Β ' λθ ' τ.2/69

Page 72: Ευκλειδης Β 58

Μαθηματικά για την Γ Λυκείου

Λύση Α. α. Η διακύμανση εκφράζει τη «συγκέντρωση»

των τιμών της μεταβλητής γύρω από τη μέ­ση τιμή . Συνεπώς στο τμήμα που έχει S = 4,5 αναμένεται να υπάρχουν περισσότεροι μαθητές με βαθμό μεγαλύτερο του Ι 7, σε σχέση με το τμήμα όπου S = 3 .

β . Είναι: 3 < 4, 5 <:=:> _2_ < 4' 5 άρα CV1 < CV2 • Ι 4 Ι4 Συνεπώς το τμήμα με S = 3 είναι πιο ομοιο­γενές σε σχέση με το τμήμα όπου S = 4,5 .

Β. α. Επίσης

β. Ο ισχυρισμός δεν ισχύει πάντα. Α ν θεωρή­σουμε δύο δείγματα μεγέθους ν , , νz αντί­στοιχα με ν 1 =1- ν 2 και μέσες τιμές χ και y

νι v

2 Σχ; + Σ Υ; αντίστοιχα, τότε: z = i= ι i= ι

ν ι + νz

- - Σ χ ί Σ Υ; χ + Υ Ι ;= ι i= ι

[ ν

1 ν

2 ] και -- = - -- + --2 2 ν1 ν2

, , - x + y οποτε προφανως: z =1- --2 ΑΣ Κ Η Σ Η 9 Δυο τμήματα Σ1 και Σ2 μεταπτυχιακών φοιτητών του τμήματος Στατιστικής έχουν μέση γραπτή βαθμολογία στο μάθημα των πιθανότητων του 1 ου εξαμήνου χ = 6 και τυπική απόκλιση s = 0.8. Το 2° εξάμηνο όλοι οι φοιτητές του τμήματος Σ, αύξησαν το γραπτό βαθμό τους στο ίδw μάθημα κατά 1,2 μονάδες. Επίσης στο τμήμα Σ2 όλοι οι φοιτητές αύξησαν το γραπτό βαθμό τους στο ίδw μάθημα κατά 20%. Ποια από τις δύο αυξήσεις θεωρείται καλύτερη για έναν «καλό» φοιτητή και ποια για έναν «αδύνατο»; Να αιτιολογήσετε την απάντησή σας.

Λύση

Στο τμήμα Σ1 κάθε φοιτητής αύξησε τη γραπτή του βαθμολογία το 2° εξάμηνο κατά Ι ,2 μονάδες οπότε η νέα μέση βαθμολογία έγινε: χ1 = χ + Ι, 2 = 6 + Ι, 2 = 7, 2 μονάδες και η τυπική απόκλιση s 1 = s = 0, 8 . Στο τμήμα Σ2 η αντίστοιχη αύξηση είναι 20% οπό­τε η νέα μέση βαθμολογία έγινε: χ2 = χ + 0, 2χ = Ι, 2χ = Ι , 2 · 6 = 7, 2 μονάδες και η τυπική απόκλιση s2 = 1, 2 · s = 1, 2 · Ο, 8 = Ο, 96 .Από τα παραπάνω παρατηρούμε ότι: χ1 = χ2 και s 1<s2, Ένας «καλός» φοιτητής θα ήθελε να "απομακρύ­νει" το βαθμό του από τη μέση βαθμολογία, οπότε θα ήθελε να είναι στο τμήμα με τη μεγαλύτερη τυ­πική απόκλιση, δηλαδή στο Σ2 • Επίσης ένας «αδύ­νατος» φοιτητής θα ήθελε το βαθμό του "κοντά" στο μέσο όρο οπότε θα τον συνέφερε το τμήμα με τη μικρότερη τυπική απόκλιση Σ1 • ΑΣΚΗΣΗ Ι Ο

Η κατανομή των υψών των αγοριών της Γ Λυ­κείου σε ένα σχολείο είναι κανονική. Το 68% των μαθητών έχουν ύψος (σε εκατοστά) που α­νήκει στο διάστημα (175, 181), ενώ οι πιο πολ­λοί μαθητές έχουν ύψος 178 cm. α. Να βρείτε τη μέση τιμή και την τυπική από­

κλιση του ύψους των μαθητών. β. Να εξετάσετε αν είναι πιο πολλοί οι μαθητές

που έχουν ύψους 174 cm ή αυτοί που έχουν ύψος 182 cm.

γ. Να εξετάσετε αν είναι εύκολο το συγκεκριμέ­νο Λύκειο να συγκροτήσει ομάδα μπάσκετ, από μαθητές της Γ Λυκείου, με μέσο όρο ύψους 188 cm.

Λύση α. Στην κανονική κατανομή η μέση τιμή είναι η

τιμή με τη μεγαλύτερη συχνότητα, άρα x= 1 78 cm. Ακόμη ισχύει: x - S = 1 75 και χ + S = 1 8 1 άρα S = 3 .

β. Η κανονική κατανομή είναι συμμετρική γύρω από τη μέση τιμή συνεπώς οι μαθητές με ύψος 1 7 4 cm είναι όσοι και οι μαθητές με ύψος 1 78 cm.

γ. Το 99,7% των μαθητών έχει ύψος στο διάστη­μα (χ - 3S, χ + 3S) = (1 69, 1 87) άρα είναι δύ-

ΕΥΚΛΕΙΔΗΣ Β ' λθ ' τ.2/70

Page 73: Ευκλειδης Β 58

Μαθηματικά για την Γ Λυκείου

σκολο να συγκροτηθεί ομάδα μπάσκετ με μέσο όρο ύψους 1 88 cm.

ΑΣ Κ Η Σ Η 1 1 Η μέση τιμή και η τυπική απόκλιση μιας μετα­βλητής Χ είναι χ και s, αντίστοιχα. Α ν για τη μεταβλητή Ψ ισχύει η σχέση 2Χ + s, · Ψ = 2χ , να υπολογιστεί η μέση τιμή και η διακύμανση της Ψ, αν s, '* Ο .

Λύση

Από τη σχέση 2Χ + s, · Ψ = 2χ καταλαβαίνουμε ότι η σχέση που συνδέει τις παρατηρήσεις χ; με τις παρατηρήσεις ψ; είναι η εξής: 2χ; + s , · Ψ; = 2χ <::::>

2 2χ Γ 'ζ , Ψ; = -- · χ ι + -. νωρι ουμε οτι: s , s , αν Ψ; = αχ ; + β και α, β Ε IR τότε - - β 1 1 ' 2 2 2 ' ψ = αχ + , sΨ = α · s , οποτε: sΨ = α s. , αρα - 2 _ 2 _ , _ 0 ψ = -- · χ + - · χ η ψ = και s , s ,

? [ 2 )2 ? ' 2 4 s; = - - · s� η sΨ = . s ,

ΑΣ Κ Η Σ Η 1 2 Α ν οι μεταβλητές Χ, Ψ συνδέονται με τη σχέση Ψ = 2Χ2 - 3Χ + 4 και ψ = 21, s, = 3 να προσ-διορίσετε τη μέση τιμή της μεταβλητής Χ.

Λύση

Είναι Ψ = 2Χ2 - 3Χ + 4 οπότε Ψ; = 2χ� - 3χ ; + 4 ν ν

ΣΨ; ΣC2χ� - 3χ ; + 4) άρα ψ = ..i.=.L_ = i=ι =

ν

2Σ χ� = ί= ι

ν

ν ν ν ν

3Σχί Σ4 ί= ι ί= ι -- + --ν ν

ν

Σ χ� οπότε: ψ = 2J::.!__ - 3χ + 4 = 2 1 ( 1 ) ν

ν

Σ χ� Επίσης: s� = J::.!__ - χ2 (γιατί;) οπότε: ν

ν ν

Σ χ� Σχ; 9 = ..E.L._ - χ2 <=> ..E.L._ = 9 + χ2 (2) ν ν

Από την ( Ι ) λόγω της (2) παίρνουμε:

2(9 + χ2 ) - 3χ - 1 7 = ο <=> 2χ2 - 3χ + 1 = ο <=>

- 1 ' - 1 Χ = - η Χ = . 2

Π ροτεινόμενες Ασκή σεις

ι 3. Για είκοσι παρατηρήσεις μιας μεταβλητής Χ βρήκαμε χ = 50 . Διαπιστώθηκε όμως ότι οι δέκα παρατηρήσεις είχαν υπερεκτιμηθεί κατά 5 μονάδες η κάθε μια, ενώ οι άλλες εννιά εί­χαν υποεκτιμηθεί κατά 1 Ο μονάδες η κάθε μια, ενώ 1 παρατήρηση είχε σωστά εκτιμηθεί. Να βρεθεί η σωστή μέση τιμή .

Απάντη ση : χ = 52 1 4. Το άθροισμα πέντε παρατηρήσεων μιας ποσο­

τικής μεταβλητής Χ είναι 25 και το άθροισμα των τετραγώνων τους 200. Να υπολογιστεί η μέση τιμή, η διακύμανση και η τυπική από­κλιση μιας μεταβλητής Ψ αν γνωρίζουμε ότι Ψ = 2 · Χ + 5 .

Απάντη ση : ψ = 1 5, s� = 60, sΨ = J60 .

1 5. Μια ποσοτική μεταβλητή Χ όταν παίρνει τις τιμές t ι , t2, . . . , ts έχει χι = 8 και s ι = 6 . Η ίδια μεταβλητή όταν πάρει τις τιμές ρ ι , ρ2, . . . , ρ 1 2 έχει χ2 = 3 και s2 = 4. Ποια είναι η μέση τιμή χ , και η τυπική απόκλιση Sx της μεταβλητής Χ όταν πάρει συνεχόμενες τις τιμές t ι , t2, . . . , ts, ρ ι , ρ2, . . . , ρ ι 2 ;

Απάντηση : χ = 5, sx = J30 . 1 6. Να βρεθεί η μέση τιμή των αριθμών Ι · 2,

2 · 3, 3 · 4, . . . , ν(ν + l) . Υπόδειξη : Χρησιμοποιήστε την ταυτότητα ν( ν + 1) = ν2 + ν .

(ν + l) · (ν + 2) Απάντηση : ���--� 3

ι 7. Δίνονται οι μεταβλητές Χ και Ψ με μέση τιμή χ και ψ αντίστοιχα και τυπικές αποκλίσεις s., Sψ αντίστοιχα. Α ν Ψ = αΧ + βχ και Sx = Sψ

και υπάρχουν δύο τουλάχιστον παρατηρήσεις της μεταβλητής Χ διάφορες μεταξύ τους, να προσδιοριστούν τα α, β Ε IR έτσι ώστε χ = 2 και ψ = 4 .

ΕΥΚΛΕΙΔΗΣ Β ' λθ ' τ.2/71

Page 74: Ευκλειδης Β 58

τιjλιι rΙu

lttιBqτιj Επιμέλεια: Αντώνης Κυριακόπουλος - Γιάννης Στρατής

ΠΕΡΙ ΠΕΡΙΟΔΙ ΚΏΝ ΣΥΝΑΡτΗΣΕΩΝ

Ο μαθητής Νίκος Μπιζάνης (Β ' Τοσίτσειο Λύ­κειο Εκάλης) μας γράφει ότι, σύμφωνα με τον ορι­σμό της περιοδικής συνάρτησης (σχολικό βιβλίο Άλγεβρα Β ' τάξης, σελίδα 1 0), μια σταθερή συ­νάρτηση είναι περιοδική και μάλιστα με περίοδο οποιονδήποτε θετικό αριθμό (σωστό). Στη συνέ­χεια ρωτάει: «Ποια είναι η κύρια (βασική) περί­

οδος μιας σταθερής συνάρτησης» .

ΑΠΑΝτΗΣΗ. Αγαπητέ Νίκο. Το σχολικό σου βιβλίο, από τις περιοδικές συναρτήσεις, έχει μόνο τον ορισμό (και κείνον . . . ). Δεν έχει τον ορισμό της κύριας περιόδου και δεν «ξεκαθαρίζει» τα πράγμα­τα, με αποτέλεσμα να προκαλείται σύγχυση . Ας αφήσουμε λοιπόν το σχολικό σου βιβλίο και ας πάρουμε τα πράγματα από την αρχή, για να σχη­ματίσεις μια, όσο το δυνατόν, ολοκληρωμένη ει­κόνα για τις περιοδικές συναρτήσεις. Έτσι, θα λυ­θεί και η απορία σου. Ορισμός. Μία συνάρτηση f:A � JR ( Α * 0,

Α ς JR ) ονομάζεται (αμφίπλευρα) πε­ριοδική αν, και μόνο αν, υπάρχει πραγματικός αριθμός τ * ο τέτοιος, ώστε για κάθε χ Ε Α να ισχύουν:

(χ + τ) Ε Α,(χ - τ) Ε Α και f(x + τ) = f(x) .

Τότε, ο αριθμός Τ λέμε ότι είναι μία περίοδος της συνάρτησης f.

Ισχύει το εξής θεώρημα: Θεώρημα 1 . Έστω ότι μία συνάρτηση f είναι

περιοδική με μία περίοδο τον α­ριθμό τ ::�; Ο . Τότε, για κάθε ν Ε Ζ* , ο αριθμός νΤ είναι επίσης μία περίοδος της f.

Έτσι, κάθε περιοδική συνάρτηση έχει άπειρες περιόδους. Ορισμός. Έστω ότι μία συνάρτηση f είναι πε­

ριοδική. Αν το σύνολο των θετικών περιόδων αυτής έχει ελάχιστο στοι­χείο, τότε αυτό ονομάζεται κύρια (ή βασική ή πρωτεύουσα ή αρχική) πε­ρίοδος αυτής.

Έτσι, μια περιοδική συνάρτηση μπορεί να έχει κύρια περίοδο, μπορεί όμως και να μην έχει (όταν το σύνολο των θετικών περιόδων της δεν έχει ελά­χιστο στοιχείο) .

Το πόσο σημαντικό είναι μία περιοδική συνάρ­τηση να έχει βασική περίοδο, φαίνεται στο παρα­κάτω θεώρημα. Θεώρημα 2. Έστω ότι μία περιοδική συνάρτη­

ση f έχει κύρια περίοδο τον αριθμό Τ (οπότε τ > Ο).Τότε, κάθε άλλη

ΕΥΚΛΕΙΔΗΣ Β ' λθ ' τ.2!74

Page 75: Ευκλειδης Β 58

Η στήλη του Μαθητή

περίοδος της f είναι ένα ακέραιο πολλαπλάσιο του Τ (δηλαδή, είναι της μορφής νΤ, όπου ν Ε Ζ* ).

- Ας έλθουμε τώρα στην απορία σου. Μία σταθε­ρή συνάρτηση f(x) = c, όπου c Ε IR , έχει σύνολο ορισμού το IR και, όπως βρίσκουμε εύκολα, είναι περιοδική με μία περίοδο έναν οποιονδήποτε αριθ­μό =1:- Ο. Έτσι, το σύνολο των θετικών περιόδων αυτής είναι το σύνολο των θετικών αριθμών. Αλ­λά, το σύνολο αυτό δεν έχει ελάχιστο στοιχείο (δεν υπάρχει θετικός αριθμός μικρότερος ή ίσος από κάθε θετικό αριθμό).

Συνεπώς, η σταθερή συνάρτηση f(x) = c (είναι μεν περιοδική, αλλά) δεν έχει κύρια περίοδο (πε­ριόδους έχει άπειρες, κύρια περίοδο δεν έχει) .

Αγαπητέ Νίκο. Τα Μαθηματικά είναι ωραία, ευχάριστα και εύκολα (μην ακούς τι λένε), αρκεί να τα πάρεις από την αρχή και να «ξεκαθαρίσεις» τις διάφορες έννοιες. Συνέχισε να διαβάζεις και εμείς είμαστε στη διάθεσή σου για ό,τι χρειαστείς.

ΥΠΟΛΟΓΙΣΜΟΣ ΤΗΣ Δ ΥΝΑΜ ΗΣ ίν Ο μαθητής Βαγγέλης Πρωτοπαπαδάκης (3° Λύκειο Χανίων Κρήτης) μας έστειλε μία εργασία του, στην οποία διατυπώνει και αποδεικνύει μια πρότα­ση με την οποία μπορούμε να βρούμε ευκολότερα τη δύναμη iv (ν Ε Ν) , ιδιαίτερα όταν ο ν είναι με-γάλος αριθμός.

Καταρχήν, παρατηρεί ότι: ίν = iu , όπου υ είναι το υπόλοιπο της διαίρεσης ν : 4 (υ = Ο, 1 , 2, 3) . Έτσι, για να βρούμε τη δύναμη ί ν , αρκεί να βρού­με τον αριθμό υ. Σχετικά με την εύρεση του υ δια­τυπώνει την πρόταση που αναφέραμε παραπάνω.

ΑΠΆΝΤΗΣΗ Αγαπητέ Βαγγέλη . Κατ' αρχήν σε συγχαίρουμε για την εργασία σου αυτή . Συνέχισε να «ψάχνειφ, γιατί έτσι εμβαθύνεις στις διάφορες μαθηματικές έννοιες. Για να χρησιμοποιήσει ένας την πρόταση που διατυπώνεις πρέπει προηγουμένως να την α­ποδείξει. Επειδή είναι ενδιαφέρουσα θα την προ­τείνουμε ως άσκηση για λύση (βλέπε παρακάτω άσκηση 36) .

ΧΡΗΣΙΜΗ ΕΠΙΣΉΜΑΝΣΗ

Η άσκηση 30 της στήλης αυτής είναι η εξής: «Να βρείτε τις συναρτήσεις f : IR � IR , για τις οποίες, για κάθε ·χ, y Ε IR , ισχύει:

jf(x) + f(y) j = jx + yj (1)»

- Τέσσερις μαθητές και δύο μαθήτριες (από το ίδιο Λύκειο) μας έστειλαν την εξής λύση :

Από την ( 1 ) με y = χ βρίσκουμε: j 2f(x) j = j2x j => j f(x) j = jx j => [f(x) = χ ή f(x) = -χ] . Άρα, οι ζητούμενες συναρτήσεις είναι f(x) = χ και f(x) = - χ.

ΑΠΆΝΤΗΣΗ Αγαπητά μας παιδιά (Νάσια, Τατιάνα, Γιάννη, . . . ) τη σχέση j f(x) j = jx j δεν την πληρούν μόνο οι συ-ναρτήσεις που βρήκατε: f(x) = χ και f(x) = - χ. , αλλά και πολλές άλλες, όπως για παράδειγμα η συνάρτηση :

{χ, f(x) = -χ, αν χ ;:::: 1 αν χ < 1 (2)

Πράγματι, αυτή είναι ορισμένη στο IR . Εξάλλου, έχουμε: • Αν χ ;:::: l , τότε f(x) = χ και άρα jf(x) j = jx j . • Αν χ < l , τότε f(x) = -χ και άρα

jf(x) j = j-x j = jx j . Συνεπώς: j f(x) j = jx j , για κάθε χ Ε IR .

Λοιπόν, η λύση σας δεν είναι σωστή, γιατί από αυτά που γράφετε δεν μπορούμε να συμπεράνουμε ότι οι ζητούμενες συναρτήσεις είναι μόνο αυτές που βρήκατε.

Τη σωστή λύση θα την δείτε παρακάτω στις λύσεις των ασκήσεων. Είναι όμως ευκαιρία να συ­ζητήσουμε ένα πολύ «λεπτό» θέμα στα Μαθημα­τικά που νομίζουμε ότι θα σας ωφελήσει πολύ και θα σας προφυλάξει από ανάλογα σφάλματα. Πριν απ' όλα πρέπει να καταλάβετε ότι μια συνάρτηση δεν είναι ένας πραγματικός αριθμός και ότι οι συ­ναρτήσεις δεν αντιμετωπίζονται όπως οι πραγμα­τικοί αριθμοί. Μία συνάρτηση f έχει μία μεταβλη­τή χ, η οποία παίρνει τιμές από το σύνολο ορισμού της (οι τιμές της είναι πραγματικοί αριθμοί) . Έτσι,

ΕΥΚΛΕΙΔΗΣ Β' λθ ' τ.2/75

Page 76: Ευκλειδης Β 58

Η στήλη του Μαθητή

όταν γράφετε μία σχέση που περιέχει το f(x) πρέ­πει να λέτε για ποια χ ισχύει (ή θέλετε να ισχύει), εκτός αν πρόκειται για εξίσωση ή aνίσωση (τότε ζητάτε τα χ για τα οποία ισχύει). Αν δεν γράφετε για ποια χ ισχύει και το υπονοείται, τότε δυσκο­λεύετε μόνοι σας τα πράγματα. Αν όμως δεν το γράφετε και ούτε το υπονοείται, τότε όχι μόνο αρ­χίζουν οι ασάφειες, αλλά είναι σίγουρο ότι θα κά­νετε και λάθη .

Στη λύση που κάνατε (πρέπει να) υπονοείτε ότι οι συνεπαγωγές ισχύουν για κάθε χ Ε JR (γιατί αν δεν το υπονοείτε, τότε αντιμετωπίζετε τις συναρ­τήσεις σαν να ήταν πραγματικοί αριθμοί, που δεν είναι σωστό). Ας ξαναγράψουμε λοιπόν τις συνε­παγωγές αυτές βάζοντας μπροστά (αυτό που υπο­νοείτε, δηλαδή) το «για κάθε χ Ε JR » και μάλιστα, για ευκολία μας, ας το γράφουμε συμβολικά: « Vx Ε JR » . Έτσι, θα έχουμε:

( [Vx Ε JR , (f(x) = χ ή f(x) = -χ)] (3) Τώρα, αν είναι σωστό ότι από την (3) συνεπά­

γεται ότι (το κρίσιμο σημείο) : [(Vx E ffi., f(x) = x) ή (Vx E ffi., f(x) = -x)] (4)

τότε θα μπορούσαμε να συμπεράνουμε ότι οι ζη­τούμενες συναρτήσεις είναι οι: f(x) = χ και f(x) = - χ και η λύση σας θα ήταν σωστή (εξετάζο­ντας και το αντίστροφο ). Δυστυχώς όμως η (3) δεν συνεπάγεται την (4), αφού, για παράδειγμα, η πα­ραπάνω συνάρτηση (2) πληροί την (3), αλλά δεν πληροί την (4) (γι' αυτό η λύση σας δεν είναι σω­στή).

Γενικότερα, πρέπει να γνωρίζετε ότι: «Α ν για δύο συναρτήσεις f : Α � JR και

g : Α � JR ισχύει: Vx Ε A,f(x) · g(x) = Ο (5)

τότε δεν έπεται αναγκαίως ότι:

(Vx Ε A,f(x) = Ο) ή (Vx Ε Α, g(x) = Ο) (6)»

Πράγματι, ας θεωρήσουμε τις συναρτήσεις:

f(x) = {χ, Ο,

αν χ :?: 2 αν χ < 2 και g(x) = {0,

4, αν χ :?: 2 αν χ < 2

• Με χ :?: 1 , έχουμε: f(x) = χ και g(x) = Ο και άρα: f(x) · g(x) = χ · Ο = Ο .

• Με χ < 1 , έχουμε: f(x) = Ο και g(x) = 4 και άρα: f(x) · g(x) = 0 · 4 = 0 .

Έτσι, έχουμε: Vx Ε JR, f(x) · g(x) = Ο , αλλά δεν ισχύει: Vx Ε JR, f(x) = Ο , ούτε Vx Ε JR, g(x) = Ο (στη λύση που μας στείλατε, αν κάνετε όλα τα βή­ματα, θα δείτε ότι χρησιμοποιήσατε τη λανθασμένη συνεπαγωγή : (5) => (6)) .

ΤΑ ΣΥΜΒΟΛΑ ΣΤΑ ΜΛΘΗ ΙVΙ Ατt Κ Α

Ο Παναγιώτης, που είναι μαθητής της Γ τάξης του Λυκείου (δεν μας γράφει το επώνυμό του, ούτε το Λύκειό του - δεν πειράζει) μας ερωτά: «'Ή συνεπαγωγή", το "για κάθε" και το "υπάρ­χει" έχουν καταργηθεί και δεν χρησιμοποιούνται στα Μαθηματικά; Εμένα με διευκολύνει να χρη­σιμοποιώ τα σύμβολά τους (=>, V,3) , μπορώ;».

ΑΠΆΝΤΗΣΗ

Αγαπητέ Παναγιώτη . Αν βγάλουμε αυτές τις έν­νοιες από τα Μαθηματικά, τότε πρέπει να καταρ­γήσουμε όλα τα Μαθηματικά, αφού οι έννοιες αυ­τές (μαζί με εκείνη της ισοδυναμίας: <=> ) υπεισέρ­χονται παντού στα Μαθηματικά. Τα σύμβολά τους λοιπόν βγάλανε από τα σχολικά βιβλία (τα παλαιότερα σχολικά βιβλία τα είχαν) και όχι τις έννοιες τους από τα Μαθηματικά, τις οποίες άλλωστε δεν θα μπορούσαν να τις βγάλουν. Αυ­τό έγινε γιατί, όπως μας είχαν πει τότε από το Παι­δαγωγικό Ινστιτούτο, όχι γιατί δεν χρειάζονται, αλλά γιατί δεν προσπάθησαν να τα κατανοήσουν και να τα χρησιμοποιούν σωστά, με αποτέλεσμα να γράφουν το ένα σύμβολο αντί του άλλου και να τα θεωρούν ως σύμβολα στενογραφίας και να τα γράφουν μέσα στα κείμενα (με την ίδια λογική όμως θα πρέπει να aπαγορεύσουμε τις μπογιές, γιατί μερικοί γράφουν στους τοίχους και τους λερώνουν).

Πάντως, αν δεν ξέρουμε πολύ καλά τους νό­μους που διέπουν τις έννοιες αυτές (είτε χρησιμο­ποιούμε τα σύμβολά τους, είτε όχι) είναι δυνατόν

ΕΥΚΛΕΙΔΗΣ Β' λθ ' τ.2/76

Page 77: Ευκλειδης Β 58

Η στήλη του Μαθητή

σε ορισμένα ζητήματα να μην μπορούμε να απα­ντήσουμε και το χειρότερο, να κάνουμε λάθη χω­ρίς να το καταλάβουμε. Αλλά χωρίς σύμβολα είναι πολύ δύσκολο, σχεδόν αδύνατο, να μάθουμε τους νόμους αυτούς. Για παράδειγμα, χωρίς σύμβολα, είναι πολύ δύσκολο να μάθουμε την άρνηση μιας συνεπαγωγής ή τους νόμους των ποσοδεικτών ( V και Ξ3 ). Όπως είδαμε προηγουμένως, οι συμμα­θητές σου λύσανε λάθος την άσκηση 30 ακριβώς γιατί δεν ξέρανε τους νόμους των ποσοδεικτών (και πώς να τους ξέρουνε αν δεν τους πει κάποιος ποιοι είναι οι νόμοι αυτοί).

Και βέβαια μπορείς να χρησιμοποιείς τα σύμ­βολα των παραπάνω εννοιών, με την προϋπόθεση ότι θα τα χρησιμοποιείς σωστά (και μάλιστα, για να κατοχυρωθείς, πριν τα χρησιμοποιήσεις, γράψε τι εννοείς με το καθένα).

Λ Υ Σ Ε ΙΣ ΑΣ Κ J-Ι Σ ΕΩΝ

, , {(x + y)(x2 - y2 ) = 9 27. Να λυθει το συστημα: .

(x - y)(x2 + y2 ) = 5

(Επροτάθη από τη μαθήτρια Θεοδώρα Ιl ί­κουλα, Λύκειο Ραφήνας) .

Λύση (από το μαθητή Αλέξανδρο Γεω ργακόπου­λο, 52° Λύκειο Αθηνών).

Έστω (χ, y) μία λύση του συστήματος αυτού. Βρίσκουμε εύκολα ότι χ * Ο, y * Ο και χ *- y .

Θέτουμε: 2::. = λ , οπότε λ * Ο και λ * 1 . 'Ετσι, χ έχουμε y = λχ. Αντικαθιστώντας στο σύστημα, βρίσκουμε: {(χ + λχ)(χ2 - λ2χ2 ) = 9 �

(χ - λχ)(χ2 + λ2 χ2 ) = 5 {χ3 (1 + λ)2 (1 - λ) = 9 (1) ::::? χ3 (1 - λ)(l + λ2 ) = 5 (1 + λ)2 = 2. � 2λ2 - 5λ + 2 = 0 � (λ = 2 ή λ = .!_) 1 + λ2 5 2

• Έστω ότι λ = 2 . Τότε y = 2χ. Θέτοντας στην ( 1 ) λ = 2, βρίσκουμε χ3 = - 1 και άρα χ = - 1 , οπό­τε y = - 2.

'Ε ' λ 1 'Ο β ' ' 2 • στω οτι ="2 . μοια ρισκουμε οτι: χ = και y = I . Άρα, τότε (χ = -1 και y = -2) ή (χ = 2 και y = 1 ) .

Α ντιστρόφως. Όπως βρίσκουμε εύκολα τα ζεύγη (-1 , -2) και (2, 1 ) επαληθεύουν το σύστημα και άρα είναι οι μοναδικές λύσεις του. Παρατη ρή σεις σύνταξης.

I ) Η επαλήθευση που έγινε είναι απαραίτητη, για­τί δεν φθάσαμε στα ζεύγη ( -1 , -2) και (2, 1 ) με ισοδυναμίες και συνεπώς από πουθενά δεν εξα­σφαλίζεται ότι τα ζεύγη αυτά επαληθεύουν το σύστημα.

2) Τα συστήματα της μορφής αυτής ονομάζονται ομογεννοί (τα πρώτα μέλη των εξισώσεων εί­ναι του αυτού βαθμού ως προς χ και y) και για την επίλυσή τους εφαρμόζουμε συνήθως την παραπάνω μέθοδο.

• Λύσεις έστειλαν και οι μαθητές: Αντώνης Μα­κρής, Στέλιος Ιωάννου, Γιάννης Παπαντωνίου, Νίκος Ζήσης, Ιωάννα Πετροπούλου.

28. Ένα μεταβλητό ορθογώνιο τρίγωνο ΑΒΓ ( Α = 90° ) έχει την υποτείνουσα α σταθερή. Να βρείτε τη μέγιστη τιμή του εμβαδού Ε του τριγώνου αυτού. Μετά, για τη μέγιστη τιμή του Ε που θα βρείτε, να βρείτε τις οξεί­ες γωνίες του τριγώνου.

(Επροτάθη από το μαθητή Αλέξανδρο Γ�;(t) ρ­

γακόπουλο, 52° Λύκειο Αθηνών) . Λύση (από το μαθητή Γ ιάννη Σοίι �ιπλιι , Αρσά­κειο Ψυχικού, Αθήνα) .

Έχουμε: 1 α2

α2 = β2 + γ2 :::: 2βγ � α2 :::: 2βγ � -βγ :5: - � 2 4 2 Ε :5: � (το ίσον ισχύει μόνο αν β = γ). 4

2 Άρα, η μέγιστη τιμή του Ε είναι � . 4 , , , α2 , , - Εστω τωρα οτι Ε = - . Τοτε β = γ, οποτε το 4

τρίγωνο είναι και ισοσκελές. Συνεπώς, τότε, Β = r = 45° .

ΕΥΚΛΕΙΔΗΣ Β ' λθ ' τ.2/77

Page 78: Ευκλειδης Β 58

Η στήλη του Μαθητή

• Λύσεις έστειλαν και οι μαθητές: Γιάννης Πα­παντωνίου, Μαίρη Γεωργιοπούλου, Στέλιος Αντωνίου, Νίκος Θεοδωρακόπουλος, Αντώνης Πετρόπουλος.

29. Καθένας από τους αριθμούς α, β και γ είναι μεyαλύτερος του -1 και ισχύει: α + β + γ ::;; 3 . Ν α δείξετε ότι:

1 1 1 3 -- + -- + -- � - (1) 1 + α 1 + β 1 + γ 2

(Επροτάθη από το μαθητή Γιάννη Σούμπλη,

Αρσάκειο Ψυχικού - Αθήνα). Λύση (από τον ίδιο μαθητή).

Θέτουμε: Ι + α = χ, Ι + β = y και Ι + γ = ω, ο­πότε χ > Ο, y > Ο, ω > Ο και χ + y + ω = 3 + α + β + γ ::;; 3 + 3 = 6 , δηλαδή :

x + y + ω :s; 6 (2) Έτσι, για να δείξουμε την ( Ι ) αρκεί να δείξουμε

ότι: Ι Ι Ι 3 - + - + - � - (3) χ y ω 2

Από τη (2), επειδή _!_ + _.!_ + _!_ > Ο , έχουμε: χ Υ ω 6[_.!_ + _.!_ + _!_) � (χ + Υ + ω) [_!_ + _.!_ + _!_) = χ Υ ω χ Υ ω

� 3 + 2 + 2 + 2 = 9

� 6[_.!_ + _.!_ + _!_) � 9 � χ Υ ω Ι Ι Ι 9 � - + - + - � - � (3) χ y ω 6

Άλλη λύση (από το μαθητή Αλέξανδρο Γεωργα­

κόπουλο, 52° Λύκειο Αθηνών). Κατ' αρχάς θα δείξουμε ότι με x , y, z, ω ε :IR

και ω > Ο και z > Ο, ισχύει: χ 2 y2 (χ + y)2 - + - � (2) ω z ω + z

Προς τούτο, αρκεί να δείξουμε ότι:

χ 2z(ω + z) + y2ω(ω + z) � ωz(χ + y)2 , (πράξεις κτλ.) ( xz - yω )2 � Ο , ισχύει.

αρκεί

Επειδή Ι + α > Ο, Ι + β > Ο και Ι + γ > Ο, σύμ-φωνα με τη (2), έχουμε:

Ι Ι Ι Ι 2 Ι 2 Ι -- + -- + -- = -- + -- + -- � Ι + α Ι + β Ι + γ Ι + α Ι + β Ι + γ (l + I)2 Ι 22 Ι 2

_ __:__,;,..__ + -- = + -- � (Ι + α) + (Ι + β) Ι + γ 2 + α + β Ι + γ (2 + Ι)2 = 9 (3) (2 + α + β) + (Ι + γ) 3 + α + β + γ

Εξάλλου, έχουμε: α + β + γ ::;; 3 � 3 + α + β + γ ::;; 6 (και επειδή

Ι Ι 3 + α + β + γ > Ο) � � - . 3 + α + β + γ 6 Έτσι, από τις (3), έχουμε:

Ι 1 Ι 9 9 3 -- + -- + -- � � - = -Ι + α l + β l + γ 3 + α + β + γ 6 2

30. Να βρείτε τις συναρτήσεις f : IR � IR , για τις οποίες, για κάθε χ, y ε IR , ισχύει:

lf(x) + f(y)l = l x + Y l (1).

(Επροτάθη από το μαθητή Σπύρο Λεονάρδο,

Κολλέγιο Αθηνών). Λύση (από το μαθητή Γιώργο Σούμπλη, Αρσά­κειο Ψυχικού, Αθήνα).

Έστω ότι για μία συνάρτηση f : IR � IR η ( Ι ) ισχύει. Από την ( 1 ) με y = χ, βρίσκουμε ότι για κάθε χ ε IR ισχύει:

Από την ( Ι ) έχουμε για κάθε χ, y ε IR : lf(x) + f(y) l 2 = lx + y l 2 � [f(x)+f(y)]2 =(x+y/ �

(2 ) f2 (x) + f2 (y) + 2f(x)f(y) = χ 2 + y2 + 2xy� f(x)f(y) = xy (3)

Από τη (2) με χ = Ι , βρίσκουμε f( l ) = Ι και άρα: f( l ) = Ι ή f( l ) = -Ι .

Από την (3 ) με χ ε IR και y = Ι , βρίσκουμε f(x)f( l ) = χ. Έτσι, αν f( l ) = Ι , τότε f(x) = χ, για κάθε χ ε :ΙR και αν f(x) = -Ι , τότε f(x) = - χ, για κάθε χ ε IR .

ΕΥΚΛΕΙΔΗΣ Β ' λθ ' τ.2/78

Page 79: Ευκλειδης Β 58

Α ντιστρόφως. Όπως βρίσκουμε εύκολα, οι συ­ναρτήσεις: f(x) = χ και f(x) = - χ πληρούν τις δο­σμένες συνθήκες και άρα είναι οι μοναδικές ζη­τούμενες. Σχόλιο Σύνταξης: Θα ήταν λάθος από τη (2), δη­λαδή από την f\x) = χ2, να συμπεράνουμε ότι f(x) = χ για κάθε χ Ε IR. ή f(x) = - χ για κάθε χ Ε 1R. (γιατί;) • Λύσεις έστειλαν και οι μαθητές: Κων/νος Πα­

παγεωργίου, Αλέξανδρος Γεωργακόπουλος. 31 . Να βρείτε (όχι Γεωμετρικά) τους αριθμούς

z Ε C , για τους οποίους ισχύουν:

lz + 1 - ψ : ; 3 (1) και lz - 3 + 2i l s; 2 (2).

(Επροτάθη από το μαθητή Γεώργιο Σούμπλη,

Αρσάκειο Ψυχικού - Αθήνα). Λύση (από το μαθητή Κων/νο Παπαγεωργίου,

Εκπαιδευτήρια «0 Πλάτων», Αθήνα). Έστω ότι για ένα αριθμό z Ε C οι σχέσεις ( 1 )

και (2) ισχύουν. Από αυτές βρίσκουμε ότι: 3 + 2 ;?: lz + 1 - i l + l z - 3 + 2i l ;?:

;::: l<z + 1 - i) - (z - 3 + 2i) l =

= 14 - 3i l = 5 => 5 ;::: lz + 1 - i l + lz - 3 + 2i l ;::: 5

=> lz + 1 - i l + lz - 3 + 2i l = 5 (3)

Λόγω της (3), οι ( 1 ) και (2) ισχύουν με το ίσον (γιατί;) . Έτσι έχουμε:

lz + 1 - i l = 3 (4) και lz - 3 + 2i l = 2 (5) Θέτουμε z = χ + yi, όπου χ, y Ε IR. . Έτσι, από

τις (4) και (5), έχουμε: => ::::> \ { lx + yi + 1 - i l = 3 { l (x + 1) + (y - 1)i l = 3

lx + yi - 3 + 2i l = 2 Ι< χ - 3) + (y + 2)i l = 2 {(x + 1)2 + (y - 1)2 = 9 => (χ - 3)2 + (y + 2)2 = 4

{χ2 + y2 + 2χ - 2y - 7 = ο => χ2 + y2 - 6x + 4y + 9 = 0

{8χ - 6y - 1 6 = ο χ 2 + / + 2x - 2y - 7 = 0

y = 4 ·--3

! χ - 2

=> 2 => χ - 2 χ - 2 χ 2 + 1 6 (-3-) + 2χ - 8-3- - 7 = 0 { χ - 2 { χ - 2 y = 4 ·-- y = 4 ·--3 => 3 25χ2 - 70χ + 49 = 0 (5χ - 7)2 = 0

Ά ' 7 4 . ρα, τοτε: z = - - -ι . 5 5 Αντιστρόφως. Όπως βρίσκουμε εύκολα, ο α-

ριθμός z = 2 - _i i επαληθεύει τις σχέσεις ( 1 ) και 5 5 (2) και άρα είναι ο μοναδικός ζητούμενος. • Λύσεις έστειλαν και οι μαθητές: Αλέξανδρος

Γεωργακόπουλος, Νίκος Παυλόπουλος, Ηρα­κλής Δεσφινιώτης, Σπύρος Λεονάρδος.

32. Για τους αριθμούς z,ω Ε C και ν Ε Ν * ισχύουν :

zv = 3 + 4i (1) και ων = 4 + 3i (2). Να δείξετε ότι:

α. Ο αριθμός !:_ είναι γνήσιος μιγαδικός. ω

β ο θ , z + ω , , . αρι μος φ = -- ειναι φανταστικος. z - ω

(Επροτάθη από το μαθητή Κων/νο Παπαγε­

ωργίου, Εκπαιδευτήρια «0 Πλάτων», Αθήνα). Λύση (από τον ίδιο μαθητή).

'Ε · z λ · λ lJ]) * τ · α. στω οτι - = , οπου Ε m,. • οτε: ω z = λω ::::> zv = λν · ων => 3 + 4i = λv (4 + 3i ) =>

=> - = - => 9 = 1 6 , άτοπο. {3 = 4λν 3 4 4 = 3λν 4 3

Άρα, ο αριθμός !:._ είναι γνήσιος μιγαδικός. ω

ΕΥΚΛΕΙΔΗΣ Β ' λθ ' τ.2/79

Page 80: Ευκλειδης Β 58

Έτσι έχουμε: 2

I I I 1 2 2 - 2 - ρ z = ρ � z = ρ � zz = ρ � z = - ο 2

Ομοια: ω = � . 'Εχουμε: ω ρ2 ρ2

- - - + -

z

- z + ω z + ω z ω ω + z φ = = = =--=- = 2 2 = -- = -φ � z - ω z - ω ρ ρ ω - z - - -

z ω φ = -φ � ( φ φανταστικός) .

• Λύσεις έστειλαν και οι μαθητές: Αλέξανδρος Γεωργακόπουλος, Γιώργος Σούμπλης, Αντώνης Μακρής, Σπύρος Λεονάρδος.

ΑΣ Κ Η Σ Ε Ι Σ Π Α Λ ΥΣΗ

33 . Θεωρούμε, ένα τρίγωνο ΑΒΓ με Γ = 2Β , ένα εσωτερικό του σημείου Μ και υποθέτουμε ότι ΑΜ = ΑΓ και ΜΒ = ΜΓ. Να δείξετε ότι: ΜΑΒ = .!.Α ο 3

(Προτείνεται από το μαθητή Γιάννη Σούμπλη,

Αρσάκειο Ψυχικού, Αθήνα).

34. Με α, β και γ πραγματικούς αριθμούς και ν Ε Ν* να δείξετε ότι, αν η ισότητα:

( α + β + γ )2 ν+ ι = α2 ν+ ι + β2 ν+ ι + γ2ν+ ι

ισχύει για ν = 1 , τότε ισχύει και για κάθε ν Ε Ν* .

(Προτείνεται από τη μαθήτρια Εβίτα Αντωνίου,

Ιταλική Σχολή, Αθηνών).

35. Να βρείτε τους ακέραιους θετικούς αριθμούς χ και y, για τους οποίους ισχύει:

2χ + y + 3Jry - sFx - 2)Υ - 3 = ο .

(Προτείνεται από τη μαθήτρια Ν ικολέτα Φεσσά,

Λύκειο Νέας Χαλκηδόνας).

36. Έστω ένας φυσικούς αριθμός με κ ψηφία: 1 ο κ-ι 1 οκ-2 ν = yκ- ι Υκ-2 " ' "Υ2Υι Υο = Υκ- ι + Υκ-2 +

+ . . . + yι l O + yo , όπου y κ- ι , y κ-2 , . . . , Υ ι , y0 είναι τα ψηφία του, δηλαδή αριθμοί του συνόλου: {0, 1 , 2, . . . , 9 } . Να δείξετε ότι οι δύο διαιρέσεις ν: 4 και (2y ι + Υο) : 4 έχουν το ίδιο υπόλοιπό.

Εφαρ�ιογή. Να βρείτε τον αριθμό: z = i885307856725987 + i 32 ι 795055725 ι 762 ι s (όπου i η φανταστική μονάδα).

(Προτείνεται από το μαθητή Βαγγέλη Π ρωτοπα­

παδάκη, 3° Λύκειο Χανιών Κρήτης) .

37. Να δείξετε ότι η συνάρτηση : f ( χ ) = αχ4 + βχ2 + γχ + δ

-3 όπου α, β, γ, δ Ε JR με α :;t Ο και α + β = -δ , 2 έχει μία τουλάχιστον ρίζα στο διάστημα [-1 , 1 ] .

(Προτείνεται από το μαθητή Γεώργιο Σούμπλη,

Αρσάκειο Ψυχικού, Αθήνα) .

38. Μία συνάρτηση f είναι ορισμένη και συνεχής στο διάστημα [0, 5 ] . Να δείξετε ότι υπάρχει Χ0 Ε (0, 5 ) με

8f ( χ0 ) = 3f ( l ) + 4f (3 ) + f ( 4) .

(Προτείνεται από το μαθητή Κων/νο Παπαγεωρ­

γίου, Εκπαιδευτήρια «0 Πλάτων», Αθήνα).

39. Να βρείτε τις συνεχείς συναρτήσεις f : JR � JR , για τις οποίες ισχύει f( 1 ) > 1 και f2 ( Χ ) - 2xf ( Χ ) - ex = 0 , για κάθε Χ Ε JR .

(Προτείνεται από το μαθητή Σπίφο Λεονάρδο,

Κολλέγιο Αθηνών).

ΕΥΚΛΕΙΔΗΣ Β ' λθ ' τ.2/80

Page 81: Ευκλειδης Β 58

,

Ί από τις εκδόσεις <<Εν Δυνάμει>>

Για την Γ' Λυκείου • Φυσική Κcιτεύβυνσπς (Τ'αλαντώσεις - Κύματα) - Γ. Θ. Ντούβαλης

• Φυσική Κατεύ8υνσπς (Μηχανική του στερεού σώματος) - Γ. Θ. Ντούβαλης

• Φυσική Κατεύβυνσπς (Κρούσεις - Φαινόμενο Doppler) - Γ. Θ. Ντούβαλης

• Ασκήσεις Βιολογίας Γενικής Παιδείας - Κ. Ρ. Παπαζήσης

• Μα8πμcmκά Κατεύ8υνσπς (Παράγωγοι) - Γ. & Π. Λοuκόποuλος

• Μαβnματικά Κατεύβυνσπς (Ολοκληρώματα) - Γ. & Π. Λοuκόποuλος

··εκφρασπ -·Εκ8εαπ - Α. Καλλή - Γ. Σοuλτάνης

• Ανάπτυξη eφαρμογcίιν σε προγραμμσnστικό περιβάλλον - Κ. Ν. Ιορδανόποuλος

• Αρχές Οικονομικής Θεωρίας - Κ. Γαροφαλάκης

Για την Α' Λυκείου • Φυσική - Κ. Ρ. Παπαζήσης

• 'Άλγεβρα - Γ. & Π. Λοuκόποuλος

Ειι Δυιιόμ&ι Ε Κ Δ Ο Σ Ε Ι Σ

ΧΡΥΣΙΠΠΟΥ 1 & ΟΥΛΟΦ ΠΑΛΜΕ - ΖΩΓΡΑΦΟΥ Τηλ. : 2 1 Ο 74 88 030. fax: 2 1 Ο 74 83 03 1

Page 82: Ευκλειδης Β 58

Μαθηματικά

Μαθηματικά Κατεύθυνσης Γ' Λυκείου (2 τεύχη) Α. τ ρaγavίτης

Α π ό τ ι s ε κδόσειs Σα Β Β ά λ α s κ υ κ λ ο φ ο ρ ο ύ ν

ΜοΒnumικίι r· ΑΥΚΕΙΟΥ 8

Μαθηματικά Γ' λυκείου Γ ενικής Παιδείας Χ & I. Στεργίοu - Χ Νάκης

Μαθηματική Ανάλυση (τεύχος Α') Θ. Μ. Ρaσσιάς

Μαθηματική Ανάλυση (τεύχος Β') Θ. Μ. Ρασσιάς

Β ι β λ ία π ο υ οξύνουν ο νου

Λ.Τ.: 9,70 € Λ.Τ.: 1 6,70 € Λ.Τ.: 1 6,70 € Λ. Τ.: 1 4,70 €

Λ.Τ.: 1 9,70 €

Λ.Τ.: 9,70 € Λ.Τ.: 9,70 €

www .savalas.gr Αθήνα: Ζ. Πηγήc; 1 8, I 06 8 1 , τηλ.: 2 1 Ο 330 1 25 1 Θεσσαλονίκη: Β. Ηρακλείου 47, τηλ: 23 1 Ο 270 226